Player FM - Internet Radio Done Right
137 subscribers
Checked 2M ago
Vor neun Jahren hinzugefügt
Inhalt bereitgestellt von Smarty PANCE | The PA Life. Alle Podcast-Inhalte, einschließlich Episoden, Grafiken und Podcast-Beschreibungen, werden direkt von Smarty PANCE | The PA Life oder seinem Podcast-Plattformpartner hochgeladen und bereitgestellt. Wenn Sie glauben, dass jemand Ihr urheberrechtlich geschütztes Werk ohne Ihre Erlaubnis nutzt, können Sie dem hier beschriebenen Verfahren folgen https://de.player.fm/legal.
Player FM - Podcast-App
Gehen Sie mit der App Player FM offline!
Gehen Sie mit der App Player FM offline!
Podcasts, die es wert sind, gehört zu werden
GESPONSERT
<
<div class="span index">1</div> <span><a class="" data-remote="true" data-type="html" href="/series/action-academy-replace-the-job-you-hate-with-a-life-you-love">Action Academy | Replace The Job You Hate With A Life You Love</a></span>


Ready to replace your 6-figure salary with real freedom? This is the podcast for high earners who feel stuck in jobs they’ve outgrown. If you’re asking, “How do I actually replace $10K–$20K/month so I can quit and never look back?” — welcome home. At Action Academy, we teach you how to buy small businesses and commercial real estate to create cash flow that actually replaces your job. Monday through Friday, you’ll learn from 7–9 figure entrepreneurs, real estate moguls, and acquisition pros who’ve done it — and show you how to do it too. Hosted by Brian Luebben (@brianluebben), who quit his 6-figure sales role in 2022 to build a global business while traveling the world. If you're a high-income earner ready to become a high-impact entrepreneur, this show is your playbook. Subscribe now and start your path to freedom — or keep pretending your job will get better someday....
The Audio PANCE and PANRE Physician Assistant Board Review Podcast
Alle als (un)gespielt markieren ...
Manage series 97199
Inhalt bereitgestellt von Smarty PANCE | The PA Life. Alle Podcast-Inhalte, einschließlich Episoden, Grafiken und Podcast-Beschreibungen, werden direkt von Smarty PANCE | The PA Life oder seinem Podcast-Plattformpartner hochgeladen und bereitgestellt. Wenn Sie glauben, dass jemand Ihr urheberrechtlich geschütztes Werk ohne Ihre Erlaubnis nutzt, können Sie dem hier beschriebenen Verfahren folgen https://de.player.fm/legal.
Multiple Choice Physician Assistant Board Review and Rotation Exam Questions on the Go – Listen and Learn
…
continue reading
69 Episoden
Alle als (un)gespielt markieren ...
Manage series 97199
Inhalt bereitgestellt von Smarty PANCE | The PA Life. Alle Podcast-Inhalte, einschließlich Episoden, Grafiken und Podcast-Beschreibungen, werden direkt von Smarty PANCE | The PA Life oder seinem Podcast-Plattformpartner hochgeladen und bereitgestellt. Wenn Sie glauben, dass jemand Ihr urheberrechtlich geschütztes Werk ohne Ihre Erlaubnis nutzt, können Sie dem hier beschriebenen Verfahren folgen https://de.player.fm/legal.
Multiple Choice Physician Assistant Board Review and Rotation Exam Questions on the Go – Listen and Learn
…
continue reading
69 Episoden
Alle Folgen
×T
The Audio PANCE and PANRE Physician Assistant Board Review Podcast


Listen to Podcast Episode 109: Ten PANCE, PANRE, and Rotation Review Questions If you can’t see the audio player, click here to listen to the full episode. Welcome to episode 109 of the Audio PANCE and PANRE Physician Assistant/Associate (PA) Board Review Podcast. Join me today as we cover ten board review questions for your PANCE, PANRE, EOR, and EOC exams. Resources and links from today’s episode: View my PANRE-LA Performance Score and see my process for taking PANRE-LA Questions Learn all about the new Smarty PANCE QBank Sign up for our PANCE and PANRE Test-taking Masterclass Sign up for the Entire Blueprint Email Series Follow Smarty PANCE and The Daily PANCE Blueprint on Instagram and Facebook Join the Smarty PANCE Member’s Community , then sign up for a study group to get updates about upcoming webinars I hope you enjoy this free audio component of the examination portion of this site. Smarty PANCE includes over 2,000 interactive board review questions, along with flashcards, ReelDx cases , integrated Picmonics , and lessons covering every blueprint topic available to all Smarty PANCE members . You can download and listen to past FREE episodes here , on iTunes , Spotify , Amazon Music , and all podcasting apps. On each episode page, you can listen to all the latest episodes, take interactive quizzes, and download more resources. Interactive Exam to Complement Today’s Podcast 1. A 70-year-old man presents for evaluation of poorly controlled hypertension despite adherence to five different classes of antihypertensive medications. His medical history includes coronary artery disease with a myocardial infarction 3 years ago, peripheral artery disease, and type 2 diabetes mellitus. He reports no changes in his medication regimen and denies any recent lifestyle modifications. Physical examination reveals a blood pressure of 190/110 mm Hg in the left arm and 180/100 mm Hg in the right arm. His BMI is 24 kg/m². On auscultation, there is no abnormal heart sound. Which of the following additional findings is most likely to be seen in this patient? A. Elevated serum aldosterone-to-renin ratio B. Continuous murmur over the carotid arteries C. Diminished femoral pulses D. Continuous bruit in the lateral periumbilical area E. Pitting edema in the lower extremities Answer and topic summary The answer is D. Continuous bruit in the lateral periumbilical area Renal artery stenosis is a significant cause of secondary hypertension, particularly in patients with a history of atherosclerotic disease, as seen in this patient with coronary artery disease and peripheral artery disease. Resistant hypertension, uncontrolled despite multiple medications, is a hallmark. A continuous bruit in the lateral periumbilical area indicates turbulent blood flow through a narrowed renal artery, supporting this diagnosis. Incorrect Answers: A. Elevated aldosterone-to-renin ratio suggests primary aldosteronism, less likely given the atherosclerotic history pointing to renal artery stenosis. B. A carotid murmur indicates carotid artery disease, unrelated to this hypertension scenario. C. Diminished femoral pulses align with peripheral artery disease, but don’t explain the resistant hypertension. E. Pitting edema suggests heart failure or venous insufficiency, not specific to renal artery stenosis. Smarty PANCE Content Blueprint Review: Covered under ⇒ PANCE Blueprint Renal System ⇒ Congenital or structural renal disorders ⇒ Renal vascular disease 2. A 70-year-old woman comes to the emergency department due to sudden-onset chest pain followed by shortness of breath. The pain started 2 hours ago, and she experienced an episode of syncope lasting about 1 minute shortly after the pain began. She has a history of untreated hypertension and hyperlipidemia. Blood pressure is 180/95 mm Hg in both arms; pulse is 110/min and irregular; and respirations are 22/min. Pulses are symmetric and equal in all extremities. There is an S4 gallop. ECG shows atrial fibrillation with rapid ventricular response. Serum troponin is elevated. Chest x-ray shows an enlarged cardiac silhouette, and a CT scan confirms aortic dissection. Which of the following is the best next step in management? A. Administration of aspirin and nitroglycerin B. Immediate thrombolytic therapy C. Initiation of anticoagulation with heparin D. Emergency pericardiocentesis E. Emergent aortic repair surgery Answer and topic summary The answer is E. Emergent aortic repair surgery This patient’s sudden chest pain, syncope, hypertension, and CT-confirmed aortic dissection necessitate emergent surgical repair to prevent catastrophic complications like rupture. The enlarged cardiac silhouette and symptoms align with this diagnosis, making surgery the priority. Incorrect Answers: A. Aspirin and nitroglycerin treat acute coronary syndromes, but nitroglycerin could worsen dissection by lowering blood pressure. B. Thrombolytics are contraindicated due to bleeding risk in dissection. C. Heparin is inappropriate in acute dissection due to bleeding risk, despite atrial fibrillation. D. Pericardiocentesis addresses tamponade, a possible complication, but isn’t the primary treatment for dissection. Smarty PANCE Content Blueprint Review: Covered under ⇒ PANCE Blueprint Cardiology ⇒ Vascular Disease ⇒ Aortic aneurysm/dissection 3. A 25-year-old woman with a history of frequent sinus infections presents to the clinic for evaluation of ear pain. She recently returned from a vacation involving several flights. During the descent of her last flight, she experienced severe pressure and pain in her ears, followed by fullness and reduced hearing in her right ear. She has tried decongestants and nasal sprays without relief. On examination, the tympanic membrane appears retracted and immobile on pneumatic otoscopy. Which of the following is the most likely diagnosis? A. Acute otitis media B. Otitis externa C. Eustachian tube dysfunction D. Chronic otitis media E. Tympanic membrane perforation Answer and topic summary The answer is C. Eustachian tube dysfunction Eustachian tube dysfunction is likely due to pressure changes during flight descent, causing ear pain, fullness, and hearing reduction. The retracted, immobile tympanic membrane on otoscopy confirms impaired pressure equalization, common after air travel in those with sinus issues. Incorrect Answers: A. Acute otitis media typically shows a bulging tympanic membrane and fever, not retraction. B. Otitis externa affects the ear canal, with pain on outer ear movement, not tympanic retraction. D. Chronic otitis media involves persistent inflammation and perforation, not acute travel-related symptoms. E. Perforation would show a hole in the tympanic membrane, not retraction. Smarty PANCE Content Blueprint Review: Covered under ⇒ PANCE Blueprint EENT ⇒ Ear Disorders (PEARLS) ⇒ Inner ear (PEARLS) ⇒ Dysfunction of the eustachian tube 4. A 68-year-old man is brought to the emergency department due to sudden difficulty with speech and comprehension. He speaks fluently, but his sentences are nonsensical, and he appears frustrated. He cannot understand simple commands, repeat phrases, or find the right words. Which branch occlusion of the following arteries is most likely responsible for this condition? A. Anterior cerebral artery B. Posterior cerebral artery C. Middle cerebral artery D. Basilar artery E. Vertebral artery Answer and topic summary The answer is C. Middle cerebral artery Middle cerebral artery (MCA) occlusion in the dominant hemisphere causes Wernicke’s aphasia, marked by fluent, nonsensical speech and poor comprehension, as seen here. The MCA supplies the lateral frontal, temporal, and parietal lobes, critical for language processing. Incorrect Answers: A. Anterior cerebral artery affects leg strength and personality, not aphasia. B. Posterior cerebral artery causes visual deficits, not language issues. D. Basilar artery impacts the brainstem, causing cranial nerve deficits, not aphasia. E. Vertebral artery affects the cerebellum and brainstem, not language areas. Smarty PANCE Content Blueprint Review: Covered under ⇒ PANCE Blueprint Neurology ⇒ Vascular Disorders ⇒ Stroke 5. A 30-year-old woman presents to the emergency department with right wrist pain after falling on an outstretched hand while playing tennis. She reports immediate pain and swelling. On examination, there is tenderness in the anatomical snuffbox and pain with axial loading of the thumb. X-ray confirms a fracture. What is the most likely diagnosis? A. Distal radius fracture B. Scaphoid fracture C. Colles’ fracture D. Lunate dislocation E. Wrist sprain Answer and topic summary The answer is B. Scaphoid fracture A scaphoid fracture, common after a fall on an outstretched hand, presents with snuffbox tenderness and pain on thumb loading. X-ray confirmation aligns with this diagnosis, though initial films may miss it, requiring follow-up imaging if suspected. Incorrect Answers: A. Distal radius fracture causes diffuse wrist pain, not isolated snuffbox tenderness. C. Colles’ fracture, a distal radius type, shows dorsal displacement, not snuffbox-specific signs. D. Lunate dislocation impairs wrist function broadly, with distinct X-ray findings. E. Wrist sprain lacks fracture-specific tenderness and X-ray evidence. Smarty PANCE Content Blueprint Review: Covered under ⇒ PANCE Blueprint Musculoskeletal ⇒ Upper Extremity Disorders ⇒ Fractures and dislocations of the forearm, wrist, and hand 6. A 45-year-old man presents to the emergency department with severe left lower abdominal pain starting suddenly two hours ago. He reports nausea and vomiting but denies prior episodes. Examination shows localized tenderness and guarding in the left lower quadrant. Vital signs are stable. Labs reveal leukocytosis with a left shift. CT scan is on order. What is the most likely diagnosis? A. Acute diverticulitis B. Acute appendicitis C. Renal colic D. Small bowel obstruction E. Acute cholecystitis Answer and topic summary The answer is A. Acute diverticulitis Acute diverticulitis typically presents with sudden left lower quadrant pain, nausea, vomiting, and leukocytosis. CT findings like pericolic fat stranding confirm this common sigmoid colon condition in adults. Incorrect Answers: B. Appendicitis causes right lower quadrant pain, not left. C. Renal colic presents with flank-to-groin pain and hematuria, not quadrant-specific tenderness. D. Small bowel obstruction shows crampy pain and distension, with distinct CT findings. E. Cholecystitis causes right upper quadrant pain, not left lower. Smarty PANCE Content Blueprint Review: Covered under ⇒ PANCE Blueprint GI and Nutrition ⇒ Colorectal disorders ⇒ Diverticular disease 7. A 50-year-old man presents with red-colored urine for 3 days. He denies pain during urination but has mild hypertension managed with lifestyle changes. Vital signs are stable (BP 125/80 mm Hg, HR 78/min). Urinalysis shows numerous red blood cells and casts, but no white cells or bacteria. What is the best next step in management? A. Abdominal ultrasound B. Measurement of 24-hour urinary protein excretion C. Reassurance and no additional workup D. Urine culture E. Cystoscopy Answer and topic summary The answer is B. Measurement of 24-hour urinary protein excretion Red blood cell casts and hematuria suggest glomerulonephritis. Measuring 24-hour urinary protein excretion assesses proteinuria severity, aiding diagnosis of glomerular disease, as dipstick tests may miss subtle changes. Incorrect Answers: A. Ultrasound evaluates non-glomerular causes like stones, not initial for glomerular hematuria. C. Reassurance is inappropriate with casts indicating pathology. D. Urine culture is unnecessary without infection signs (no WBCs/bacteria). E. Cystoscopy targets lower tract issues, not glomerular causes suggested by casts. Smarty PANCE Content Blueprint Review: Covered under ⇒ PANCE Blueprint Renal System ⇒ Acute kidney injury (ReelDx + Lecture) 8. A 6-year-old boy presents with swelling and pain in his left knee after a fall. He has had similar episodes after minor injuries. Examination shows a swollen, tender knee with limited motion, no bruising or petechiae. Labs reveal prolonged PTT and normal platelet count. Which additional finding is most likely upon further history and examination? A. Hematuria B. Frequent nosebleeds C. Sister with heavy menstrual bleeding D. Maternal grandfather with bleeding history E. Petechiae Answer and topic summary The answer is D. Maternal grandfather with history of bleeding following minor trauma Hemophilia, an X-linked recessive disorder, causes hemarthrosis after minor trauma, with prolonged PTT and normal platelets. A maternal grandfather with bleeding history supports this genetic pattern. Incorrect Answers: A. Hematuria occurs less often than joint bleeding in hemophilia. B. Nosebleeds are less typical than hemarthrosis in hemophilia. C. Heavy menstrual bleeding suggests von Willebrand disease, not hemophilia (male-predominant). E. Petechiae indicate platelet issues, not clotting factor deficiencies like hemophilia. Smarty PANCE Content Blueprint Review: Covered under ⇒ PANCE Blueprint Hematology ⇒ Coagulation Disorders ⇒ Clotting factor disorders Also covered on the Internal Medicine EOR and Emergency Medicine EOR topic lists 9. A 19-year-old man is admitted with progressive lower extremity weakness. He had tingling in his toes 7 days ago, now with difficulty walking, swallowing, and facial weakness. He had a GI illness 3 weeks ago. Neurologic exam shows 1/5 lower and 3/5 upper extremity strength, absent lower reflexes, and decreased upper reflexes. He struggles to hold his head up. What is the best next step in management? A. Intravenous immunoglobulin therapy B. Mechanical ventilation C. Spinal MRI D. Muscle biopsy E. Lumbar puncture Answer and topic summary The answer is B. Mechanical ventilation Guillain-Barré syndrome (GBS) presents with ascending weakness and areflexia post-infection. Difficulty swallowing and facial weakness signal respiratory compromise, necessitating immediate mechanical ventilation to prevent failure. Incorrect Answers: A. IVIG treats GBS but isn’t urgent over respiratory support. C. Spinal MRI aids diagnosis but isn’t immediate priority. D. Muscle biopsy is irrelevant for GBS management. E. Lumbar puncture supports diagnosis but delays critical respiratory intervention. Smarty PANCE Content Blueprint Review: Covered under ⇒ PANCE Blueprint Neurology ⇒ Peripheral Nerve Disorders ⇒ Guillain-Barré syndrome 10. A 25-year-old woman presents with severe, sharp chest pain for 48 hours, radiating to her left shoulder. It worsens lying down or with deep breaths, improves sitting forward. She had a recent upper respiratory infection. Temperature is 37.9°C (100.2°F), ECG shows diffuse ST elevations. What is the most appropriate initial treatment? A. Acetaminophen and antibiotics B. Ibuprofen and colchicine C. Acetaminophen and steroids D. Ibuprofen and steroids E. Steroids and antibiotics Answer and topic summary The answer is B. Ibuprofen and colchicine Acute pericarditis, often viral post-infection, causes sharp chest pain relieved by sitting forward, with diffuse ST elevations on ECG. Ibuprofen (NSAID) and colchicine reduce inflammation and prevent recurrence as first-line treatment. Incorrect Answers: A. Acetaminophen lacks anti-inflammatory effect; antibiotics aren’t indicated for viral etiology. C. Steroids aren’t first-line due to recurrence risk; acetaminophen is less effective. D. Ibuprofen is correct, but steroids aren’t initial therapy. E. Steroids and antibiotics are not standard first-line for viral pericarditis. Smarty PANCE Content Blueprint Review: Covered under ⇒ PANCE Blueprint Cardiology ⇒ Traumatic, infectious, and inflammatory heart conditions (PEARLS) ⇒ Acute pericarditis This Podcast is Available on Every Device! Download your FREE copy of the PANCE/PANRE/EOR Content Blueprint Checklists . Print it up and start crossing out the topics you understand, marking the ones you don’t, and making notes of key terms you should remember. The PDF version is interactive and linked directly to the individual lessons on Smarty PANCE. Smarty PANCE is not sponsored or endorsed by, or affiliated with, the National Commission on Certification of Physician Assistants.…
T
The Audio PANCE and PANRE Physician Assistant Board Review Podcast


Listen to Podcast Episode 108: Ten PANCE, PANRE, and Rotation Review Questions If you can’t see the audio player, click here to listen to the full episode. Welcome to episode 108 of the Audio PANCE and PANRE Physician Assistant/Associate (PA) Board Review Podcast. Join me today as we cover ten board review questions for your PANCE, PANRE, EOR, and EOC exams. Resources and links from today’s episode: Sign up for our new PANCE and PANRE Test-taking Masterclass Review the Asthma GINA guidelines and the lung cancer screening guidelines Sign up for the Entire Blueprint Email Series Follow Smarty PANCE and The Daily PANCE Blueprint on Instagram and Facebook Join the Smarty PANCE Member’s Community , then sign up for a study group to get updates about upcoming webinars. I hope you enjoy this free audio component of the examination portion of this site. Smarty PANCE includes over 2,000 interactive board review questions, along with flashcards, ReelDx cases , integrated Picmonics , and lessons covering every blueprint topic available to all Smarty PANCE members . You can download and listen to past FREE episodes here , on iTunes , Spotify , Amazon Music , and all podcasting apps. On each episode page, you can listen to all the latest episodes, take interactive quizzes, and download more resources. Interactive exam to complement today’s podcast 1. A 25-year-old woman comes to the office due to fatigue and dizziness for the past several months. She works as a nurse and finds it increasingly difficult to complete her shifts. The patient reports heavy menstrual periods lasting 7-8 days each month. She has no significant medical history and is not on any medications. Blood pressure is 100/60 mm Hg and pulse is 75/min. BMI is 22 kg/m². Physical examination reveals pale conjunctivae and spoon-shaped nails. Hemoglobin is 8.5 g/dL. Which of the following sets of additional laboratory findings is most likely to be seen in this patient? A. Low ferritin, high TIBC, low serum iron B. High ferritin, low TIBC, high serum iron C. Low ferritin, low TIBC, high serum iron D. High ferritin, high TIBC, low serum iron E. Low ferritin, high TIBC, high serum iron Answer and topic summary The answer is A. Low ferritin, high TIBC, low serum iron The most likely additional laboratory findings in this patient are low ferritin, high total iron-binding capacity (TIBC), and low serum iron, which are indicative of iron deficiency anemia. Her history of heavy menstrual periods and symptoms of fatigue and pallor suggest chronic blood loss leading to iron deficiency. Ferritin is a marker of iron stores, and low levels indicate depletion of iron reserves. High TIBC reflects increased capacity of the blood to bind iron due to low iron levels. Smarty PANCE Content Blueprint Review: Covered under ⇒ PANCE Blueprint Hematology ⇒ Cytopenias ⇒ Anemias ⇒ Iron deficiency 2. A 4-year-old girl presents with fever and neck pain. She has had a runny nose, cough, and sore throat for the past five days. Two days ago, she developed a high fever and worsening neck pain, and today, she has refused to eat or drink. Her immunizations are up to date. Temperature is 39.5°C (103.1°F), pulse is 130/min, and respirations are 26/min. Examination reveals bilateral anterior cervical lymphadenopathy, a muffled voice, and trismus. The child holds her neck stiffly and resists any movement. The tonsils are erythematous and covered with white exudates. Lateral neck radiograph reveals a widened prevertebral space. Which of the following is the most likely diagnosis? A. Epiglottitis B. Peritonsillar abscess C. Retropharyngeal abscess D. Bacterial tracheitis E. Acute bacterial sinusitis Answer and topic summary The answer is C. Retropharyngeal abscess Retropharyngeal abscess is the most likely diagnosis given the presentation of fever, neck pain, refusal to eat or drink, trismus, and a widened prevertebral space on lateral neck radiograph. This condition commonly occurs in young children and can result from a spread of infection from the upper respiratory tract. The stiff neck and resistance to movement are classic signs of a deep neck space infection such as a retropharyngeal abscess. Smarty PANCE Content Blueprint Review: Covered under ⇒ PANCE Blueprint EENT ⇒ Oropharyngeal disorders ⇒ Diseases of the teeth and gums => Infectious and inflammatory disorders => Deep neck infection 3. A 30-year-old woman comes to the office due to left hand and wrist pain after tripping and falling onto an outstretched left hand while jogging. Examination shows severe tenderness to palpation of the radial dorsal aspect of the wrist. Range of motion and sensation are preserved. X-ray of the wrist is normal. Which of the following is the best next step in management of this patient? A. Immediate MRI of the wrist B. Application of a thumb spica splint and follow-up in 1-2 weeks C. Reassurance and advise rest and ice D. Referral to an orthopedic surgeon E. Repeat X-ray in 1 week Answer and topic summary The answer is B. Application of a thumb spica splint and follow-up in 1-2 weeks The best next step in management is the application of a thumb spica splint and follow-up in 1-2 weeks. The clinical presentation is suggestive of a scaphoid fracture, which may not be visible on initial X-rays. Immobilization with a thumb spica splint helps to stabilize the wrist and prevent complications. Follow-up imaging or clinical re-evaluation can help confirm the diagnosis if symptoms persist. Smarty PANCE Content Blueprint Review: Covered under ⇒ PANCE Blueprint Musculoskeletal -> Upper extremity disorders -> Fractures and dislocations of the forearm, wrist and hand 4. A 38-year-old man comes to the office due to left knee pain for the past 4 days. He twisted his knee while playing soccer and felt a sharp pain immediately but was able to continue playing. The next morning, he noticed swelling in the knee and significant pain on the medial side of the knee, especially with stairs and squatting. He is able to fully extend the knee but experiences a catching sensation when walking, and the pain is interfering with his daily activities. Which of the following is most likely to be found on examination of this patient’s knee? A. Positive Lachman test B. Positive McMurray test C. Positive patellar apprehension test D. Positive anterior drawer test E. Positive varus stress test Answer and topic summary The answer is B. Positive McMurray test The most likely finding on examination of this patient’s knee is a positive McMurray test. This test is used to detect meniscal tears, which are suggested by the patient’s history of twisting injury, medial knee pain, swelling, and the catching sensation. A positive McMurray test indicates a meniscal tear when pain or a click is felt during the maneuver. Smarty PANCE Content Blueprint Review: Covered under ⇒ PANCE Blueprint Musculoskeletal ⇒ Disorders of the Knee ⇒ Soft tissue injuries of the knee 5. A 16-year-old girl is brought to the emergency department after a syncopal episode during a basketball game. Fifteen minutes prior to fainting, she experienced palpitations and shortness of breath. She was walking towards the bench when she suddenly “passed out” and regained consciousness a few seconds later. She has no prior history of fainting or head injury. She feels better after receiving intravenous normal saline. Vital signs are normal. Physical examination reveals a grade 3/6 systolic ejection murmur at the left upper sternal border. Which of the following is the most likely cause of this patient’s syncope? A. Vasovagal syncope B. Hypertrophic cardiomyopathy C. Long QT syndrome D. Aortic stenosis E. Wolff-Parkinson-White syndrome Answer and topic summary The answer is B. Hypertrophic cardiomyopathy Hypertrophic cardiomyopathy (HCM) is the most likely cause of this patient’s syncope. HCM is characterized by asymmetric left ventricular hypertrophy, which can obstruct blood flow during exertion. The symptoms of palpitations, shortness of breath, and the presence of a systolic ejection murmur at the left upper sternal border are suggestive of HCM. This condition is a common cause of sudden cardiac death in young athletes. Smarty PANCE Content Blueprint Review: Covered under ⇒ PANCE Blueprint Cardiology ⇒ Cardiomyopathy ⇒ Hypertrophic Cardiomyopathy 6. A 60-year-old woman comes to the office for a routine wellness examination. She has a history of chronic obstructive pulmonary disease (COPD), hypertension, hyperlipidemia, and squamous cell carcinoma of the skin. The patient has a 35-pack-year smoking history and currently smokes one pack of cigarettes per day. She typically consumes one glass of wine each night. Family history is significant for lung cancer and stroke. The patient had a routine screening mammogram 2 years ago that was normal. During the visit, it is discussed that she is at increased risk for developing lung cancer due to her smoking history. According to the US Preventive Services Task Force, which of the following is the most appropriate lung cancer screening test for this patient? A. Chest X-ray B. Sputum cytology C. Low-dose computed tomography (LDCT) D. Positron emission tomography (PET) scan E. Magnetic resonance imaging (MRI) Answer and topic summary The answer is C. Low-dose computed tomography (LDCT) Low-dose computed tomography (LDCT) is the most appropriate lung cancer screening test for this patient. According to the US Preventive Services Task Force, annual screening with LDCT is recommended for adults aged 50 to 80 years who have a 20-pack-year or more smoking history and currently smoke or have quit within the past 15 years. LDCT is preferred over chest X-ray because it is more sensitive in detecting early-stage lung cancer. Smarty PANCE Content Blueprint Review: Covered under ⇒ PANCE Blueprint Pulmonary ⇒ Neoplasms ⇒ Lung cancer Also covered as part of the Family Medicine EOR and Emergency Medicine EOR topic lists 7. A 30-year-old man has had asthma for the past 8 years. His symptoms are frequently triggered by pollen and exercise. Over the past year, he has visited the emergency department twice for asthma exacerbations, each time requiring oral corticosteroids. He uses his albuterol inhaler about 3 times per week for symptom relief and experiences nighttime symptoms approximately once a week. According to the GINA guidelines, which of the following would be the most appropriate step in therapy to maintain remission between his asthmatic attacks? A. As-needed low-dose ICS-formoterol (LABA) B. Daily low-dose ICS-formoterol (LABA) C. Medium-dose ICS-formoterol (LABA) D. Add-on long-acting muscarinic antagonist (LAMA) E. Oral prednisone Answer and topic summary The answer is B. Daily low-dose ICS-formoterol (LABA) According to the GINA guidelines, the most appropriate step in therapy for this patient, given his symptoms and frequency of exacerbations, is daily low-dose ICS-formoterol (LABA). This recommendation corresponds to Step 3 of the GINA guidelines, which is suitable for patients who have symptoms most days, wake with asthma once a week or more, or have frequent use of a reliever inhaler. Smarty PANCE Content Blueprint Review: Covered under ⇒ PANCE Blueprint Pulmonary ⇒ Other Pulmonary Disease ⇒ Asthma 8. A 3-week-old boy is brought in for a routine health visit. He was born to a 30-year-old multigravida at 39 weeks gestation by cesarean delivery for breech presentation. Birth weight was appropriate for gestational age at 3.5 kg (7 lb 11 oz); current weight is 3.8 kg (8 lb 6 oz). Physical examination reveals a positive Ortolani maneuver with a palpable “clunk” on the left hip. An ultrasound of both hips confirms a dislocated left hip with a shallow acetabulum. Which of the following is the best next step in management of this patient? A. Immediate closed reduction and spica casting B. Pavlik harness application C. Referral for physical therapy D. Observation and follow-up in 6 months E. Surgical open reduction Answer and topic summary The answer is B. Pavlik harness application The best next step in the management of a newborn with a confirmed hip dislocation and shallow acetabulum is the application of a Pavlik harness. The Pavlik harness is the first-line treatment for developmental dysplasia of the hip (DDH) in infants younger than 6 months. It maintains the hip in flexion and abduction, allowing the acetabulum to mold around the femoral head and promote normal hip development. Smarty PANCE Content Blueprint Review: Covered under ⇒ PANCE Blueprint Musculoskeletal ⇒ Lower extremity disorders ⇒ Developmental Dysplasia of the Hip 9. A 65-year-old woman presents with a sore on her nose. Four weeks ago, she noticed a painless lesion on the tip of her nose; in the past week, the lesion became ulcerated. The patient has a history of significant sun exposure from working as a gardener. Examination shows a 5- x 6-mm scaly papule with a central ulcer located on the tip of the nose. The rest of the facial skin, including the oral mucosa and tongue, is free of lesions. There are no palpable cervical or submandibular lymph nodes. The rest of the physical examination is normal. Which of the following is the most likely diagnosis? A. Basal cell carcinoma B. Actinic keratosis C. Herpes simplex virus infection D. Squamous cell carcinoma E. Melanoma Answer and topic summary The answer is A. Basal cell carcinoma The most likely diagnosis is basal cell carcinoma (BCC), the most common type of skin cancer. It typically presents as a painless, slowly growing lesion that may ulcerate. The location on sun-exposed skin and the description of a scaly papule with a central ulcer are characteristic of BCC. BCC rarely metastasizes but can cause significant local destruction if not treated. Smarty PANCE Content Blueprint Review: Covered under ⇒ PANCE Blueprint Dermatology ⇒ Dermatologic Neoplasms ⇒ Basal cell carcinoma Also covered as part of the General Surgery EOR and Family Medicine EOR topic list 10. A 62-year-old woman is being seen in the primary care clinic for evaluation of leg pain. She has a history of hypertension, diabetes mellitus, and coronary artery disease. She reports having right calf pain associated with activity. The woman states that after walking approximately 2 or 3 blocks, she develops a cramping discomfort in her calf region that resolves with rest. She denies any change in skin color or temperature, edema, numbness, muscular weakness, chest pain, palpitations, shortness of breath, dizziness, or other associated symptoms. Her blood pressure is 145/85 mm Hg, pulse rate is 78/min, respirations are 18/min, oxygen saturation is 99% on room air, and temperature is 36.9°C (98.4°F). Her heart and lung examinations are normal. Evaluation of her lower extremity includes some dependent rubor with noted blanching on elevation. Her dorsalis pedis and posterior tibial pulses are palpable but diminished. There is mild muscular atrophy and hair loss compared to the contralateral extremity. Which of the following corresponds to the appropriate ankle-brachial index (ABI) used in the diagnosis of this patient’s disorder? A. ABI greater than 1.4 B. ABI 1.0 – 1.4 C. ABI 0.9 – 1.0 D. ABI 0.5 – 0.8 E. ABI less than 0.5 Answer and topic summary The answer is D. ABI 0.5 – 0.8 The appropriate ankle-brachial index (ABI) for the diagnosis of peripheral artery disease (PAD) in this patient is between 0.5 and 0.8. An ABI within this range indicates moderate arterial disease, which is consistent with the patient’s symptoms of claudication (pain with walking that resolves with rest) and physical findings such as diminished pulses and dependent rubor. The recommendation for patients with an ABI in this range is to refer to a vascular specialist. Smarty PANCE Content Blueprint Review: Covered under ⇒ PANCE Blueprint Cardiology ⇒ Vascular Disease ⇒ Peripheral artery disease Also covered as part of the General Surgery PAEA EOR topic list This podcast is available on every device! You can download and listen to past FREE episodes here , on iTunes , Spotify , Amazon Music , and all podcasting apps. Download Interactive Content Blueprint Checklists for the PANCE, PANRE, EOR, and PANRE/PANRE-LA Follow this link to download your FREE copy of the PANCE/PANRE/EOR Content Blueprint Checklists . Print it up and start crossing out the topics you understand, marking the ones you don’t, and making notes of key terms you should remember. The PDF version is interactive and linked directly to the individual lessons on Smarty PANCE. Smarty PANCE is not sponsored or endorsed by, or affiliated with, the National Commission on Certification of Physician Assistants.…
T
The Audio PANCE and PANRE Physician Assistant Board Review Podcast


Listen to Podcast Episode 107: This vs. That – PANCE Blueprint Comparisons You Need to Know (Episode 1) In today’s session, we will be discussing five questions related to PANCE/PANRE Blueprint topics. These questions will cover similar presentations and crucial comparisons that are important for you to know. These topics are often used by PANCE/PANRE test question writers, so it’s essential to learn how to differentiate between them. This is the first part (episode 1) of a series. If you can’t see the audio player, click here to listen to the full episode. Links from today’s episode: Sign up for our new PANCE and PANRE Test-Taking Masterclass . Check out my first blog in our “This vs. That” Blueprint series: The PANCE Blueprint Showdown: Crohn’s Disease vs. Ulcerative Colitis . Want a question of the day that covers the Blueprint – ALL of it? Sign up for the Entire Blueprint Email Series . Follow Smarty PANCE and The Daily PANCE Blueprint on Instagram and Facebook for more daily questions. Join the Smarty PANCE Member’s Community , then sign up for a study group to get updates about upcoming webinars. I hope you enjoy this free audio component of the examination portion of this site. Smarty PANCE includes over 2,000 interactive board review questions, along with flashcards, ReelDx cases , integrated Picmonics , and lessons covering every blueprint topic available to all Smarty PANCE members . You can download and listen to past FREE episodes here , on iTunes , Spotify , Google Podcasts , Stitcher , Amazon Music , and all podcasting apps. You can listen to all the latest episodes, take interactive quizzes, and download more resources on each episode page. Interactive exam to complement today’s podcast 1. A 32-year-old woman presents with a 6-month history of loose bowel movements, approximately eight per day. Blood has been present in many of them. She has lost 30 pounds. For the past 6 weeks, she has had intermittent fever. She has had no previous gastrointestinal (GI) problems, and there is no family history of GI problems. On examination, the patient looks ill. Her blood pressure is 130/ 70 mm Hg. Her pulse is 108 beats/ minute and regular. There is generalized abdominal tenderness with no rebound. A sigmoidoscopy reveals a friable rectal mucosa with multiple bleeding points. Which of the following is the most likely diagnosis? A) Crohn’s Disease B) Ulcerative Colitis C) Infectious Colitis D) Irritable Bowel Syndrome (IBS) E) Ischemic Colitis Answer and topic summary The answer is B) Ulcerative Colitis The patient’s symptoms of chronic bloody diarrhea, weight loss, fever, and the sigmoidoscopy findings of a friable rectal mucosa with multiple bleeding points are consistent with a diagnosis of ulcerative colitis (UC), which is a form of inflammatory bowel disease (IBD). UC typically involves the rectum and may extend proximally to involve other parts of the colon. Incorrect answers: A) Crohn’s Disease: This is another type of IBD. However, Crohn’s usually presents with non-bloody diarrhea, abdominal pain, and may involve any part of the GI tract from mouth to anus, often with skip lesions. In this case, the bloody diarrhea and the findings on sigmoidoscopy are more indicative of ulcerative colitis. C) Infectious Colitis: Although infectious causes can lead to similar symptoms, the duration of this patient’s symptoms (6 months) is much longer than typically seen with infectious colitis. Additionally, fever is less common in infectious colitis. D) Irritable Bowel Syndrome (IBS): IBS is a functional GI disorder characterized by abdominal pain with a change in bowel habit. It does not cause weight loss, fever, or bloody stools. E) Ischemic Colitis: This typically presents acutely in older patients or those with vascular risk factors. The clinical presentation often includes abrupt onset of abdominal pain and bloody diarrhea. The duration and pattern of symptoms in this patient are more consistent with IBD. Smarty PANCE Content Blueprint Review: Covered under ⇒ PANCE Blueprint GI and Nutrition ⇒ Colorectal disorders ⇒ Inflammatory bowel disease 2. A 27-year-old female presents to the emergency department with a 3-day history of a widespread painful rash. She reports having started a new medication for her seizures 1 week ago. On examination, you note erythematous macules that are coalescing into large areas of epidermal detachment. The mucous membranes of her mouth, eyes, and genital region are also affected, and the skin involvement covers more than 30% of her body surface area. A skin biopsy reveals full-thickness epidermal necrosis. Based on her presentation and the medication history, which of the following diagnoses is she most likely suffering from, and which medication most likely contributed to this condition? A) Erythema multiforme major secondary to Levetiracetam (Keppra) B) Toxic Epidermal Necrolysis (TEN) secondary to Carbamazepine C) Stevens-Johnson syndrome (SJS) secondary to Metformin D) Acute generalized exanthematous pustulosis secondary to Lisinopril E) Stevens-Johnson syndrome (SJS) secondary to Atorvastatin Answer and topic summary The answer is B) Toxic Epidermal Necrolysis (TEN) secondary to Carbamazepine The patient’s presentation with widespread epidermal necrosis, mucous membrane involvement, and skin detachment affecting more than 30% of her body surface area is suggestive of Toxic Epidermal Necrolysis (TEN). SJS and TEN are considered a spectrum of the same disease, with SJS affecting less than 10% of body surface area, SJS-TEN overlap affecting 10-30%, and TEN affecting more than 30%. The diagnosis can be confirmed by biopsy (showing necrotic epithelium) if clinical characteristics (eg, target lesions progressing to bullae, ocular and mucous membrane involvement, Nikolsky sign, desquamation in sheets) are inconclusive. Immediate discontinuation of the offending drug is paramount. Patients with TEN often require transfer to a burn unit or an intensive care unit for supportive care. Fluid and electrolyte balance, pain control, and prevention of secondary infections are critical. Immunomodulatory agents like IVIG (intravenous immunoglobulin) or cyclosporine may be considered, but their efficacy is still under debate. Option A: Erythema multiforme major : Presents with targetoid lesions and is less severe than SJS and TEN. Levetiracetam is not strongly associated with SJS or TEN. Option D: Acute generalized exanthematous pustulosis is an extensive formation of nonfollicular sterile pustules on erythematous background combined with fever and peripheral blood leukocytosis. This uncommon eruption is most often an allergic reaction because of drugs such as aminopenicillins and sulfonamides inter alia. A good way to remember the body surface area affected in TEN is “ T hree x T en = T hirty percent in Toxic Epidermal Necrolysis The distinction between SJS, SJS/TEN overlap, and TEN is based on the type of lesions and the amount of the body surface area with blisters and erosions Blisters and erosions cover between 3% and 10% of the body in SJS 11–30% in SJS/TEN overlap over 30% in TEN Smarty PANCE Content Blueprint Review: Covered under ⇒ PANCE Blueprint Dermatology ⇒ Desquamation ⇒ Toxic epidermal necrolysis and Stevens-Johnson syndrome 3. A 32-year-old male presents to his primary care physician complaining of persistent fatigue, unintentional weight loss over the past 3 months, and a painless swelling in the left side of his neck. On examination, there is a non-tender, rubbery 3cm lymphadenopathy in the left cervical region. A subsequent excisional biopsy of the node is performed, and microscopy reveals large atypical cells with abundant cytoplasm and bilobed nuclei, reminiscent of an “owl’s eye” appearance. What is the most likely diagnosis? A) Acute lymphoblastic leukemia (ALL) B) Non-Hodgkin’s Lymphoma C) Burkitt’s lymphoma D) Hodgkin’s Lymphoma E) Chronic lymphocytic leukemia (CLL) Answer and topic summary The answer is D) Hodgkin’s Lymphoma The patient’s clinical presentation with painless cervical lymphadenopathy and constitutional symptoms, coupled with the histological finding of large cells bearing the “owl’s eye” appearance (Reed-Sternberg cells) , is characteristic of Hodgkin’s Lymphoma. Hodkin’s lymphoma is the most common type of lymphoma and usually presents as a solitary cervical lymph node that has been there for > 30 days. It is commonly manifested with painless cervical adenopathy; there may be splenomegaly or enlargement of other immune tissue, fever, weight loss, fatigue, or night sweats. Upper body lymph nodes are the most common. Diagnosis is based on clinical presentation and confirmed by an excisional biopsy of an involved lymph node. Histology often reveals the pathognomonic Reed-Sternberg cells , which have bilobed nuclei. A chest radiograph should be obtained to search for mediastinal adenopathy. Treatment for Hodgkin’s Lymphoma depends on the stage of the disease and typically involves a combination of chemotherapy , most commonly the ABVD regimen (Adriamycin, Bleomycin, Vinblastine, Dacarbazine). Radiation therapy might be added, especially for localized disease. Answer Choices Explanations: A) Acute lymphoblastic leukemia (ALL) : Affects mainly children and presents with bone marrow failure symptoms. B) Non-Hodgkin’s Lymphoma: A group of lymphoid malignancies. Reed-Sternberg cells distinguish Hodgkin’s Lymphoma from Non-Hodgkin’s types. C) Burkitt’s lymphoma: Fast-growing non-Hodgkin’s lymphoma linked to the Epstein-Barr virus. Exhibits a “starry sky” appearance on microscopy. E) Chronic lymphocytic leukemia (CLL) : A malignancy of mature B cells in older adults. Increased lymphocyte count and smudge cells on a peripheral blood smear. Smarty PANCE Content Blueprint Review: Covered under ⇒ PANCE Blueprint Hematology ⇒ Neoplasms, premalignancies, and malignancies ⇒ Lymphoma (ReelDx + Lecture) 4. A 65-year-old male presents to your office complaining of fatigue and shortness of breath with exertion. The patient reports minimal cough. On physical exam, you note a thin, barrel-chested man with decreased heart and breath sounds, pursed-lip breathing, end-expiratory wheezing, and scattered rhonchi. Chest X-ray reveals a flattened diaphragm, hyperinflation, and a small, thin-appearing heart. PFTs show a decreased FEV1 / FVC ratio. A) Asthma B) Chronic bronchitis C) Emphysema D) Pulmonary fibrosis E) Bronchiectasis Answer and topic summary The answer is C. Emphysema The patient’s presentation of shortness of breath with exertion , barrel-chested appearance , pursed-lip breathing , findings on chest X-ray (e.g., flattened diaphragm , hyperinflation, and a small heart), and pulmonary function test results all align with the diagnosis of emphysema . Emphysema is a lung disease that causes shortness of breath. It is one of the two main conditions that make up chronic obstructive pulmonary disease (COPD). The other condition is chronic bronchitis . In emphysema, the air sacs (alveoli) in the lungs are damaged . Over time, the inner walls of the air sacs weaken and rupture — creating larger air spaces instead of many small ones. This reduces the surface area of the lungs and, in turn, the amount of oxygen that reaches your bloodstream. The main cause of emphysema is cigarette smoking. Other causes include air pollution and chemical fumes. A small percentage of cases are caused by a familial or genetic disorder, alpha-1-antitrypsin deficiency. The body’s natural response to ↓ lung function is chronic hyperventilation = Pink Puffers! CO2 Retainers – the body must increase ventilation to blow off CO2 Minimal cough (compared to chronic bronchitis), quiet lungs Minimal sputum (compared to chronic bronchitis) Thin, underweight, and barrel chest Answer Choices Explanations: A) Asthma : An obstructive lung disease that is reversible and is marked by bronchoconstriction. It typically presents with episodes of wheezing triggered by factors such as allergens, exercise, or infections. B) Chronic bronchitis : Defined by a productive cough for 3 consecutive months in 2 successive years. It’s a subtype of chronic obstructive pulmonary disease (COPD), with its main causative factor being smoking. The primary concern is mucus production, as opposed to the alveolar wall destruction seen in emphysema. D) Pulmonary fibrosis : Represents a set of disorders causing scarring of the lung tissue, leading to a restrictive lung disease pattern. Symptoms might include a dry cough, finger clubbing, and inspiratory crackles upon examination. E) Bronchiectasis : It is marked by the chronic dilation of bronchi or bronchioles due to repeated infections and inflammation. Patients often have a chronic cough and produce significant amounts of sputum. Smarty PANCE Content Blueprint Review: Covered under ⇒ PANCE Blueprint Pulmonary ⇒ Chronic obstructive pulmonary diseases ⇒ Emphysema Also covered as part of the Internal Medicine EOR , Family Medicine EOR , and General Surgery EOR topic lists 5. A 68-year-old woman presents to the emergency department with chest pain that started 5 hours ago. She describes the pain as a heavy pressure radiating to her left arm. It began at rest and has persisted. She has a history of hypertension and diabetes mellitus. An ECG shows ST-segment depressions in leads II, III, and aVF. Troponin I levels are elevated at 0.55 ng/mL (normal: <0.04 ng/mL). Which of the following is the most likely diagnosis? A) Stable angina B) Unstable angina C) Non-ST segment elevation myocardial infarction (NSTEMI) D) ST-segment elevation myocardial infarction (STEMI) E) Prinzmetal angina Answer and topic summary The answer is C) Non-ST segment elevation myocardial infarction (NSTEMI) This patient’s presentation with chest pain at rest, ST-segment depressions on ECG, and elevated troponin I levels are consistent with NSTEMI . NSTEMI is characterized by myocardial ischemia severe enough to result in myocyte injury and elevated cardiac biomarkers but not sufficient to cause ST-segment elevation on ECG. NSTEMI is diagnosed clinically with supporting findings from ECG changes , especially ST-segment depressions or T-wave inversions, and elevated cardiac biomarkers (troponins or CK-MB). Elevated troponins, in particular, differentiate NSTEMI from unstable angina. Immediate management includes antiplatelet agents (e.g., aspirin, clopidogrel), anticoagulation (e.g., heparin or enoxaparin), and nitrates for symptom relief. A coronary angiography, followed by possible revascularization (percutaneous coronary intervention or coronary artery bypass grafting), might be required based on risk assessment. Answer Choices Explanations: A) Stable angina: Characterized by predictable chest pain or discomfort with exertion or stress, which is relieved by rest or nitroglycerin. B) Unstable angina: Presents as chest pain at rest, new-onset angina, or angina that is more frequent, longer in duration, or not relieved by rest/nitroglycerin. Crucially, cardiac biomarkers (e.g., troponins) remain normal, differentiating it from NSTEMI. D) ST-segment elevation myocardial infarction (STEMI): Acute myocardial infarction characterized by ST-segment elevation on ECG. Requires immediate reperfusion therapy. E) Prinzmetal angina: Caused by coronary artery spasm leading to transient ST-segment elevation. Pain typically occurs at rest, often in the early morning hours. Smarty PANCE Content Blueprint Review: PANCE Blueprint Cardiology => Coronary Heart Disease (PEARLS) => Non-ST-Segment Elevation MI (NSTEMI) ReelDx This podcast is available on every device! You can download and listen to past FREE episodes here , on iTunes , Spotify , Google Podcasts , Stitcher , Amazon Music , and all podcasting apps. Download Interactive Content Blueprint Checklists for the PANCE, PANRE, EOR, and PANRE-LA Follow this link to download your FREE copy of the PANCE/PANRE/EOR Content Blueprint Checklists . Print it up and start crossing out the topics you understand, marking the ones you don’t, and making notes of key terms you should remember. The PDF version is interactive and linked directly to the individual lessons on Smarty PANCE. Smarty PANCE is not sponsored or endorsed by, or affiliated with, the National Commission on Certification of Physician Assistants.…
T
The Audio PANCE and PANRE Physician Assistant Board Review Podcast


1 Podcast Episode 106: Ten PANCE, PANRE, and Rotation Questions + Review of Adrenal Insufficiency 28:55
Listen to Podcast Episode 106: Ten PANCE, PANRE, and Rotation Review Questions + Review of Adrenal Insufficiency If you can’t see the audio player, click here to listen to the full episode. Welcome to episode 106 of the Audio PANCE and PANRE Physician Assistant/Associate Board Review Podcast. Join me today as we cover ten board review questions for your PANCE, PANRE, EOR, and EOC exams. Links from today’s episode: Sign up for our new PANCE and PANRE Test Taking Masterclass Sign up for the Entire Blueprint Email Series Follow Smarty PANCE and The Daily PANCE Blueprint on Instagram and Facebook Review adrenal insufficiency and adrenal disorders Join the Smarty PANCE Member’s Community , then sign up for a study group to get updates about upcoming webinars. I hope you enjoy this free audio component of the examination portion of this site. Smarty PANCE includes over 2,000 interactive board review questions, along with flashcards, ReelDx cases , integrated Picmonics , and lessons covering every blueprint topic available to all Smarty PANCE members . You can download and listen to past FREE episodes here , on iTunes , Spotify , Google Podcasts , Stitcher , Amazon Music , and all podcasting apps You can listen to all the latest episodes, take interactive quizzes, and download more resources on each episode page. Interactive exam to complement today’s podcast 1. A 22-year-old female with an unknown past medical history presents to the ER with a prolonged seizure lasting more than 5 minutes per EMS. Her airway was supported, and IV access was obtained en route to the ER. Her blood glucose is 120. Her vitals are stable, and laboratory studies are relatively unremarkable. Which of the following would be an appropriate medication to give to this patient? A. Calcium gluconate B. Propofol infusion C. Lorazepam D. Clonidine E. Narcan Answer and topic summary The answer is C. Lorazepam The patient has status epilepticus , which is defined as >5 minutes of continuous seizures or > 2 discrete seizures between which there is incomplete recovery of consciousness. Most episodes of status epilepticus in adults are due to a brain lesion or a toxic/metabolic disturbance (e.g., alcohol withdrawal, hypoglycemia, etc). Immediate management of status epilepticus includes stabilizing the airway, placing pulse oximetry and cardiorespiratory monitors, establishing IV access, and getting a glucose level. The first-line pharmacological agent given is a benzodiazepine (e.g., lorazepam or diazepam). Keppra, valproate, or other anti-seizure medication can be given as well. Smarty PANCE Content Blueprint Review: Covered under ⇒ PANCE Blueprint Neurology ⇒ Seizure disorders ⇒ Status epilepticus Also covered as part of the Internal Medicine EOR and Emergency Medicine EOR topic list 2. A 41-year-old male presents to the ER after a construction accident that left him with severe right eye pain and decreased visual acuity. On a physical exam, you notice a teardrop-shaped pupil. Which of the following is the most likely diagnosis? A. Globe rupture B. Acute angle-closure glaucoma C. Retinal detachment D. Corneal laceration E. Metallic foreign body Answer and topic summary The answer is A. Globe rupture Mechanical globe injuries occur when there is a laceration or full-thickness rupture through the cornea and/or sclera. Globe rupture (also called an open globe) follows blunt eye injury (e.g., motor vehicle crash, assault, thrown ball, etc). Globe lacerations occur after trauma from a sharp-penetrating object (e.g., knife or high-velocity projectile). PE signs include subconjunctival hemorrhage, irregularly-shaped pupil, hyphema (anterior chamber bleeding), decreased visual acuity, and limited EOM. This is an emergency and a patient should see an ophthalmologist immediately . Smarty PANCE Content Blueprint Review: Covered under ⇒ PANCE Blueprint EENT ⇒ Traumatic disorders ⇒ Globe rupture 3. A 45-year-old male veteran presents to your clinic complaining of the following for 4 months: problems with concentration, difficulty staying asleep, persistent negative state, inability to remember events that happened while he was in Iraq, feelings of detachment from others, avoidance of other veterans, and distress when seeing violence on the TV. He denies hallucinations. He has been a veteran for 5 years. Which of the following is the most likely diagnosis? A. Acute stress disorder B. Cyclothymic disorder C. Brief psychotic episode D. Derealization/depersonalization disorder E. Post-traumatic stress disorder Answer and topic summary The answer is E. Post-traumatic stress disorder Post-traumatic stress disorder is a complex disorder defined by the behavioral, somatic, cognitive, and emotional effects of trauma. The most common causes are sexual relationship violence (33%), death or traumatic event of a loved one (30%), interpersonal violence like physical assault (12%), and participation in organized violence like combat (11%). The DSM-V criteria for PTSD is: Exposure to a traumatic event(s). And then for ≥1 month of each of these that started/worsened after a traumatic event and are associated with the traumatic event: 1) Intrusion symptoms 2) Avoidance of stimuli 3) Alterations in cognitions and mood 4) Alterations in arousal/reactivity Treatment is SSRIs + counseling/psychotherapy. About 50% of patients will recover in 3 months with treatment. Smarty PANCE Content Blueprint Review: Covered under ⇒ PANCE Blueprint Psychiatry ⇒ Trauma and stressor-related disorders ⇒ Post-traumatic stress disorder Also covered in Emergency Medicine EOR , and Family Medicine EOR Blueprint 4. Which of the following clinical findings would you likely expect in a patient with primary adrenal insufficiency? A. Hypernatremia, hypokalemia, tired B. Moon facies, depression, bruising C. Polyuria, polydipsia, intense thirst D. Fatigue, hyponatremia, hyperpigmentation E. Hypothermia, bradycardia, goiter Answer and topic summary The answer is d. Fatigue, hyponatremia, hyperpigmentation Adrenal insufficiency is defined as insufficient production of hormones by the adrenal glands. Remember, the MOST common cause of adrenal insufficiency overall is the sudden withdrawal of exogenous steroids . The most common primary cause of adrenal insufficiency is autoimmune adrenalitis (aka Addison disease, aka 21-hydroxylase antibodies against the adrenal gland). Patients will have symptoms and signs of glucocorticoid & mineralocorticoid deficiency, such as fatigue (MC), weight loss , nausea , vomiting , hyponatremia , hyperkalemia, skin hyperpigmentation (due to increased production of proopiomelanocortin prohormone), hypotension , and salt craving . Remember, secondary adrenal insufficiency does NOT lead to hyperpigmentation, only primary adrenal insufficiency (this is commonly asked in tests). Diagnosis of adrenal insufficiency is made by serum cortisol level and then a cosyntropin stimulation test . Treatment is glucocorticoids +/- mineralocorticoids. Smarty PANCE Content Blueprint Review: Covered under ⇒ PANCE Blueprint Endocrinology ⇒ Adrenal Disorders ⇒ Primary adrenal insufficiency Also covered as part of the Family Medicine EOR , Internal Medicine EOR , and Emergency Medicine EOR topic list 5. A 42-year-old male presents with right upper extremity numbness, dysesthesia, weakness, and arm pain with exertion. The symptoms are aggravated by activity and reaching overhead. Which of the following is the most likely diagnosis? A. Thoracic outlet syndrome B. Median nerve compression C. Biceps tendon rupture D. Carpal tunnel syndrome E. Subclavian steal syndrome Answer and topic summary The answer is A. Thoracic outlet syndrome Thoracic outlet syndrome (TOS) refers to a constellation of signs & symptoms that occur due to compression of the neurovascular bundle in the space of the thoracic outlet. Causes include trauma, repetitive injuries, anatomical defects, and pregnancy. The clinical presentation depends on what is being compressed (i.e., nerve, vein, and/or artery), but may include the following: pain, numbness, weakness, muscle weakness, fatigue, swelling, or coldness. Often these symptoms are aggravated by activity or raising the arm overhead. Diagnostic tests include ultrasound, x-ray, CT scan, EMG, MRI, etc. Treatment options depend on what type of TOS the patient has (e.g., physical therapy for neurogenic TOS). Smarty PANCE Content Blueprint Review: Covered under ⇒ PANCE Blueprint Musculoskeletal ⇒ Spinal Disorders ⇒ Thoracic outlet syndrome 6. A 34-year-old female is diagnosed with immune thrombocytopenic purpura. What is NOT a known cause or risk factor of ITP? A. Hepatitis C B. Systemic lupus erythematosus C. HIV D. COVID-19 E. Warfarin Answer and topic summary The answer is E. Warfarin Immune thrombocytopenic purpura (ITP) is acquired thrombocytopenia caused by autoantibodies against platelet antigens. Primary ITP is specifically due to autoimmune mechanisms, whereas secondary ITP is associated with other conditions or drugs. Some examples of causes of secondary ITP include systemic lupus erythematosus ( SLE ), HIV infection, hepatitis C infection, COVID-19 , and thyroid dysfunction. There are also many drugs that can lead to ITP, but warfarin is not known to be one of them. Clinical symptoms include petechiae, purpura, easy bleeding, bruising . Obviously, platelets will be low. Work-up for ITP should include HIV and HCV testing, TSH, ANA, coagulation studies, and peripheral blood smear. A bone marrow biopsy can be done to rule out something more pathological. Treatment includes steroids (high-dose) and IVIG . Definitive treatment is splenectomy . Smarty PANCE Content Blueprint Review: Covered under ⇒ PANCE Blueprint Hematology ⇒ Coagulation Disorders ⇒ Thrombocytopenia ⇒ Idiopathic thrombocytopenic purpura Also covered as part of the Internal Medicine EOR topic list 7. A 72-year-old African American male presents to the clinic complaining of weight loss, fatigue, and back pain. During the physical exam, you palpate an enlarged, irregular, nodular prostate. Which of the following is the most important risk factor for the development of this most likely diagnosis? A. Genetic factors B. Ethnicity C. Increasing age D. Obesity E. High meat diet Answer and topic summary The answer is C. Increasing age Prostate cancer is the most common cancer among men (excluding skin cancer) — as about 1 in 8 men will get it. The most important risk factor for the development of prostate cancer is increasing age. Other risk factors include ethnicity, genetic factors, obesity, tobacco, and a diet high in animal fat. Clinical manifestations may include non-specific urinary symptoms (frequency, urgency, etc), hematuria , bone pain , elevated PSA, and an indurated/asymmetrical prostate with nodules on digital rectal exam. Diagnosis is made by a transrectal biopsy of the prostate. Smarty PANCE Content Blueprint Review: Covered under ⇒ PANCE Blueprint Genitourinary ⇒ Neoplasms of the Genitourinary System ⇒ Prostate cancer Also found in Internal Medicine EOR PAEA blueprint 8. A 52-year-old female with a history of previous abdominal surgeries presents to the ER with nausea, emesis, abdominal pain, and inability to pass gas. On physical exam, you notice abdominal distention and auscultate high-pitched tinkering sounds in the upper abdominal quadrants. Which of the following is the most likely diagnosis? A. Mesenteric ischemia B. Diverticulitis C. Gastric ulcer perforation D. Small bowel obstruction E. Ischemic colitis Answer and topic summary The answer is D. Small bowel obstruction A bowel obstruction happens when the normal flow of the GI tract is interrupted. The most common cause of small bowel obstruction is intraperitoneal adhesions (s/t previous surgeries). Other causes include hernias or neoplasms. Clinical features include nausea, vomiting, crampy abdominal pain, and obstipation (inability to pass stool). On physical exam you may note abdominal distention , tinkling high-pitch sounds on auscultation , and hyperresonance to percussion . Initial diagnostic modalities include an abdominal x-ray and CT with contrast. These patients need to be admitted for further evaluation and management. Surgery should be consulted. Smarty PANCE Content Blueprint Review: Covered under ⇒ PANCE Blueprint GI and Nutrition ⇒ Diseases of the Small Intestine ⇒ Small bowel obstruction Also covered as part of the Family Medicine EOR , Emergency Medicine EOR , and General Surgery EOR topic list 9. Which of the following is the most common cause of acute cor pulmonale? A. Cardiac tamponade B. Pulmonary embolism C. Mitral regurgitation D. Congestive heart failure E. Portopulmonary syndrome Answer and topic summary The answer is B. Pulmonary embolism The most common cause of acute cor pulmonale is a massive pulmonary embolism (PE). Cor pulmonale is defined by RV failure/enlargement due to a pulmonary issue. The reason a PE leads to acute right heart failure is because a PE is a mechanical obstruction that increases vascular resistance in lungs and increases RV afterload. The RV can’t unload sufficiently and this results in dilation of the RV. The dilated RV impedes on the LV, leading to decreased LV output and decreased supply to the coronary arteries. It also causes RV wall tension/pressure to build and thus coronary perfusion is impeded to the right heart, further leading to ischemia. Also, the dilation of the RV can lead to tricuspid regurgitation, which further reduces blood flow. As the RV becomes ischemic, it can’t contract as well and this further decreases RV output and LV output — leading to a dangerous cycle toward shock. You can risk stratify the degree of RV compromise by getting an echocardiogram , BNP , & troponin . Smarty PANCE Content Blueprint Review: Covered under ⇒ PANCE Blueprint Pulmonary ⇒ Pulmonary Circulation ⇒ Cor pulmonale 10. A 25-year-old female with no past medical history presents with nausea, emesis, and pelvic pain. She is normotensive, afebrile, and slightly tachycardic (HR 102 bpm). On physical exam, you palpate an ovarian/pelvic mass. Beta-hCG is negative. The pelvic US is pending. Which of the following is the most likely diagnosis? A. Ectopic pregnancy B. Appendicitis C. Pelvic inflammatory disorder D. Ovarian torsion E. Placenta abruption Answer and topic summary D. Ovarian torsion Ovarian torsion refers to the rotation of the ovary, often leading to complete or partial obstruction of blood supply. The most common predisposing factor is an ovarian mass. Presenting features include pelvic pain (90%), nausea, emesis, and fever. On physical exam they may have pelvic and/or abdominal tenderness. A pelvic ultrasound is the mainstay of evaluation when ovarian torsion is suspected. You can get a beta-hCG to rule out an ectopic pregnancy. The definitive diagnosis is made by directly visualizing a rotated ovary at the time of surgical evaluation. Surgery and OB-GYN should be consulted for further management. Smarty PANCE Content Blueprint Review: Covered under ⇒ PANCE Reproductive System Blueprint ⇒ Ovarian Torsion Also covered as part of the Women’s Health PAEA EOR topic lists This podcast is available on every device! You can download and listen to past FREE episodes here , on iTunes , Spotify , Google Podcasts , Stitcher , Amazon Music , and all podcasting apps. Download Interactive Content Blueprint Checklists for the PANCE, PANRE, EOR, and PANRE-LA Follow this link to download your FREE copy of the PANCE/PANRE/EOR Content Blueprint Checklists . Print it up and start crossing out the topics you understand, marking the ones you don’t, and making notes of key terms you should remember. The PDF version is interactive and linked directly to the individual lessons on Smarty PANCE. Smarty PANCE is not sponsored or endorsed by, or affiliated with, the National Commission on Certification of Physician Assistants.…
T
The Audio PANCE and PANRE Physician Assistant Board Review Podcast


Listen to Podcast Episode 105: Ten PANCE, PANRE, and Rotation Review Questions If you can’t see the audio player, click here to listen to the full episode. Welcome to episode 105 of the Audio PANCE and PANRE Physician Assistant/Associate Board Review Podcast. Join me today as we cover ten board review questions for your PANCE, PANRE, EOR, and EOC exams. Links from today’s episode: Sign up for the Entire Blueprint Email Series Follow Smarty PANCE and The Daily PANCE Blueprint on Instagram Follow Smarty PANCE and The Daily PANCE Blueprint on Facebook Review systemic lupus erythematosus (SLE) Review the diabetes diagnostic guidelines Review gestational diabetes screening guidelines Review basal cell carcinoma and our comparison tables of the Blueprint dermatologic neoplasms Join the Smarty PANCE Member’s Community , then sign up for a study group to get updates about upcoming webinars. I hope you enjoy this free audio component of the examination portion of this site. Smarty PANCE includes over 2,000 interactive board review questions, along with flashcards, ReelDx cases, integrated Picmonics, and lessons covering every blueprint topic available to all Smarty PANCE members . You can download and listen to past FREE episodes here , on iTunes , Spotify , Google Podcasts , Stitcher , Amazon Music , and all podcasting apps You can listen to all the latest episodes, take interactive quizzes, and download more resources on each episode page. Interactive exam to complement today’s podcast 1. A 68-year-old male farmer presents with a flesh-colored papule with a rolled border located on the right side of his forehead. As you examine the lesion closely, you notice something else about the lesion. Which of the following physical exam findings would make you more suspicious of malignancy? A. Telangiectasia B. Nikolsky sign C. Hypopigmentation D. Tenderness to palpation E. Central umbilication Answer and topic summary The answer is A. Telangiectasia The patient has basal cell carcinoma , which is a skin cancer with low metastatic potential. It commonly occurs on the face (70% of the time). There are different types of BCC (nodular vs. superficial vs. infiltrative). However, nodular is the most common (80%) and typical characteristics you may see include a papule with a rolled border , pearly-like look, flesh-colored , and telangiectasia . Risk factors for BCC include UV radiation, certain genes, inherited disorders, etc. Smarty PANCE Content Blueprint Review: Covered under ⇒ PANCE Blueprint Dermatology ⇒ Dermatologic Neoplasms ⇒ Basal cell carcinoma Also covered as part of the General Surgery EOR and Family Medicine EOR topic list 2. Which of the following is the most common type of elder abuse? A. Neglect B. Emotional abuse C. Physical abuse D. Sexual abuse E. Financial exploitation Answer and topic summary The answer is A. Neglect The most common type of elder abuse is neglect, which refers to the failure of a trusted person to protect an older person from harm or provide for their needs. Self-neglect is also common, which is when an older person can’t proide their own care. Some warning signs include bruising , lacerations , skin tears , spiral fractures , malnutrition , pressure ulcers , and dehydration . If you even suspect an older adult is being neglected, you should report this immediately to adult protective services (or similar agencies) and treat the medical complications immediately. Smarty PANCE Content Blueprint Review: Covered under ⇒ PANCE Blueprint Psychiatry ⇒ Abuse and Neglect ⇒ Child/elder abuse 3. Which of the following is the most common bacterial cause of a hordeolum? A. Streptococcus pyogenes B. Staphylococcus aureus C. Clostridium perfringens D. Propionibacterium species E. Enterobacter aerogenes Answer and topic summary The answer is B. Staphylococcus aureus A hordeolum (stye) is an abscess of the eyelid that is acute. It will usually present with pain and swelling of the eyelid. The most common pathogen indicated is Staphylococcus aureus . Risk factors include dirty eye makeup, rosacea, and seborrheic dermatitis. A way to differentiate hordeolums from chalazions is that H ordeolums are H ot (aka warm/tender). Treatment may include warm compresses (10 min up to 5 times daily) plus massaging the eyelid to aid in drainage. Smarty PANCE Content Blueprint Review: Covered under ⇒ PANCE Blueprint EENT ⇒ Disorders of the Eye ⇒ Lid disorders ⇒ Hordeolum Also covered as part of the Family Medicine EOR topic list 4. A 19-year-old female presents to the clinic with fatigue, weight loss, arthralgia, and an erythematous rash in a malar distribution over the cheeks and nose (that spares the nasolabial folds). Which of the following is the most common cardiac manifestation of the patient’s likely diagnosis? A. Third-degree AV block B. Mitral regurgitation C. Myocarditis D. Pericarditis E. Bradydysrhythmias Answer and topic summary The answer is D. Pericarditis Systemic lupus erythematosus (SLE) is a chronic autoimmune disease that can impact almost any organ. It’s important to be aware of how SLE clinically presents in order to get to a diagnosis and treatment. The most common clinical features overall are fatigue (80-100%), arthralgia/arthritis (>90%), fever (>50%), and Raynaud phenomenon (~50%). You may also see the classic malar rash that spares nasolabial folds. Cardiac complications are common (50%) in SLE, and they lead to morbidity and mortality. The most common cardiac manifestation is pericarditis . Other cardiac issues these patients may have include pericardial effusion, myocarditis (rare), and valvular dysfunction. Smarty PANCE Content Blueprint Review: Covered under ⇒ PANCE Blueprint Musculoskeletal ⇒ Rheumatologic Disorders ⇒ Systemic lupus erythematosus Also covered as part of the Internal Medicine EOR and Family Medicine EOR topic list 5. Which of the following is the most common and important risk factor for aortic dissection? A. Bicuspid aortic valve B. Connective tissue disorder C. Turner’s syndrome D. Systemic hypertension E. Aortic surgery Answer and topic summary The answer is D. Systemic hypertension Aortic dissection is a life-threatening emergency and is defined by a tear of the inner layer of the aorta. On an exam, you may be asked about the classifications: – Standford Classification: Stanford A is ascending aorta and Standford B is descending aorta – DeBakey Classifications : type I is B oth (ascending + descending aorta), type II is A scending aorta, and III is D escending aorta. (Hint: It spells out BAD .) The most important & common risk factor is systemic hypertension. Known causes of acute, severe increases in blood pressure include weight lifting, energy drinks, and cocaine. The classic presentation is “ tearing and ripping chest pain with pulse deficits .” Treatment is lowering the BP via IV beta-blockers for both Type A and B and then surgery (for Type A). Smarty PANCE Content Blueprint Review: Covered under ⇒ PANCE Blueprint Cardiology ⇒ Vascular Disease ⇒ Aortic aneurysm/dissection 6. Which of the following is characterized by a rash that has a “slapped cheek” appearance at first and then progresses into a maculopapular rash on the arms and trunk in a reticular pattern? A. Roseola infantum B. Rubella C. Measles D. Erythema infectiosum E. Kawasaki disease Answer and topic summary The answer is D. Erythema infectiosum Erythema infectiosum is a condition characterized by a fever with a rash. It is caused by parvovirus B19 infection. The classic presentation is a “ slapped cheek ” rash on the face and an erythematous, lacy reticular rash on the trunk and extremities . Some call it the “fifth disease.” It is a common childhood viral exanthem. Diagnosis is usually clinical (serology can be done though). The disease is self-limiting and no specific therapy is warranted (most of the time). Smarty PANCE Content Blueprint Review: Covered under ⇒ PANCE Blueprint Infectious Disease ⇒ Viral Infectious Disease ⇒ Erythema infectiosum Also covered as part of the Pediatric Rotation EOR topic list 7. Which of the following is the most common cause of generalized musculoskeletal pain in young adult females? A. Multiple sclerosis B. Rheumatoid arthritis C. Polymyalgia rheumatica D. Menstruation E. Fibromyalgia Answer and topic summary The answer is E. Fibromyalgia Fibromyalgia is the most common cause of generalized muscle/bone pain in young adult females . Patients with fibromyalgia also usually have fatigue, cognitive fog, sleep disturbances, morning stiffness, headaches, paresthesias, and depression. On a physical exam, you will see tenderness to palpation at multiple sites . Initial treatment includes patient education, aerobic exercise, management of concurrent issues (e.g., sleep and psychiatric), & tricyclic antidepressants (e.g., amitriptyline 10 mg). Smarty PANCE Content Blueprint Review: Covered under ⇒ PANCE Blueprint Musculoskeletal ⇒ Rheumatologic Disorders ⇒ Fibromyalgia Also covered as part of the Internal Medicine EOR and Family Medicine EOR topic list 8. Which of the following is the best first-line treatment for a post-menopausal woman with osteoporosis and no pathological fractures? A. Bisphosphonates B. Parathyroid hormone analogs C. RANKL inhibitors D. Selective estrogen receptor modulators E.Calcitonin Answer and topic summary The answer is A. Bisphosphonates Osteoporosis is characterized by low bone mass and skeletal fragility leading to decreased bone strength . There are no clinical symptoms of osteoporosis until a patient has an actual fracture. Vertebral fractures in particular, are the MOST common clinical manifestation of osteoporosis. Diagnosis is made by a DEXA scan (T-score < -2.5 standard deviations) or a fragility fracture . Lifestyle changes that can be made include exercise and cessation of smoking. The first-line pharmacological agent is oral bisphosphonates (e.g., alendronate or risedronate). Remember bisphosphonates should be administered first thing in the AM and >30 min before the first food. Patients should stay upright for >30 minutes. Smarty PANCE Content Blueprint Review: Covered under ⇒ PANCE Blueprint Musculoskeletal ⇒ Rheumatologic Disorders ⇒ Osteoporosis Also covered as part of the Family Medicine EOR topic list 9. A 72-year-old female with sick sinus syndrome (s/p pacemaker) presents to your clinic with fever and chills. On a physical exam, you appreciate a new murmur located at the left 5th intercostal space at the midclavicular line. You also notice petechiae on the patient’s conjunctivae. Which of the following is the most likely diagnosis? A. Bacterial pneumonia B. Pericarditis C. Rheumatic fever D. Infective endocarditis E. Pleural effusion Answer and topic summary The answer is D. Infective endocarditis The patient has infective endocarditis (IE), which is an infection of the endocardial surface of the heart. Risk factors include pre-existing valvular or congenital heart disease , IVDU , indwelling cardiac devices , IV catheters , or recent dental/surgical procedures . The most common symptoms include fever (90%), new murmur (85%), and petechiae (20-50%). Other textbook findings include Janeway lesions (non-tender macules on palms/soles), Osler nodes (tender nodules on pads of fingers/toes), and Roth spots (hemorrhagic lesions of the retina). The best diagnostic test is a transesophageal echocardiogram. The modified Duke criteria can be used as a diagnostic guide but must be interpreted in light of pretest probability. At least 3 sets of blood cultures should be obtained. Treatment includes IV antibiotics and removal of any infected device. Smarty PANCE Content Blueprint Review: Covered under ⇒ PANCE Blueprint Cardiology ⇒ Traumatic, infectious, and inflammatory heart conditions ⇒ Acute and subacute bacterial endocarditis Also covered as part of the Emergency Medicine EOR , Internal Medicine EOR , and Family Medicine EOR topic list 10. Which of the following is not considered diagnostic for diabetes mellitus? A. Random plasma glucose > 200 mg/dL with symptoms of hyperglycemia B. A1c > 6.5 C. Fasting plasma glucose > 126 mg/dL on at least two separate occasions D. Plasma glucose > 200 mg/dL measured after a glucose tolerance test E. All of the above are diagnostic Answer and topic summary The answer is E. All of the above are diagnostic Type 2 diabetes mellitus is a very prevalent, serious disease that impacts about 9% of adults in the United States. Risk factors include family history, obesity, sedentary lifestyle, smoking, and poor diet habits. Initial symptoms include polyuria , polydipsia , polyphagia , and blurred vision . Diagnosis is made by one of the following answer choices seen on the first slide. First-line therapies include metformin and lifestyle changes (i.e., weight loss + exercise). Early combination therapy can be considered at treatment initiation to extend the time to treatment failure. It is VERY important to reach tight glycemic control as soon as possible in diabetic patients. Complications of diabetes include microvascular (retinopathy, nephropathy, neuropathy) and macrovascular (stroke, myocardial infarction). Smarty PANCE Content Blueprint Review: Covered under ⇒ PANCE Blueprint Endocrinology ⇒ Diabetes Mellitus ⇒ Diabetes Mellitus Type 2 Also covered as part of the Family Medicine EOR and Emergency Medicine EOR topic list This podcast is available on every device! You can download and listen to past FREE episodes here , on iTunes , Spotify , Google Podcasts , Stitcher , Amazon Music , and all podcasting apps. Download Interactive Content Blueprint Checklists for the PANCE, PANRE, EOR, and PANRE-LA Follow this link to download your FREE copy of the PANCE/PANRE/EOR Content Blueprint Checklists . Print it up and start crossing out the topics you understand, marking the ones you don’t, and making notes of key terms you should remember. The PDF version is interactive and linked directly to the individual lessons on Smarty PANCE. Smarty PANCE is not sponsored or endorsed by, or affiliated with, the National Commission on Certification of Physician Assistants.…
T
The Audio PANCE and PANRE Physician Assistant Board Review Podcast


Listen to Podcast Episode 104: Ten PANCE, PANRE, and Rotation Review Questions If you can’t see the audio player, click here to listen to the full episode. Welcome to episode 104 of the Audio PANCE and PANRE Physician Assistant/Associate Board Review Podcast. Join me today as we cover ten board review questions for your PANCE, PANRE, EOR, and EOC exams. Links from today’s episode: Sign up for the Entire Blueprint Email Series Follow Smarty PANCE and The Daily PANCE Blueprint on Instagram Follow Smarty PANCE and The Daily PANCE Blueprint on Facebook Join the Smarty PANCE Member’s Community , then sign up for a study group to get updates about upcoming webinars. I hope you enjoy this free audio component of the examination portion of this site. Smarty PANCE includes over 2,000 interactive board review questions, along with flashcards, ReelDx cases, integrated Picmonics, and lessons covering every blueprint topic available to all Smarty PANCE members . You can download and listen to past FREE episodes here , on iTunes , Spotify , Google Podcasts , Stitcher , Amazon Music , and all podcasting apps You can listen to all the latest episodes, take interactive quizzes, and download more resources on each episode page. Smarty PANCE is not sponsored or endorsed by, or affiliated with, the National Commission on Certification of Physician Assistants. Interactive exam to complement today’s podcast When is screening for gestational diabetes done? A. 16 weeks B. 22 weeks C. 24 weeks D. 32 weeks E. 34 weeks Answer and topic summary The answer is C. 24 weeks Prenatal care is extremely important. Screening for gestational diabetes is routinely done in pregnant patients at 24 weeks of gestation (typically until 28 weeks). Pregnancy is associated with insulin resistance, mostly because of the placenta’s secretion of human placental lactogen . There are bad consequences of gestational diabetes, so it is critical it is diagnosed and treated adequately. The initial test is a one-hour 50-gram oral glucose tolerance test (GTT) . A positive test >135 mg/dL. If a patient tests positive, they need to undergo the second test, which is a three-hour 100 mg oral GTT. The cut-offs are debated, but generally, the following are positive results: fasting > 95 mg/dL, 1 hour>180 mg/dL, 2 hours>155 mg/dL, 3 hours>140 mg/dL. Smarty PANCE Content Blueprint Review: Covered under ⇒ PANCE Blueprint Reproductive System ⇒ Complicated Pregnancy ⇒ Gestational diabetes Also covered as part of the Women’s Health EOR topic list 2. A 60-year-old male with a history of alcohol abuse and esophageal varices is brought to the ER with lethargy, delirium, weakness, and nausea. He is normotensive and afebrile. On physical exam, he is ill-appearing with jaundice, spider angiomas, a distended abdomen, and 3+ pretibial pitting edema. Based on his history and clinical presentation, which of the following electrolyte abnormalities would you expect to see in this patient? A. Hyponatremia B. Hypocalcemia C. Hypercalcemia D. Hyperphosphatemia E. Hypermagnesemia Answer and topic summary The answer is A. Hyponatremia The patient has hypervolemic hyponatremia secondary to cirrhosis. The causes of hypervolemic hyponatremia are cirrhosis, nephrotic syndrome, and CHF. Symptoms include nausea, headache, lethargy, and seizures. It’s important to have an approach to hyponatremia since it is the most common electrolyte abnormality in the hospital. First, it’s important to rule out pseudohyponatremia due to proteins, glucose, or mannitol. Also, make sure it’s not a diuretic causing hyponatremia. Next, consider the volume status – are they hypervolemic, hypovolemic, or euvolemic? Hypovolemic causes are more obvious (emesis, hemorrhage, etc.); however, urinary sodium can help differentiate between hypovolemia and euvolemia. If uNA < 20 , then this means the renin-angiotensin-aldosterone system is on and trying to maintain pressure/volume; therefore, it is likely the patient is hypovolemic . Treatment of hyponatremia depends on the cause. It usually involves fluid restriction and possibly (and carefully) a hypertonic solution. Remember – rapid correction of hyponatremia can lead to central pontine myelinolysis. Smarty PANCE Content Blueprint Review: Covered under ⇒ PANCE Blueprint Renal System ⇒ Fluid and Electrolyte Disorders ⇒ Hyponatremia Also covered as part of the Internal Medicine EOR topic list 3. A 12-year-old male patient presents with a circular, expanding rash located where a tick had bitten him. He had recently been camping in the upper Midwest. He also complains of a headache and mild fever. Which of the following is a known cardiac complication of the patient’s most likely diagnosis? A. Supraventricular tachycardia B. Atrioventricular block C. Wandering atrial pacemaker D. Sinus bradycardia E. None of the above Answer and topic summary The answer is B. Atrioventricular block The patient most likely has Lyme disease , which is a tick-borne illness usually caused by Borrelia burgdorferi . It is more prevalent in the Northeast and upper Midwest. Erythema migrans , which is a circular, expanding rash that may look like a “bull’s eye” is seen about 20% of the time with Lyme disease. Clinical features in the early localized disease include fever, headaches, malaise, arthralgia, and lymphadenopathy. Treatment is doxycycline for non-pregnant adults and children. Amoxicillin is a second-line treatment. A well-known cardiac complication is an atrioventricular block . Smarty PANCE Content Blueprint Review: Covered under ⇒ PANCE Blueprint Infectious Disease ⇒ Spirochetal Disease ⇒ Lyme disease Also covered as part of the Internal Medicine EOR and Family Medicine EOR topic list 4. Which of the following is not a manifestation of congenital rubella syndrome? A. Deafness B. Cataracts C. Heart defects D. Microcephaly E. Fetal growth acceleration Answer and topic summary The answer is E. Fetal growth acceleration Clinical features of a congenital rubella infection (CRI) include deafness, cataracts, and cardiac disease. Patients may also have CNS abnormalities, such as microcephaly. Typically, a congenital rubella infection will lead to fetal growth restriction , not fetal growth acceleration. A classic finding that you may see on exams is petechiae and purpura — some call these “ blueberry muffin lesions .” In general, CRI should be considered in patients who have a suspected rubella infection during their pregnancy. Laboratory studies on the child (before the age of one) are done to help confirm the diagnosis. There is no treatment, and supportive care/surveillance is the main way to manage a congenital rubella infection. Smarty PANCE Content Blueprint Review: Covered under ⇒ PANCE Blueprint Infectious Disease ⇒ Viral Infectious Disease ⇒ Rubella (German Measles) Also covered as part of the Pediatric EOR topic list 5. Which of the following is the most common cause of cardiogenic shock? A. Right ventricle failure B. Tachydysrhythmias C. Myocardial infarction D. Coronary vasospasms E. Papillary muscle rupture Answer and topic summary The answer is C. Myocardial Infarction Cardiogenic shock is a type of shock that occurs due to cardiac pump failure; essentially, it is the most severe expression of heart failure. It is defined by systemic tissue hypoperfusion secondary to poor cardiac output despite adequate intravascular volume. The most common cause of cardiogenic shock is a myocardial infarction. Other causes include atrial/ventricular tachyarrhythmias, aortic or mitral valve insufficiency, septal defects, free wall rupture, etc. Therapy options vary but may include inotropic support (e.g., dobutamine), intra-aortic balloon pump, and percutaneous ventricular assist devices. Smarty PANCE Content Blueprint Review: Covered under ⇒ PANCE Blueprint Cardiology ⇒ Hypotension ⇒ Cardiogenic shock Also covered as part of the Emergency Medicine EOR topic list 6. A 20-year-old female with asthma presents to the clinic with “white stuff” in her mouth for the past few days. On physical exam, you notice a few white plaques on the buccal mucosa and tongue. The lesions are easily scraped off with a tongue depressor. Which of the following is the best initial treatment of choice for the most likely diagnosis? A. Fluconazole B. Amphotericin B C. Lidocaine solution D. Nystatin E. Bactrim Answer and topic summary The answer is D. Nystatin The patient has oropharyngeal candidiasis , which is a local infection in patients with certain risk factors (e.g., AIDs, dentures, radiation, inhaled glucocorticoids, etc.). This patient has asthma so the likely causative culprit is her inhaled glucocorticoids . The most common cause of oropharyngeal candidiasis is C. albicans . On a physical exam, you will see white plaques in the mouth (which can be scrapped). In smokers, the lesions may look more yellow/brown. Diagnosis is usually clinical but can be confirmed by scraping the lesion. A KOH test can be performed on the scrapings — budding yeast with pseudohyphae are seen. The treatment of choice is nystatin swish and swallow or topical clotrimazole. If a patient has a severe disease or fails topical therapy, oral fluconazole can be given. Smarty PANCE Content Blueprint Review: Covered under ⇒ PANCE Blueprint Dermatology ⇒ Fungal Infections ⇒ Candidiasis Also covered as part of the Internal Medicine EOR and Pediatric Rotation EOR topic list 7. A 51-year-old male presents to the clinic. His wife explains his symptoms: brief, involuntary movements of his limbs, depression, impaired thinking, and weight loss. Which of the following is the most likely diagnosis? A. Parkinson disease B. Multiple sclerosis C. Alzheimer disease D. Vascular dementia E. Huntington’s disease Answer and topic summary The answer is E. Huntington’s disease Huntington’s disease is an autosomal-dominant disorder characterized by progressive neurodegeneration. It is due to the expansion of the cytosine-adenine-guanine (CAG) trinucleotide in the HTT gene. The main clinical features include choreiform movements , dementia , cognitive impairment , and psychiatric problems . Patients typically present during mid-life. Diagnosis is confirmed by genetic testing. A brain MRI may reveal caudate atrophy. Treatment is generally supportive and symptomatic. There is no cure for Huntington’s disease. Smarty PANCE Content Blueprint Review: Covered under ⇒ PANCE Blueprint Neurology ⇒ Movement Disorders ⇒ Huntington Disease Also covered as part of the Internal Medicine EOR topic list 8. A 42-year-old male with a history of diabetes presents the ER with a “bad rash” on his left thigh. He said he had previously gotten a bug bite there. He is febrile (102.2F), tachycardic (110 bpm), and hypotensive (80/50 mmHg). On physical exam, you notice a 2-inch erythematous circular lesion that is warm to the touch. Which of the following is the best next step in the management of this patient? A. Initiate IV antibiotics and fluids B. Surgical consult for amputation C. Outpatient dermatology referral D. Order venous ultrasound of the leg E. Send home on oral antibiotics Answer and topic summary The answer is A. Initiate IV antibiotics and fluids This patient has a severe case of cellulitis and sepsis . The best next step is to initiate intravenous antibiotics and fluids. A broad-spectrum antibiotic (like cefepime) plus coverage for MRSA (vancomycin) would be appropriate. The most common cause of cellulitis is Group A beta-hemolytic Streptococcus pyogenes . What differentiates cellulitis from erysipelas is that erysipelas is more well-defined. Antibiotic options for more mild cellulitis cases include Bactrim , clindamycin , and Keflex . It’s important to keep in mind that events don’t happen in a vacuum — the bug bite led to cellulitis, and the patient’s body responded to the cutaneous infection in a complex and dysregulated way (i.e. sepsis), eventually leading to hypotension and instability. Smarty PANCE Content Blueprint Review: Covered under ⇒ PANCE Blueprint Dermatology ⇒ Dermatologic Infectious diseases ⇒ Bacterial Infections ⇒ Cellulitis Also covered as part of the Family Medicine EOR , Emergency Medicine EOR , and General Surgery EOR topic list 9. An 8-year-old male is brought into the clinic by his mother, who is concerned about his behavior in school and at home for the past 7 months. In both settings, he lacks the ability to pay attention, does not follow directions or listen, is easily distracted, is reluctant to do tasks, and constantly loses his homework and pencils. Which of the following is a common side effect of the first-line treatment for this condition? A. Cognitive fog B. Appetite suppression C. Constipation D. Somnolence E. Xerosis Answer and topic summary The answer is B. Appetite suppression The patient has attention-deficit hyperactivity disorder (ADHD). There are two main patterns: inattentive or hyperactive-impulse (some may have a combined presentation). The DSM V has criteria for diagnosing ADHD and the various subtypes, but in general, symptoms should be seen in multiple settings , last for 6 months or more , and negatively impact social/academic activities . The first-line pharmacological treatment for ADHD, in most cases, is a stimulant (e.g., methylphenidate, dextroamphetamine, etc.). The most adverse effects of stimulants include moodiness and irritability, and appetite suppression . It’s important to monitor children’s growth and weight when they are taking a stimulant for this very reason. Smarty PANCE Content Blueprint Review: Covered under ⇒ PANCE Blueprint Psychiatry ⇒ Neurodevelopmental disorders ⇒ Attention-Deficit/Hyperactivity Disorder Also covered as part of the Pediatric Rotation and Psychiatry EOR topic list 10. Which of the following patients should be screened for lung cancer, according to the USPSTF? A. 82-year-old with 34 pack-years who quit 2 years ago B. 51-year-old with 31 pack-years who quit 10 years ago C. 70-year-old with 14 pack-years who quit 5 years ago D. 40-year-old with 20 pack-years who quit 2 weeks ago Answer and topic summary The answer is B. 51-year-old with 31 pack-years who quit 10 years ago The U.S. Preventive Services Task Force (USPSTF) recently changed the lung cancer screening guidelines in March 2021. An annual low-dose chest CT scan (NOT chest x-ray) should be done for those who are 50 to 80 years old with at least a 20-pack-year history and who currently smoke or have quit within the past 15 years . It is also important to note that these guidelines do not apply to those who have another illness that already severely limits their life expectancy. Smarty PANCE Content Blueprint Review: Covered under ⇒ PANCE Blueprint Pulmonary ⇒ Pulmonary Neoplasms ⇒ Lung cancer Also covered as part of the Family Medicine EOR and Emergency Medicine EOR topic list This podcast is available on every device! You can download and listen to past FREE episodes here , on iTunes , Spotify , Google Podcasts , Stitcher , Amazon Music , and all podcasting apps. Download Interactive Content Blueprint Checklists for the PANCE, PANRE, EOR, and PANRE-LA Follow this link to download your FREE copy of the PANCE/PANRE/EOR Content Blueprint Checklists . Print it up and start crossing out the topics you understand, marking the ones you don’t, and making notes of key terms you should remember. The PDF version is interactive and linked directly to the individual lessons on Smarty PANCE.…
T
The Audio PANCE and PANRE Physician Assistant Board Review Podcast


Listen to Podcast Episode 103: Ten PANRE & PANRE-LA Intervention Complex Practice Question If you can’t see the audio player, click here to listen to the full episode. Welcome to episode 103 of the Audio PANCE and PANRE Physician Assistant/Associate Board Review Podcast. Join me today as we cover ten NCCPA-style board review questions for your PANRE and PANRE-LA exams. Special from today’s episode: Take the new PANRE & PANRE-LA (Intervention Complex) Practice Exam : Covers all the topics tested within the new PANRE (Intervention Complex) performance expectation with links to Smarty PANCE lessons. PANRE & PANRE-LA Blueprint 8-Week Schedule and Study Planner Read The New 2023 PANRE and PANRE-LA: Everything you Need to Know Members can try out the newly updated PANRE-LA Smart Search Tool (you must log in to access the search bar) Sign up for the Entire Blueprint Email Series to get daily questions for the next 478 days! Follow Smarty PANCE and The Daily PANCE Blueprint on Instagram Follow Smarty PANCE and The Daily PANCE Blueprint on Facebook Join the Smarty PANCE Member’s Community , then sign up for a study group to get updates about upcoming webinars I hope you enjoy this free audio component of the examination portion of this site. Smarty PANCE includes over 2,000 interactive board review questions, along with flashcards, ReelDx cases, integrated Picmonics, and lessons covering every blueprint topic available to all Smarty PANCE members . You can download and listen to past FREE episodes here , on iTunes , Spotify , Google Podcasts , Stitcher , and most podcasting apps. You can listen to all the latest episodes, take interactive quizzes, and download more resources on each episode page. Here is an interactive exam to complement today’s podcast 1. A 65-year-old man presents to your office with complaints of constipation for the past six months. He says that he has difficulty passing stools, which are hard and dry. He also reports occasional abdominal pain and bloating. He denies any weight loss, blood in stools, fever, or night sweats. His medical history is significant for hypertension and type 2 diabetes mellitus. His medications include metformin, lisinopril, and aspirin. He does not smoke or drink alcohol. On physical examination, his vital signs are normal. His abdomen is soft and nontender, with normal bowel sounds. There are no masses or organomegaly palpable. Which of the following is the most appropriate next step in evaluating this patient? A) Colonoscopy B) Barium enema C) Thyroid function tests D) Stool osmolarity E) Dietary modification Answer and topic summary The correct answer is A) Colonoscopy Colonoscopy is a procedure that involves inserting a flexible tube with a camera into the colon to visualize the mucosa and detect any abnormalities such as polyps, tumors, inflammation, or bleeding. It is indicated for patients with chronic constipation who are older than 50 years or have any red flag features for colorectal malignancy, such as weight loss, blood in stools, anemia, or a family history of colon cancer. This patient meets the age criterion and should undergo colonoscopy to rule out any serious causes of his constipation. Answer explanations: Barium enema is an imaging test that involves injecting a contrast agent (barium sulfate) into the rectum and taking X-rays of the colon. It can show structural abnormalities such as diverticula, strictures, masses, or volvulus. However, it is less sensitive and specific than colonoscopy for detecting colorectal malignancy. Therefore, it is not the preferred test for this patient. Thyroid function tests are blood tests that measure the levels of thyroid hormones (T3, T4) and thyroid-stimulating hormone (TSH). They can help diagnose thyroid disorders such as hypothyroidism or hyperthyroidism. Hypothyroidism can cause constipation due to decreased gastrointestinal motility. However, this patient has no other signs or symptoms of hypothyroidism such as fatigue, cold intolerance, dry skin, hair loss, or bradycardia. Therefore, thyroid function tests are not necessary for this patient. Stool osmolarity is a test that measures the concentration of solutes in stool water. It can help differentiate between osmotic diarrhea (high stool osmolarity) and secretory diarrhea (low stool osmolarity). However, this test is not useful for evaluating constipation which is defined by infrequent or difficult defecation. Dietary modification is a nonpharmacological measure that involves increasing fiber and fluid intake to improve stool consistency and frequency. It may be effective for patients with primary constipation which has no identifiable organic cause. However, this patient has chronic constipation which requires further evaluation before initiating treatment. (Review PANRE Blueprint Topic: Constipation ) 2. A 60-year-old male presents to the emergency department with chest pain and shortness of breath. ECG reveals ST segment elevation in the anterior leads. Which of the following laboratory tests is the most specific for the diagnosis of acute myocardial infarction (AMI)? A) Troponin B) Creatinine kinase-MB (CK-MB) C) Myoglobin D) C-reactive protein (CRP) E) Brain natriuretic peptide (BNP) Answer and topic summary The correct answer is A) Troponin Acute myocardial infarction (AMI) is a serious medical emergency that requires prompt diagnosis and treatment. Cardiac biomarkers such as troponin, creatinine kinase-MB (CK-MB), and myoglobin are commonly used to diagnose AMI. Among these biomarkers, troponin is the most specific for the diagnosis of AMI. Troponin is a protein found in cardiac muscle cells, and its release into the bloodstream is a specific marker of myocardial injury. Elevated troponin levels are typically seen within 3-4 hours after the onset of symptoms and can remain elevated for up to 10-14 days after an AMI. Incorrect answer explanations: Creatinine kinase (CK) is an enzyme found in various tissues, including skeletal and cardiac muscle. CK-MB is a specific isoform of CK found predominantly in cardiac muscle cells. Elevated levels of CK-MB can be seen in the early stages of AMI, but CK-MB is less specific for the diagnosis of AMI compared to troponin. Myoglobin is a protein found in skeletal and cardiac muscle cells. Elevated myoglobin levels can be seen within 1-3 hours after the onset of symptoms, but myoglobin is less specific for the diagnosis of AMI compared to troponin. Elevated myoglobin levels can also be seen in other conditions that cause muscle injury, such as rhabdomyolysis. C-reactive protein (CRP) is an acute-phase protein that is elevated in response to tissue injury, inflammation, and infection. While elevated CRP levels can be seen in patients with AMI, CRP is not specific for the diagnosis of AMI and cannot be used as a diagnostic tool on its own. Brain natriuretic peptide (BNP) is a hormone released by the heart in response to increased pressure and volume in the cardiac chambers. Elevated BNP levels can be seen in patients with heart failure and other cardiac conditions, but BNP is not specific for the diagnosis of AMI. (Review PANRE Blueprint Topic: Acute myocardial infarction: ST-segment elevation myocardial infarction (STEMI) ) 3. A 65-year-old woman with a history of atrial fibrillation, hypertension, and obesity presents to the emergency department with acute onset of severe left leg pain and swelling. She has been taking warfarin for anticoagulation but admits to missing some doses in the past week. Her blood pressure is 180/100 mmHg, pulse is 110 beats/min and irregular, and respiratory rate is 20 breaths/min. Her physical examination reveals a warm, tender, erythematous left lower extremity with prominent superficial veins and a positive Homan’s sign. Her international normalized ratio (INR) is 1.5. What is the most appropriate next step in the management of this patient? A) Start heparin infusion and adjust warfarin dose B) Order duplex ultrasonography of the lower extremities C) Administer tissue plasminogen activator (tPA) D) Perform venography of the left leg E) Apply compression stockings and elevate the leg Answer and topic summary The correct answer is B) Order duplex ultrasonography of the lower extremities Duplex ultrasonography is a non-invasive test that uses sound waves to visualize the blood flow in the veins of the legs. It can detect thrombi by measuring the diameter, compressibility, and flow characteristics of the veins. It has high sensitivity and specificity for diagnosing DVT. It is considered the first-line diagnostic test for patients with suspected DVT. If negative, it can be repeated in one week or combined with D-dimer testing to rule out DVT. Incorrect answer explanations: Heparin infusion and warfarin dose adjustment are indicated for patients with confirmed deep vein thrombosis (DVT) who have subtherapeutic INR levels. However, this patient has not yet been diagnosed with DVT and needs further diagnostic testing before initiating anticoagulation therapy. tPA is a thrombolytic agent that dissolves blood clots by activating plasminogen into plasmin. Plasmin degrades fibrin, which forms the meshwork of clots. tPA may be used for patients with massive or life-threatening DVT who have contraindications to anticoagulation or who have failed anticoagulation therapy. However, it carries a high risk of bleeding complications and should not be used without confirming DVT diagnosis first. Venography is an invasive procedure that involves injecting contrast dye into a vein and taking X-ray images to visualize any obstruction or narrowing in the venous system. It was once considered the gold standard for diagnosing DVT but has been largely replaced by duplex ultrasonography due to its higher cost, invasiveness, discomfort, and risk of allergic reactions or nephrotoxicity from contrast dye. Compression stockings are elastic garments that apply graduated pressure on the legs to improve venous return and prevent edema formation. They may be used as an adjunctive treatment for patients with confirmed DVT, along with anticoagulation therapy. Elevation of the leg can also reduce swelling and pain by decreasing hydrostatic pressure in the veins. However, these interventions do not address the underlying cause of DVT or prevent clot propagation or embolization. They should not be used as the sole therapy for suspected or confirmed DVT without diagnostic testing or anticoagulation therapy. (Review PANRE Blueprint Topic: Deep venous thrombosis ) 4. A 7-year-old child presents with honey-colored crusting lesions on the face and extremities. The lesions started as small red papules that quickly progressed to vesicles and then ruptured, leaving behind a honey-colored crust. The child has no fever and is otherwise healthy. What is the most appropriate initial treatment for this condition? A) Topical corticosteroids B) Oral antihistamines C) Oral doxycycline D) Topical mupirocin E) Oral acyclovir Answer and topic summary The correct answer is D) Topical mupirocin The presentation of honey-colored crusting lesions on the face and extremities in a child is consistent with impetigo, a bacterial skin infection caused by Staphylococcus aureus or Streptococcus pyogenes. The most appropriate initial treatment for impetigo is topical antibiotics such as mupirocin. This is because impetigo is a localized skin infection, and topical antibiotics can effectively treat it without the need for systemic antibiotics. The topical antibiotic is applied to the affected area 2-3 times per day for 5-7 days. Incorrect answer explanations: Topical corticosteroids are not appropriate for impetigo as they can exacerbate the infection by suppressing the immune response and promoting bacterial growth. Oral antihistamines are used to treat allergic reactions and are not effective in treating bacterial skin infections such as impetigo. Oral doxycycline can be used to treat impetigo, but it is not the initial treatment of choice. Oral antibiotics are reserved for cases of extensive or severe impetigo or for patients who cannot tolerate topical antibiotics. Oral acyclovir is an antiviral medication used to treat viral skin infections such as herpes simplex virus (HSV). It is not effective in treating bacterial skin infections such as impetigo. (Review PANRE Blueprint Topic: Impetigo ) 5. A 72-year-old woman with a history of hypertension, diabetes mellitus, and atrial fibrillation presents to the emergency department with sudden onset of left-sided weakness and slurred speech. She was last seen normal 2 hours ago by her daughter. Her vital signs are: blood pressure 180/100 mmHg, pulse 110 beats/min irregularly irregular, respiratory rate 18 breaths/min, temperature 36.5°C (97.7°F), and oxygen saturation 98% on room air. On physical examination, she has left facial droop, left hemiparesis (4/5 strength), and dysarthria. Her NIH Stroke Scale score is 8. A non-contrast head CT scan shows no evidence of hemorrhage. What is the most appropriate next step in management? A) Administer intravenous alteplase B) Administer intravenous heparin C) Perform carotid endarterectomy D) Perform mechanical thrombectomy E) Start oral aspirin Answer and topic summary The correct answer is A. Administer intravenous alteplase The patient has a clinical diagnosis of acute ischemic stroke, which is caused by occlusion of a cerebral artery by a thrombus or embolus. The most important factor in determining the treatment of acute ischemic stroke is the time from symptom onset to presentation. If the patient presents within 4.5 hours of symptom onset and has no contraindications, intravenous alteplase (a tissue plasminogen activator or tPA) should be administered as soon as possible to dissolve the clot and restore blood flow to the ischemic brain tissue. Intravenous alteplase has been shown to improve functional outcomes and reduce disability after acute ischemic stroke. The patient meets the criteria for intravenous alteplase because she presented within 4.5 hours of symptom onset, has no evidence of hemorrhage on head CT scan, and has no other contraindications such as recent surgery, active bleeding, severe hypertension (>185/110 mmHg), or use of anticoagulants. The other answer choices are incorrect because they are not indicated or effective in this scenario. Incorrect answer explanations: Intravenous heparin administration is not recommended for acute ischemic stroke because it does not improve outcomes and may increase the risk of bleeding complications. Carotid endarterectomy is a surgical procedure that removes plaque from the carotid artery to prevent recurrent strokes in patients with significant carotid stenosis (>70%) who are asymptomatic or have had a transient ischemic attack (TIA) or minor stroke within the past 6 months. It is not indicated for patients with acute ischemic stroke who have not been stabilized medically. Mechanical thrombectomy is an endovascular procedure that uses a catheter-based device to remove large vessel occlusions in patients with acute ischemic stroke who have a large penumbra (area of potentially salvageable brain tissue). It can be performed within 24 hours of symptom onset if intravenous alteplase is contraindicated or ineffective. However, it should not delay or replace intravenous alteplase if eligible. Oral aspirin is recommended for secondary prevention of ischemic stroke after initial treatment with intravenous alteplase or mechanical thrombectomy. It can also be used as initial treatment for patients who present more than 24 hours after symptom onset or who have contraindications to intravenous alteplase. However, it should not be given within 24 hours of receiving intravenous alteplase because it may increase the risk of bleeding complications. (Review PANRE Blueprint Topic: Stroke ) [dt_sc_button type=”type2″ link=”https://smartypance.com/lessons/vascular-disorders/stroke-reeldx266/” size=”small” target=”_blank” timeline_button=”no”]View blueprint lesson[/dt_sc_button] 6. A 65-year-old man with a history of hypertension and peptic ulcer disease presents to your clinic for a routine follow-up. He reports feeling well and has no complaints. His medications include lisinopril and omeprazole. His vital signs are normal. A complete blood count (CBC) shows: Hemoglobin: 10 g/dL (normal: 13-17 g/dL) Hematocrit: 30% (normal: 40-50%) Mean corpuscular volume (MCV): 70 fL (normal: 80-100 fL) Red cell distribution width (RDW): 18% (normal: 11-15%) White blood cell count: 6 x 10^9/L (normal: 4-11 x 10^9/L) Platelet count: 250 x 10^9/L (normal: 150-450 x 10^9/L) What is the most likely cause of this patient’s anemia? A) Chronic kidney disease B) Folate deficiency C) Gastrointestinal bleeding D) Thalassemia trait E) Vitamin B12 deficiency Answer and topic summary The answer is C. Gastrointestinal bleeding The patient has a microcytic anemia, which is characterized by a low MCV (<80 fL). The most common cause of microcytic anemia is iron deficiency. Iron deficiency can result from inadequate dietary intake, malabsorption, increased demand, or chronic blood loss. In this patient, the most likely source of chronic blood loss is gastrointestinal bleeding due to his history of peptic ulcer disease and the use of omeprazole. Omeprazole can mask the symptoms of gastrointestinal bleeding by reducing acid secretion and healing ulcers but does not prevent recurrence or complications. The patient may also have occult blood loss that is not visible in the stool. The RDW is elevated (>15%), indicating increased variation in red blood cell size due to iron deficiency. To confirm iron deficiency, iron studies such as serum ferritin, serum iron, total iron-binding capacity (TIBC), and transferrin saturation should be ordered. The patient should also undergo endoscopy to evaluate for the source and severity of gastrointestinal bleeding and rule out malignancy. The other answer choices are incorrect because they are not causes of microcytic anemia but rather causes of normocytic or macrocytic anemia. Incorrect answer explanations: Chronic kidney disease can cause normocytic anemia due to reduced production of erythropoietin by the kidneys. The MCV would be normal (80-100 fL). Folate deficiency can cause macrocytic anemia due to impaired DNA synthesis in red blood cell precursors. The MCV would be high (>100 fL). Thalassemia trait can cause microcytic anemia due to reduced synthesis of alpha or beta globin chains that form hemoglobin. However, thalassemia trait usually has a normal or low RDW (<15%) because red blood cells are uniformly small. Thalassemia trait also has a genetic basis and is more common in people of Mediterranean, African, or Southeast Asian descent. Vitamin B12 deficiency can also cause macrocytic anemia due to impaired DNA synthesis in red blood cell precursors as well as neurological symptoms such as peripheral neuropathy, ataxia, dementia, or psychosis. The MCV would be high (>100 fL). (Review PANRE Blueprint Topic: Iron Deficiency Anemia ) 7. A 42-year-old woman with a history of GERD presents to your clinic for follow-up. She has been taking omeprazole 20 mg daily for the past 6 months and reports significant improvement in her heartburn and regurgitation symptoms. She has also made lifestyle modifications such as avoiding spicy and fatty foods, quitting smoking, and elevating the head of her bed. She asks you if she can stop taking omeprazole or reduce the dose. What is the most appropriate next step in management? A) Continue omeprazole 20 mg daily indefinitely B) Discontinue omeprazole and monitor symptoms C) Switch to famotidine 20 mg twice daily D) Taper omeprazole to every other day for 4 weeks E) Perform an upper endoscopy Answer and topic summary The correct answer is D) Taper omeprazole to every other day for 4 weeks Explanation: The patient has a history of GERD that has responded well to PPI therapy and lifestyle modifications. The goal of treatment is to achieve symptom relief and prevent complications such as esophagitis, stricture, Barrett’s esophagus, or adenocarcinoma. PPIs are more effective than H2 blockers or antacids for healing erosive esophagitis and maintaining remission. However, long-term use of PPIs may be associated with adverse effects such as increased risk of fractures, infections, hypomagnesemia, vitamin B12 deficiency, and chronic kidney disease. Therefore, it is reasonable to attempt a step-down approach after achieving symptom control with PPIs for at least 8 weeks. This involves tapering the dose of PPI gradually over 2-4 weeks to avoid rebound acid hypersecretion. If symptoms recur after discontinuation of PPIs, then switching to an H2 blocker or restarting PPI maintenance therapy with the lowest effective dose may be indicated. Incorrect answer choices: Continuing omeprazole 20 mg daily may expose the patient to unnecessary risks of long-term PPI use without attempting a trial of dose reduction or discontinuation. Discontinuing omeprazole and monitoring symptoms may cause rebound acid hypersecretion and recurrence of GERD symptoms due to abrupt withdrawal of PPI therapy. Immediately switching to an H2 blocker (such as famotidine) may not provide adequate symptom control for GERD, as H2 blockers are less potent than PPIs in suppressing gastric acid secretion. Slowly taper off the PPI first over 2-4 weeks (the higher the dose, the longer the taper). If symptoms return, it would be appropriate to start again with an H2 blocker. If long-term treatment is needed, H2 blockers allow better absorption of nutrients than PPIs and so potentially have fewer long-term adverse effects. If symptoms are still difficult to control, consider adding the PPI back at the lowest effective dose. Perform an upper endoscopy: This option is not indicated for patients with uncomplicated GERD who have responded well to medical therapy. Endoscopy is reserved for patients who have alarm features such as dysphagia, odynophagia, weight loss, anemia, (Review PANRE Blueprint Topic: GERD ) 8. Which of the following is an appropriate step-down therapy for a patient with well-controlled asthma on medium-dose inhaled corticosteroids (ICS) and long-acting beta agonists (LABA)? A) Discontinue LABA and continue medium-dose ICS B) Discontinue ICS and continue LABA C) Reduce ICS dose by 50% and continue LABA D) Reduce both ICS and LABA doses by 50% E) Switch to low-dose ICS/formoterol as needed Answer and topic summary The answer is E. Switch to low-dose ICS/formoterol as needed This patient has well-controlled asthma on medium-dose ICS and LABA, which corresponds to step 4 of the asthma treatment algorithm. Step-down therapy can be considered for patients with at least 3 months of continuous good control of asthma . The goal of step-down therapy is to reduce medication use to the lowest effective dose while maintaining asthma control. According to the Global Initiative for Asthma (GINA), one option for stepping down from step 4 is to switch to low-dose ICS/formoterol as needed . This regimen involves using a combination inhaler containing low-dose ICS (budesonide) and formoterol (a fast-acting LABA) both as maintenance therapy and as reliever therapy instead of a short-acting beta agonist (SABA). This strategy has been shown to reduce exacerbations, improve symptom control, and decrease steroid exposure compared with conventional maintenance therapy with higher doses of ICS/LABA plus SABA as needed. (Review PANRE Blueprint Topic: Asthma ) 9. A 35-year-old woman presents to her primary care provider with complaints of chronic worry and nervousness for the past 8 months. She says she worries about everything, such as her health, her family, her work, and her finances. She has difficulty sleeping, concentrating, and relaxing. She also experiences muscle tension, headaches, and palpitations. She denies any history of trauma, substance abuse, or other psychiatric disorders. She has no medical problems and takes no medications. Her vital signs are normal and her physical examination is unremarkable. Which of the following is the most likely diagnosis? A) Panic disorder B) Obsessive-compulsive disorder C) Post-traumatic stress disorder D) Generalized anxiety disorder E) Adjustment disorder Answer and topic summary The correct answer is D) Generalized anxiety disorder Generalized anxiety disorder (GAD) is characterized by persistent, excessive, and unrealistic worry about everyday things that lasts for at least 6 months. The worry causes significant distress or impairment in social, occupational, or other areas of functioning. The anxiety is not attributable to any specific triggers or stressors. The diagnosis of GAD requires at least 3 of the following symptoms: restlessness, fatigue, difficulty concentrating, irritability, muscle tension, and sleep disturbance. Panic disorder is characterized by recurrent unexpected panic attacks that cause fear of having another attack or avoidance of situations that might trigger an attack. Panic attacks are sudden episodes of intense fear or discomfort that peak within minutes and are accompanied by at least 4 physical or cognitive symptoms such as palpitations, sweating, trembling, shortness of breath, chest pain, nausea, dizziness, derealization/depersonalization, fear of losing control or dying. The patient does not report having panic attacks. Obsessive-compulsive disorder (OCD) is characterized by recurrent obsessions (intrusive thoughts or images that cause anxiety) and/or compulsions (repetitive behaviors or mental acts that aim to reduce anxiety). The patient does not report having obsessions or compulsions. Post-traumatic stress disorder (PTSD) is characterized by exposure to a traumatic event that involved actual or threatened death, serious injury, or sexual violence, and subsequent re-experiencing, avoidance, negative alterations in cognition and mood, and increased arousal related to the event. The patient denies any history of trauma. Adjustment disorder is characterized by emotional or behavioral symptoms that develop within 3 months of an identifiable psychosocial stressor and cause significant impairment in social, occupational, or other areas of functioning. The symptoms usually resolve within 6 months after the termination of the stressor unless it has chronic consequences. The patient’s anxiety is not related to any specific stressors and has lasted longer than 6 months. (Review PANRE Blueprint Topic: Generalized Anxiety Disorder ) 10. A 65-year-old man with a history of type 2 diabetes mellitus presents to the emergency department with altered mental status, polyuria, and polydipsia. He has been feeling unwell for the past week with a urinary tract infection that he has been self-treating with cranberry juice. His vital signs are: blood pressure 180/100 mmHg, heart rate 110 beats per minute, respiratory rate 24 breaths per minute, temperature 37.8°C (100°F), and oxygen saturation 95% on room air. His physical examination reveals dry mucous membranes, poor skin turgor, and decreased level of consciousness. His laboratory tests show: Serum glucose: 900 mg/dL Serum sodium: 150 mEq/L Serum potassium: 4.0 mEq/L Serum bicarbonate: 18 mEq/L Serum osmolality: 350 mOsm/kg Urine ketones: negative Which of the following is the most appropriate initial treatment? A) Intravenous insulin infusion B) Intravenous normal saline infusion C) Intravenous sodium bicarbonate infusion D) Intravenous potassium chloride infusion E) Subcutaneous insulin glargine injection Answer and topic summary The correct answer is B) Intravenous normal saline infusion This patient has hyperosmolar hyperglycemic syndrome (HHS), which is a complication of diabetes mellitus characterized by severe hyperglycemia (>600 mg/dL), hyperosmolality (>320 mOsm/kg), and dehydration in the absence of ketoacidosis. HHS is often triggered by an acute stressor such as infection, medication noncompliance, or excessive carbohydrate intake. The mainstay of treatment for HHS is fluid replacement with isotonic saline to correct dehydration, lower serum glucose and osmolality, and improve renal function. Insulin therapy can be initiated after adequate fluid resuscitation, usually at a low dose (0.05-0.1 units/kg/hour). Electrolyte abnormalities such as hypokalemia or acidosis should be corrected as needed. Incorrect answer explanations: Insulin infusion is not the first-line treatment for HHS, as it can worsen dehydration, hypokalemia, and cerebral edema if given before adequate fluid replacement. Insulin should be started after fluid resuscitation at a low dose to avoid rapid drops in serum glucose and osmolality. Sodium bicarbonate infusion is not indicated for HHS unless there is severe acidosis (pH <7.0). The patient’s serum bicarbonate level is mildly low (18 mEq/L), but not enough to warrant bicarbonate therapy. Moreover, sodium bicarbonate can increase serum osmolality and worsen cerebral edema. Intravenous potassium chloride infusion is unnecessary for HHS unless there is hypokalemia (<3.5 mEq/L). The patient’s serum potassium level is normal (4.0 mEq/L), so potassium supplementation is not required. However, potassium levels should be monitored closely during fluid and insulin therapy, as they may drop rapidly due to intracellular shifts. Insulin glargine injection is a long-acting insulin that provides basal coverage for up to 24 hours. It is not suitable for treating acute hyperglycemia in HHS, as it has a slow onset of action and cannot be titrated easily according to blood glucose levels. Moreover, subcutaneous insulin administration may be unreliable in patients with poor perfusion due to dehydration. This podcast is available on every device! You can download and listen to past FREE episodes here , on iTunes , Spotify , Google Podcasts , Stitcher , Amazon Music , and all podcasting apps. Download Interactive Content Blueprint Checklists for the PANCE, PANRE, EOR, and PANRE-LA Follow this link to download your FREE copy of the PANCE/PANRE/EOR Content Blueprint Checklists . Print it up and start crossing out the topics you understand, marking the ones you don’t, and making notes of key terms you should remember. The PDF version is interactive and linked directly to the individual lessons on Smarty PANCE.…
T
The Audio PANCE and PANRE Physician Assistant Board Review Podcast


Listen to Podcast Episode 102: Ten PANCE, PANRE, and Rotation Review Questions If you can’t see the audio player, click here to listen to the full episode. Welcome to episode 102 of the Audio PANCE and PANRE Physician Assistant/Associate Board Review Podcast. Join me today as we cover ten board review questions for your PANCE, PANRE, EOR™, and EOC™ exams. Special from today’s episode: Read The New 2023 PANRE and PANRE-LA: Everything you Need to Know Members can try out the newly updated PANRE-LA Smart Search (you must log in to access the search bar) Sign up for the Entire Blueprint Email Series Follow Smarty PANCE and The Daily PANCE Blueprint on Instagram Follow Smarty PANCE and The Daily PANCE Blueprint on Facebook Join the Smarty PANCE Member’s Community then sign up for a study group to get updates about upcoming webinars. I hope you enjoy this free audio component of the examination portion of this site. Smarty PANCE includes over 2,000 interactive board review questions, along with flashcards, ReelDx cases, integrated Picmonics, and lessons covering every blueprint topic available to all Smarty PANCE members . You can download and listen to past FREE episodes here , on iTunes , Spotify , Google Podcasts , Stitcher , and most podcasting apps. You can listen to all the latest episodes, take interactive quizzes, and download more resources on each episode page. Interactive exam to complement today’s podcast 1. Which of the following is NOT true about a non-ST elevation myocardial infarction? A. Non-enteric-coated, chewable aspirin 325 mg should be given B. Troponins are elevated C. It happens due to a partially occluded epicardial coronary artery D. Patients need a 12-lead EKG E. You will always see ST depressions Answer and topic summary The answer is E. You will always see ST depressions A non-ST elevation myocardial infarction (NSTEMI) is defined by the absence of persistent ST-elevation with elevated cardiac biomarkers (e.g., troponin I or T, CKMB, etc). It happens due to a partially occluded epicardial coronary artery (leading to subendocardial ischemia ). NSTEMI typically presents as pressure-type chest pain. Patients with a suspected NSTEMI should receive a 12-lead EKG within 10 minutes of arrival. ST depression, transient ST-elevation, and/or T-wave inversions may be seen on EKG, but they are NOT required for the diagnosis of NSTEMI. The most important medication to give is non-enteric-coated chewable aspirin 325 mg . Other meds include sublingual nitroglycerin, oxygen as needed, beta-blockers (assuming no C/I), high-intensity statin, ACE inhibitors (if CKD, DM, or EF < 40%), P2Y12 inhibitor (e.g., clopidogrel), anticoagulation, and possibly PCI with stenting or CABG. Smarty PANCE Content Blueprint Review: Covered under ⇒ PANCE Blueprint Cardiology ⇒ Coronary Heart Disease ⇒ Acute myocardial infarction ⇒ Non-ST-Segment Elevation MI (NSTEMI) Also covered as part of the Family Medicine EOR , Internal Medicine EOR , Emergency Medicine EOR topic list 2. A 22-year-old G1P0 female at 28 weeks gestation with a history of diabetes presents to the clinic with fever, chills, and dysuria. Vitals show tachycardia (115 bpm), tachypnea (22 bpm), and hypotension (90/58 mmHg). Physical exam reveals suprapubic tenderness. Labs reveal leukocytosis, hyponatremia, and hyperglycemia. Urine dipstick is positive for nitrites, blood, glucose, and ketones. Which of the following is the next best step? A. Send home with oral antibiotics and analgesics B. Reassurance and oral rehydration solution C. Admit to hospital for antibiotics, fluids, & insulin D. Order outpatient CT scan of the abdomen E. Refer to an outpatient nephrologist for workup Answer and topic summary The answer is C. Admit to hospital for antibiotics, fluids, & insulin The patient most likely has pyelonephritis , which is an infection of the upper urinary tract and kidneys . It is one of the most common causes of sepsis in pregnancy. The pyelonephritis is also precipitating diabetic ketoacidosis in this diabetic patient. Clinical features of pyelonephritis include fever, flank pain, N/V , and possible CVA tenderness. Pregnant patients are at high risk for obstetric and medical complications from the infection. It is recommended that pregnant women with acute pyelonephritis should be admitted for IV antibiotics (typically at least until the woman is febrile for 1-2 days and symptomatically improved). Antibiotic options include cefepime, piperacillin-tazobactam, and meropenem. This patient will also need insulin and fluids. Smarty PANCE Content Blueprint Review: Covered under ⇒ PANCE Blueprint Genitourinary ⇒ Infectious Disorders ⇒ Pyelonephritis Also covered as part of the Family Medicine EOR , Internal Medicine EOR , and Emergency Medicine EOR topic list 3. Which of the following is the most common benign neoplasm of the liver? A. Hepatocellular carcinoma B. Hepatic hemangioma C. Liver angiosarcoma D. Hepatocellular adenoma E. Hepatoblastoma Answer and topic summary The answer is B. Hepatic hemangioma Hepatic hemangiomas are the most common benign liver lesions. The typical patient is a 30 to a 50-year-old woman. Exposure to estrogen may increase the size of hepatic hemangiomas. Patients are typically asymptomatic; however, if they do have symptoms they may have RUQ abdominal pain. An ultrasound will show a homogeneous, hyperechoic mass. If lesions are < 5 cm, you usually don’t need to do anything. If > 5 cm, monitoring is needed Q6-12 months via MRI. Smarty PANCE Content Blueprint Review: Covered under ⇒ PANCE Blueprint GI and Nutrition ⇒ Gastrointestinal System Neoplasms ⇒ Liver neoplasms 4. A 12-year-old male presents with a sore throat, fever, dysphagia, and a muffled voice. On physical exam, the uvula is deviated. Which of the following is the most likely diagnosis? A. Pharyngitis only B. Retropharyngeal abscess C. Peritonsillar abscess D. Oral candidiasis E. Foreign body Answer and topic summary The answer is C. Peritonsillar abscess A peritonsillar abscess is a collection of pus near the tonsils. The most common causes are Streptococcus pyogenes (group A), Streptococcus anginosus , & Staphylococcus aureus . Clinical features include muffled voice, sore throat, fever, drooling, neck pain, fatigue, and decreased PO intake. On physical exam, the uvula may be deviated. Diagnostic tests that can be done include ultrasound or CT with contrast (depends). Treatment is antibiotics, drainage, and supportive care. Smarty PANCE Content Blueprint Review: Covered under ⇒ PANCE Blueprint EENT ⇒ Oropharyngeal disorders ⇒ Infectious and inflammatory disorders ⇒ Peritonsillar abscess Also covered as part of the Pediatric EOR , Family Medicine EOR , and Emergency Medicine EOR topic list 5. Which of the following is the most common cause of acute epiglottitis? A. Staphylococcus aureus B. Haemophilus influenzae C. Streptococcus pneumoniae D. Streptococcus pyogenes E. Neisseria meningitidis Answer and topic summary The answer is B. Haemophilus influenzae Acute epiglottitis is an inflammatory condition of the epiglottis (usually due to a bacterial infection). In severe situations, it is life-threatening. The most common cause overall is said to be Haemophilus influenzae ; however, in adults, Streptococcal species now tend to be more common. The management of this disease should focus on giving antibiotics and steroids . Airway management and securing an airway is the most important part of treatment. Smarty PANCE Content Blueprint Review: Covered under ⇒ PANCE Blueprint Pulmonary ⇒ Infectious Pulmonary Disorders ⇒ Acute epiglottitis Also covered as part of the Pediatric EOR and Emergency Medicine EOR topic list 6. A 32-year-old female presents with pain around the nail fold on the index finger of her right hand. She is afebrile. On physical exam, you appreciate erythema and swelling of the proximal nail fold. The area is not fluctuant. Which of the following is the best treatment option? A. Topical antibiotics and warm water soaks B. Intravenous antibiotics C. Oral antibiotics and incision and drainage D. Surgical consult for removal E. None of the above Answer and topic summary The answer is A. Topical antibiotics and warm water soaks The patient has acute paronychia, which is a very common condition and essentially is just inflammation of the nail folds. The most common bacterial causes are Staph aureus and Strep pyogenes . Risk factors include manicures, nail biting, and picking at nails. Clinical features include sudden onset of painful erythema and swelling. Sometimes an abscess will be present. In this patient’s case, there was no fluctuance and so topical antibiotics and warm water soaks would be appropriate. If the patient had an abscess, oral antibiotics and incision/drainage (with number 11 surgical blade) would be reasonable. Smarty PANCE Content Blueprint Review: Covered under ⇒ PANCE Blueprint Infectious Disease ⇒ Bacterial Disease ⇒ Methicillin-resistant Staphylococcus aureus infection 7. A 60-year-old male presents with fatigue and weight loss. His physical exam is remarkable for gingival hyperplasia and splenomegaly. Labs reveal pancytopenia. A bone marrow biopsy reveals Auer rods. What is the likely diagnosis? A. Acute myeloid leukemia B. Acute lymphocytic leukemia C. Chronic myeloid leukemia D. Chronic lymphocytic leukemia E. Non-Hodgkin’s lymphoma Answer and topic summary The answer is A. Acute myeloid leukemia The patient has acute myeloid leukemia (AML) , which is a group of cancers that involve the myeloid precursor cells. It is characterized by clonal proliferation (excessive growth) of abnormal myeloid precursor cells. Risk factors include smoking and chemotherapy/radiation. Clinical features include fatigue, pallor, weakness, bone pain, gingival bleeding, and organomegaly. Lab findings are variable but may include pancytopenia (decrease in RBCs, WBCs, and platelets), electrolyte derangements, and/or hypoxemia. Workup should include a peripheral blood smear and bone marrow biopsy. The bone marrow biopsy will classically show Auer rods and >20% blasts. Smarty PANCE Content Blueprint Review: Covered under ⇒ PANCE Blueprint Hematology ⇒ Neoplasms, premalignancies, and malignancies ⇒ Acute and chronic myelogenous leukemia Also covered as part of the Family Medicine EOR topic list 8. A 42-year-old female presents for an annual physical exam with some complaints of fatigue and dyspnea. On physical exam, you appreciate a fixed S2 split. Which of the following is the most likely diagnosis? A. Ventricular septal defect B. Mitral regurgitation C. Aortic stenosis D. Atrial septal defect E. Patent ductus arteriosus Answer and topic summary The answer is D. Atrial septal defect Atrial septal defect is the most common congenital heart lesion in adults. People often don’t have symptoms until adulthood. The most common type is ostium secundum . Symptoms include fatigue and dyspnea. A classic physical exam finding on a test would be a systolic ejection crescendo-decrescendo flow murmur @ LUSB with FIXED S2, loud S1 . Echocardiography is the initial imaging modality of choice. Keep in mind that a complication of ASD is that the left to right shunt can cause volume and pressure overload of the right heart and pulmonary circulation, leading to pulmonary hypertension. Smarty PANCE Content Blueprint Review: Covered under ⇒ PANCE Blueprint Cardiology ⇒ Congenital Heart Disease ⇒ Atrial septal defect Also covered as part of the Pediatric EOR topic list 9. Which of the following is not a test for acute appendicitis? A. Murphy sign B. Rovsing sign C. Obturator sign D. Psoas sign E. McBurney sign Answer and topic summary The answer is A. Murphy sign Appendicitis is one of the most common indications for emergent abdominal surgery. The most common physical exam finding is RLQ tenderness. There are some tests that may help with the diagnosis — keep in mind though that these tests/signs are NOT sensitive. Rovsing sign: pain in the RLQ with palpation of the LLQ (indicative of peritoneal irritation) McBurney sign: tenderness about 2 inches from the ASIS (on a straight line from ASIS to the umbilicus) Psoas sign: RLQ pain with passive right hip extension Obturator sign: flexion of the right hip and knee, followed by internal rotation of the right hip elicits RLQ pain Murphy’s sign is positive in acute cholecystitis, not appendicitis. A positive test is RUQ pain on inspiration. Smarty PANCE Content Blueprint Review: Covered under ⇒ PANCE Blueprint GI and Nutrition ⇒ Diseases of the Small Intestine ⇒ Appendicitis Also covered as part of the Emergency Medicine EOR , Family Medicine EOR , Pediatric EOR , and General Surgery EOR topic list 10. A 31-year-old female presents with pain and numbness in her anterior tibial region. On physical exam, the area is pale, cold, and pulseless. Which of the following is the best definitive treatment for this likely diagnosis? A. Antiplatelets B. Amputation C. Physical therapy D. Fasciotomy E. IV heparin drip Answer and topic summary The answer is D. Fasciotomy The patient has acute compartment syndrome , which is when the tissue pressure within a closed muscle compartment exceeds the perfusion pressure. It can result in ischemia. The most common location is the calf. Many cases of acute compartment syndrome in the lower extremity are associated with fractures, burn injuries, crush injuries, or soft tissue infections. The “5 P’s” can help you remember the symptoms – pain , pallor , paresthesia , pulselessness , paralysis . Extremity fasciotomy is the treatment for acute compartment syndrome. Smarty PANCE Content Blueprint Review: Covered under ⇒ PANCE Blueprint Musculoskeletal ⇒ Compartment Syndrome This podcast is available on every device! You can download and listen to past FREE episodes here , on iTunes , Spotify , Google Podcasts , Stitcher , Amazon Music , and all podcasting apps. Download Interactive Content Blueprint Checklists for the PANCE, PANRE, EOR, and PANRE-LA Follow this link to download your FREE copy of the PANCE/PANRE/EOR Content Blueprint Checklists . Print it up and start crossing out the topics you understand, marking the ones you don’t, and making notes of key terms you should remember. The PDF version is interactive and linked directly to the individual lessons on Smarty PANCE.…
T
The Audio PANCE and PANRE Physician Assistant Board Review Podcast


Welcome to episode 101 of the Audio PANCE and PANRE Physician Assistant/Associate Board Review Podcast. Join me today as we cover ten board review questions for your PANCE, PANRE, and rotation exams. Special from today’s episode: Join the Smarty PANCE Member’s Community then sign up for a study group to get updates about upcoming webinars. Check out our updated End of Curriculum™ (EOC) Exam Course Sign up for the Entire Blueprint Email Series Follow Smarty PANCE and The Daily PANCE Blueprint on Instagram Follow Smarty PANCE and The Daily PANCE Blueprint on Facebook I hope you enjoy this free audio component of the examination portion of the Smarty PANCE website. The full board review website includes over 2,000 interactive board review questions, flashcards, and blueprint lessons available to all members of Smarty PANCE . You can download and listen to past FREE episodes here , on iTunes , Spotify , Google Podcasts , Stitcher , and most podcasting apps. You can listen to all the latest episodes, take interactive quizzes, and download more resources on each episode page. Listen to Podcast Episode 101: Ten PANCE, PANRE, and Rotation Review Questions If you can’t see the audio player, click here to listen to the full episode. Interactive exam to complement today’s podcast 1. A 75-year-female smoker with a history of atrial fibrillation and hypertension presents to the ER complaining of a 2-hour history of right-sided weakness and aphasia that has now resolved. Her physical exam and vital signs are completely unremarkable. CT head is unremarkable. Which of the following is the most likely diagnosis? A. Subarachnoid hemorrhage B. Transient ischemic attack C. Cerebral venous sinus thrombosis D. Multiple sclerosis E. Complicated migraine Answer and topic summary The answer is B. Transient ischemic attack The patient had a transient ischemic attack (TIA), which is characterized by transient neurological symptoms without objective evidence of acute infarction. Symptoms vary, but patients may have hemiparesis , hemiplegia , aphasia , or vision loss . Risk factors include alcohol, hypertension, smoking, diabetes, illicit drug use, and atrial fibrillation. Urgent evaluation is needed in patients with symptoms of TIA (e.g., coagulation studies, TTE, EKG, CT or MRI head, imaging of the cervicocephalic vasculature via carotid US, CTA, or MRA). Treatment for high-risk patients includes dual antiplatelet therapy (ASA + clopidogrel) for 21 days and risk factor management (statin, exercise, etc). Remember the risk of an actual stroke is high after a TIA. View blueprint lesson Smarty PANCE Content Blueprint Review: Covered under ⇒ PANCE Blueprint Neurology ⇒ Vascular Disorder ⇒ Transient ischemic attack Also covered as part of the Internal Medicine EOR , Family Medicine EOR , and Emergency Medicine EOR topic list 2. A 26-year-old female presents with a history of miscarriages and recurrent pulmonary embolism. She is diagnosed with antiphospholipid syndrome. Which of the following is the mainstay of treatment for this condition? A. Dual antiplatelet therapy (DAPT) B. Aspirin and heparin C. Dabigatran D. Warfarin E. Heparin Answer and topic summary The answer is D. Warfarin Antiphospholipid syndrome (APS) is an autoimmune disease defined by venous thromboembolism, arterial thrombosis, and obstetric morbidity in the presence of circulating antiphospholipid antibodies (aPLs). It is the most common form of acquired thrombophilia . aPLs include lupus anticoagulant , anticardiolipin , anti-b2-glycoprotein I antibodies. Clinical features include DVTs (32%), thrombocytopenia (22%), livedo reticularis (20%), stroke (13%), PEs (9%), and fetal loss (8%). The mainstay of treatment is warfarin . Other anticoagulants have been found to be less effective than warfarin so far. View blueprint lesson Smarty PANCE Content Blueprint Review: Covered under ⇒ PANCE Blueprint Hematology ⇒ Coagulation Disorders ⇒ Hypercoagulable states Also covered as part of the Internal Medicine EOR and Emergency Medicine EOR topic list 3. Which of the following is the most common cause of acute bronchitis in the United States? A. Virus B. Bacteria C. Fungus D. Allergies E. Tuberculosis Answer and topic summary The answer is A. Virus Acute bronchitis is characterized by a cough due to inflammation of the large airways and trachea with no evidence of pneumonia. Acute bronchitis often follows an upper respiratory infection. The most common cause of acute bronchitis is viral. Symptoms include cough (10-20 days), headache, shortness of breath, and wheezing. The presence of fever should make you consider pneumonia or influenza. Diagnosis is clinical. A chest radiograph should be given if it is hard to clinically distinguish between pneumonia and bronchitis. Treatment is mainly supportive care (rest, hydration) and symptomatic management (antitussives for cough, bronchodilators for wheezing, etc). Antibiotics should NOT be given for acute bronchitis. View blueprint lesson Smarty PANCE Content Blueprint Review: Covered under ⇒ PANCE Blueprint Pulmonary ⇒ Infectious Pulmonary Disorders ⇒ Acute bronchitis Also covered as part of the Emergency Medicine EOR topic list 4. A 27-year-old male presents to the clinic with shortness of breath, chest pain, and fatigue. His physical exam is remarkable for a crescendo-decrescendo systolic murmur heard at the apex. An echocardiogram reveals asymmetric septal hypertrophy and left ventricular hypertrophy. Which of the following is the best initial treatment option? A. Procainamide B. Hydrochlorothiazide C. Beta-blockers D. Aspirin E. ACE inhibitor Answer and topic summary The answer is C. Beta-blockers The patient has hypertrophic cardiomyopathy (HCM), which is a genetic disease of the heart muscle due to mutations in the sarcomere genes . It often presents with fatigue, dyspnea, chest pain, or syncope. The murmur on the physical exam is due to LV obstruction; it will be a harsh crescendo-decrescendo systolic murmur heard at the apex and LLSB . It’s important to keep in mind that the murmur intensity will decrease with more venous return (squatting) and increase with less venous return (Valsalva). Diagnostic tests include an EKG , echocardiogram , and exercise stress testing . Treatment is indicated for symptomatic patients and includes a negative inotropic agent (i.e., nondihydropyridine CCBs or beta-blockers) and diuretics as needed for volume overload. If patients are refractory to medications, septal myectomy and percutaneous septal ablation can be performed. View blueprint lesson Smarty PANCE Content Blueprint Review: Covered under ⇒ PANCE Blueprint Cardiology ⇒ Cardiomyopathy ⇒ Hypertrophic Cardiomyopathy Also covered as part of the Pediatric Rotation EOR topic list 5. A 22-year-old female presents to the ER with crampy lower abdominal pain and vaginal bleeding for the past 2 hours. She missed her last menstrual cycle. The physical exam is remarkable for an open cervical os. No products of conception are observed. Which of the following is the most likely diagnosis? A. Threatened abortion B. Incomplete abortion C. Missed abortion D. Septic abortion E. Inevitable abortion Answer and topic summary The answer is E. Inevitable abortion The patient most likely has a spontaneous abortion, which is a nonviable intrauterine pregnancy up to 20 weeks gestation. The most common cause of miscarriages in the first trimester is chromosomal abnormalities . There are many different types of spontaneous abortions. The patient specifically has an inevitable abortion , which typically presents with vaginal bleeding, crampy abdominal pain, and an open external cervical os without expulsion of products of conception. The fetus is not viable. View blueprint lesson Smarty PANCE Content Blueprint Review: Covered under ⇒ PANCE Blueprint Reproductive System ⇒ Complicated Pregnancy ⇒ Abortion Also covered as part of the Women’s Health EOR topic list 6. Which of the following is not a risk factor for Methicillin-resistant Staphylococcus aureus (MRSA)? A. Previous antibiotic use B. Injection drug use C. Indwelling hemodialysis catheter D. Presence in long-term facility E. All of the above are risk factors Answer and topic summary The answer is E. All of the above are risk factors Methicillin-resistant Staphylococcus aureus (MRSA) infection is caused by a certain type of S. aureus that is resistant to several antibiotics. Risk factors include previous antibiotic use, injection drug use, indwelling hemodialysis catheter, living in a long-term care facility, HIV infection, and recent hospitalization. The most common site for MRSA infections to occur is the skin/soft tissue . Oral antibiotics that cover MRSA include Bactrim , doxycycline , and clindamycin . For severe MRSA infections, IV vancomycin is preferred. View blueprint lesson Smarty PANCE Content Blueprint Review: Covered under ⇒ PANCE Blueprint Infectious Disease ⇒ Bacterial Disease ⇒ Methicillin-resistant Staphylococcus aureus infection 7. A 60-year-old male presents with fatigue and weight loss. His physical exam is remarkable for gingival hyperplasia and splenomegaly. Labs reveal pancytopenia. A bone marrow biopsy reveals Auer rods. What is the likely diagnosis? A. Acute myeloid leukemia B. Acute lymphocytic leukemia C. Chronic myeloid leukemia D. Chronic lymphocytic leukemia E. Non-Hodgkin’s lymphoma Answer and topic summary The answer is A. Acute myeloid leukemia The patient has acute myeloid leukemia (AML) , which is a group of cancers that involve the myeloid precursor cells. It is characterized by clonal proliferation (excessive growth) of abnormal myeloid precursor cells. Risk factors include smoking and chemotherapy/radiation. Clinical features include fatigue, pallor, weakness, bone pain, gingival bleeding, and organomegaly. Lab findings are variable but may include pancytopenia (decrease in RBCs, WBCs, and platelets), electrolyte derangements, and/or hypoxemia. The workup should include a peripheral blood smear and bone marrow biopsy. The bone marrow biopsy will classically show Auer rods and >20% blasts. View blueprint lesson Smarty PANCE Content Blueprint Review: Covered under ⇒ PANCE Blueprint Hematology ⇒ Neoplasms, premalignancies, and malignancies ⇒ Acute and chronic myelogenous leukemia Also covered as part of the Internal Medicine EOR , Emergency Medicine EOR , Family Medicine EOR , and Pediatric EOR topic list 8. Giant Cell Arteritis: The Daily PANCE Blueprint A 55-year-old female presents with right-sided headaches, jaw pain upon chewing, and mild dizziness for the past few months. On physical exam, the right temporal artery is tender to palpation. Labs reveal an elevated ESR and CRP. What is the definitive diagnosis for this condition? A. CT head B. Temporal artery biopsy C. PET scan D. Ultrasound E. IgA antibody level Answer and topic summary The answer is B. Temporal artery biopsy The patient has temporal arteritis, which is a vasculitis of large and medium vessels. Clinical features often include fever , fatigue , weight loss , headache , jaw claudication , and transient vision loss . The typical clinical vignette is an older female presenting with a headache and jaw claudication. On physical exam, you may appreciate a temporal artery that is thickened, tender, or erythematous. ESR and CRP are sensitive, but not specific for the diagnosis. The definitive diagnosis of temporal arteritis is a temporal artery biopsy . Treatment is high-dose steroids . It’s important that temporal arteritis is treated because it can lead to blindness, thrombotic events, and other complications. View blueprint lesson Smarty PANCE Content Blueprint Review: Covered under ⇒ PANCE Blueprint Cardiology ⇒ Vascular Disease ⇒ Giant cell arteritis Also covered as part of the Internal Medicine EOR topic list 9. Which of the following is one of the most common initial clinical symptoms in multiple sclerosis? A. Vertigo B. Extreme weakness C. Depression D. Arthralgias E. Optic neuritis Answer and topic summary The answer is E. Optic neuritis Multiple sclerosis (MS) is a demyelinating disease of the central nervous system. There are many types of MS patterns. Clinical features suggestive of MS include sensory loss in limbs or one side of the face, gait/balance issues , heat sensitivity (Uhthoff), fatigue, and Lhermitte sign. The most common presenting symptom is optic neuritis . MRI of the brain (+/- spinal cord) is the diagnostic test of choice. The McDonald diagnostic criteria can be used to help make the diagnosis of MS. Treatment of MS includes disease-modifying pharmacotherapy , such as glatiramer acetate (Copaxone), dimethyl fumarate, natalizumab (Tysabri), interferons, and many more. Acute exacerbations of MS are managed with short-term, high-dose glucocorticoids . View blueprint lesson Smarty PANCE Content Blueprint Review: Covered under ⇒ PANCE Blueprint Neurology ⇒ Neuromuscular disorders ⇒ Multiple sclerosis Also covered as part of the Internal Medicine EOR topic list 10. A 52-year-old male presents to the clinic complaining of “sharp burning” in his lower chest after eating a large meal. The pain goes away when he raises the head of the bed. Which of the following is the best pharmacological agent for this likely diagnosis? A. Esomeprazole B. Nitroglycerin C. Aspirin D. Codeine E. Sucralfate Answer and topic summary The answer is A. Esomeprazole The patient has gastrointestinal reflux disease (GERD), which is a common condition characterized by stomach acid flowing up into the esophagus. Classic symptoms include heartburn (burning sensation around the sternum) after eating, water brash , regurgitation , and dysphagia. The diagnosis is usually clinical; however, if the patient has alarming symptoms (e.g., weight loss, anemia, etc), it is appropriate to get an upper endoscopy. The gold standard is an ambulatory 24-hour pH monitoring system . Initial management includes weight loss, elevating the head of the bed, and eliminating triggers (spicy foods, chocolate, etc). Most recommend starting with histamine 2 receptor antagonist therapy first (e.g., ranitidine) and then transitioning to proton pump inhibitors (e.g., esomeprazole). View blueprint lesson Smarty PANCE Content Blueprint Review: Covered under ⇒ PANCE Blueprint GI and Nutrition ⇒ Esophageal Disorders ⇒ Gastroesophageal reflux disease Also covered as part of the Family Medicine EOR , Internal Medicine EOR , and Pediatric EOR topic list This podcast is available on every device! You can download and listen to past FREE episodes here , on iTunes , Spotify , Google Podcasts , Stitcher , Amazon Music , and all podcasting apps. Download Interactive Content Blueprint Checklists for the PANCE, PANRE, EOR, and PANRE-LA Follow this link to download your FREE copy of the PANCE/PANRE/EOR Content Blueprint Checklists . Print it up and start crossing out the topics you understand, marking the ones you don’t, and making notes of key terms you should remember. The PDF version is interactive and linked directly to the individual lessons on Smarty PANCE.…
T
The Audio PANCE and PANRE Physician Assistant Board Review Podcast


Welcome to episode one hundred of the Audio PANCE and PANRE Physician Assistant/Associate Board Review Podcast. Join me today as we cover atrial fibrillation for the PANCE, PANRE, and EOR™ exams. Special from today’s episode: Join the Smarty PANCE Member’s Community then sign up for the Sunday Funday Study Group . Once you have signed up, you can access the Webinar replay of this episode here Check out our updated End of Curriculum™ (EOC) Exam Course (now with orthopedics!) Sign up for the Entire Blueprint email series Follow Smarty PANCE and The Daily PANCE Blueprint on Instagram Follow Smarty PANCE and The Daily PANCE Blueprint on Facebook Below you will find the audio of today’s podcast. The Audio PANCE/PANRE and EOR PA Board Review Podcast I hope you enjoy this free audio component to the examination portion of this site. The full board review course includes over 2,000 interactive board review questions and is available to all members of Smarty PANCE . You can download and listen to past FREE episodes here , on iTunes , Spotify , Google Podcasts , Stitcher , and most podcasting apps. You can listen to all the latest episodes, take interactive quizzes, and download more resources on each episode page. Listen to Podcast Episode 100: Atrial Fibrillation for the PANCE and PANRE If you can’t see the audio player, click here to listen to the full episode.…
T
The Audio PANCE and PANRE Physician Assistant Board Review Podcast


Welcome to episode 99 of the Audio PANCE and PANRE Physician Assistant/Associate Board Review Podcast. Join me as I cover ten PANCE, PANRE, and EOR™ review questions from the Smarty PANCE Instagram/Facebook page and the smartypance.com board review website. Special from today’s episode: Join the Smarty PANCE Member’s Community Check out our all-new End of Curriculum™ (EOC) Exam Course (still in development) Follow Smarty PANCE and The Daily PANCE Blueprint on Instagram Follow Smarty PANCE and The Daily PANCE Blueprint on Facebook Below you will find an interactive exam to complement today’s podcast. The Audio PANCE/PANRE and EOR PA Board Review Podcast I hope you enjoy this free audio component to the examination portion of this site. The full board review course includes over 2,000 interactive board review questions and is available to all members of Smarty PANCE . You can download and listen to past FREE episodes here , on iTunes , Spotify , Google Podcasts , Stitcher , and most podcasting apps. You can listen to the latest episode, take an interactive quiz, and download more resources below. Listen Carefully Then Take the Practice Exam If you can’t see the audio player, click here to listen to the full episode. Podcast Episode 99: Ten PANCE/PANRE and EOR Topic Blueprint Questions 1. A 42-year-old male on lithium presents with polyuria, nocturia, and polydipsia. Laboratory findings are remarkable for slightly elevated sodium. Which of the following is the most likely diagnosis? A. Neurogenic diabetes insipidus B. Nephrogenic diabetes insipidus C. Type 2 diabetes mellitus D. SIADH E. Adrenal insufficiency Click here to see the answer The answer is B. Nephrogenic diabetes insipidus Nephrogenic diabetes insipidus (DI) is a condition where the kidneys are resistant to the effects of ADH. Nephrogenic DI can occur secondary to lithium toxicity or chronic lithium use , pregnancy , inherited disorders , and electrolyte issues . Clinical features include polyuria, nocturia, and polydipsia. Serum sodium is either normal or high. In this patient, lithium has entered the collecting duct, accumulated, and interfered with ADH’s capacity to increase water permeability. Treatment options for this patient include amiloride or thiazide diuretics . In general, whether or not lithium is discontinued depends on many factors. VIEW BLUEPRINT LESSON Smarty PANCE Content Blueprint Review: Covered under ⇒ PANCE Blueprint Endocrinology ⇒ Pituitary Disorders ⇒ Diabetes insipidus Also covered as part of the Internal Medicine EOR and Emergency Medicine PAEA EOR topic list 2. A 50-year-old female presents with poor appetite, low energy, poor concentration, and feelings of hopelessness on most days for the past 3 years. She denies suicidal ideation. She has never had a past manic or hypomanic episode. Which of the following is the best treatment option? A. Haloperidol B. Fluoxetine C. Lorazepam D. Amitriptyline E. Risperidone Click here to see the answer The answer is B. Fluoxetine This patient has dysthymia, or persistent depressive disorder. The DSM V criteria follow: Depressed mood ≥ 2 years on most days At least 2 of the following: appetite changes, sleep changes, low energy, low self-esteem, poor concentration, hopelessness Not without symptoms > 2 months at a time No mania or hypomania episodes , ever Like other psychiatric disorders, symptoms can’t be attributable to drugs, and the symptoms must cause distress/impairment. 1st line treatment is selective serotonin reuptake inhibitors and psychotherapy . VIEW BLUEPRINT LESSON Smarty PANCE Content Blueprint Review: Covered under ⇒ PANCE Blueprint Psychiatry ⇒ Depressive disorders ⇒ Persistent depressive disorder (dysthymia) Also covered as part of the Psychiatry EOR , Emergency Medicine EOR , and Pediatric PAEA EOR topic list 3. An 85-year-old male with a history of chronic kidney disease presents to the ER with muscle cramps. Laboratory studies reveal potassium of 7.8 mEq/L. EKG reveals peaked T waves. Which of the following is the best initial med to give? A. Insulin B. Albuterol C. Furosemide D. Sodium bicarbonate E. Calcium gluconate Click here to see the answer The answer is E. Calcium gluconate Hyperkalemia is caused by many things: iatrogenic (ACEI/ARBs), cellular destruction (hemolysis, tumor lysis syndrome, burns), renal failure , adrenal insufficiency , etc. Symptoms are nonspecific and include muscle weakness, N/V, decreased DTRs, etc. Classically on EKG, you may see peaked T waves (there are many other findings though…such as QRS widening, PR interval prolongation, sine-wave, etc). Management for severe hyperkalemia includes calcium gluconate (stabilizes the resting membrane potential of the myocardial membrane), shifting potassium intracellularly (via beta-agonists , sodium bicarbonate , insulin ), and removing potassium (via diuretics , kayexalate , or hemodialysis ). VIEW BLUEPRINT LESSON Smarty PANCE Content Blueprint Review: Covered under ⇒ PANCE Blueprint Renal System ⇒ Fluid and Electrolyte Disorders ⇒ Hyperkalemia/hypokalemia Also covered as part of the Emergency Medicine EOR and General Surgery PAEA EOR topic list 4. A 28-year-old male with sickle cell disease presents to the ER with chest pain, dyspnea, and a cough for the past day. Vitals are remarkable for SpO2 91% and T 102.2F. A CXR reveals bilateral pulmonary infiltrates. On physical exam the patient is alert, speaking in full sentences, and breathing without accessory muscle use. Which of the following is the most appropriate management option for this patient? A. Surgical consultation, antibiotics, plasmapheresis, IVIG B. Pain control, hydration, blood transfusions, oxygen, antibiotics, VTE prophylaxis C. Immediate endotracheal intubation, aggressive intravenous fluids, antibiotics, blood transfusions D. Blood transfusions and oxygen only E. Empiric antibiotics and oxygen only Click here to see the answer The answer is B. Pain control, hydration, blood transfusions, oxygen, antibiotics, VTE prophylaxis The patient has acute chest syndrome (ACS), which is defined as a new radiodensity on CXR with fever and/or pulmonary symptoms . It occurs due to vaso-occlusion in the pulmonary microvasculature, subsequently leading to deoxygenation of hemoglobin and sickling of RBCs. ACS is the main cause of death in patients with sickle cell disease (SCD). About 1 in 2 patients with SCD get ACS. Patients may present with chest pain, extremity pain, and shortness of breath. Causes of ACS include fat emboli, infection, asthma, oversedation, and post-op issues. Treatment typically includes fluids , oxygen , pain control , blood transfusions , antibiotics , and VTE prophylaxis . VIEW BLUEPRINT LESSON Smarty PANCE Content Blueprint Review: Covered under ⇒ PANCE Blueprint Hematology ⇒ Hemoglobinopathies ⇒ Sickle cell anemia Also covered as part of the Internal Medicine EOR and Emergency Medicine PAEA EOR topic list 5. Which of the following is the most common cause of bacterial sialadenitis? A. Bacteroides B. Staphylococcus aureus C. Streptococcus pneumoniae D. Escherichia coli E. Streptococcus viridans Click here to see the answer The answer is B. Staphylococcus aureus Bacterial sialadenitis is defined by inflammation of a salivary gland due to a bacterial infection. It is most commonly caused by Staphylococcus aureus . Usually, bacterial sialadenitis happens in the setting of a salivary gland stone (reduced salivary flow leads to bacteria building up in the mouth). Other risk factors include bad oral hygiene and old age. The most common gland affected is the parotid gland . Clinical features include acute onset of fever, chills, and swelling/tenderness of the affected gland (possible to see purulent drainage). The treatment is broad-spectrum antibiotics: IV ampicillin-sulbactam . Surgical drainage is needed if an abscess develo VIEW BLUEPRINT LESSON Smarty PANCE Content Blueprint Review: Covered under ⇒ PANCE Blueprint EENT ⇒ Salivary disorders ⇒ Sialadenitis Also covered as part of the Family Medicine PAEA EOR topic list 6. A 52-year-old female presents to the clinic complaining of chronic bone pain, constipation, and fatigue. Her last lab results reveal a decline in the glomerular filtration rate. Which of the following is the most likely diagnosis? A. Acute myelogenous leukemia B. Bronchogenic carcinoma C. Multiple myeloma D. Polymyalgia rheumatica E. Colorectal cancer Click here to see the answer The answer is C. Multiple myeloma The patient has multiple myeloma , which is a clonal proliferation of abnormal plasma cells. It can present with “ BREAK ” symptoms: B one pain, R ecurrent infections, E levated calcium, A nemia, and K idney failure. A serum and urine protein electrophoresis will reveal a monoclonal spike . Other diagnostic studies include a CT or MRI (reveals lytic bone lesions ), peripheral blood smear ( normocytic anemia in rouleaux formation ), and urinalysis ( Bence Jones proteins aka free light chains). The definitive diagnosis is a bone marrow biopsy . Treatment is high-dose chemotherapy with autologous hematopoietic cell transplantation. VIEW BLUEPRINT LESSON Smarty PANCE Content Blueprint Review: Covered under ⇒ PANCE Blueprint Hematology ⇒ Neoplasms, premalignancies, and malignancies ⇒ Multiple myeloma Also covered as part of the Internal Medicine PAEA EOR topic list 7. A 56-year-old post-menopausal G0P0 female presents to the clinic complaining of abnormal uterine bleeding and weight loss for the past 5 months. Physical exam is unremarkable. Which of the following is not a risk factor for this patient’s likely diagnosis? A. Nulliparity B. Tamoxifen C. Chronic anovulation D. Obesity E. Late menarche Click here to see the answer The answer is E. Late menarche The patient has endometrial cancer , the most commonly encountered gynecologic cancer. The cardinal symptom is abnormal uterine bleeding . Other clinical features include pelvic pain, pelvic masses, and weight loss. Risk factors include conditions or medications that lead to chronically increased estrogen levels, such as nulliparity , tamoxifen , anovulation , and obesity . Early menarche and late menopause would be risk factors, not late menarche. VIEW BLUEPRINT LESSON Smarty PANCE Content Blueprint Review: Covered under ⇒ PANCE Blueprint Reproductive System ⇒ Neoplasms of the breast and reproductive tract ⇒ Endometrial cancer Also covered as part of the Women’s Health PAEA EOR topic list 8. Which of the following are the two most common etiologies of peptic ulcer disease? A. H. pylori and NSAIDs B. Spicy foods and acute stress C. Chemical ingestion and GERD D. Smoking and alcohol use E. Chronic stress and radiation Click here to see the answer The answer is A. H. pylori and NSAIDs Peptic ulcer disease (PUD) is a common GI disease defined by a defect in the mucosal lining of the stomach or duodenum. The two most common causes are H. pylori and NSAIDs . Other more rare culprits include Zollinger-Ellison Syndrome, cancer, stress, and radiation. Surprisingly, patients are asymptomatic about 70% of the time. Common symptoms are epigastric abdominal pain , dyspepsia , and bloating . Overall, an upper endoscopy is the best diagnostic test. A biopsy should be done for ulcers with malignant features on all gastric ulcers. The preferred treatment is PPIs . If H. pylori is the cause, an antibiotic regimen is warranted (e.g., amoxicillin, clarithromycin, and a PPI). VIEW BLUEPRINT LESSON Smarty PANCE Content Blueprint Review: Covered under ⇒ PANCE Blueprint GI and Nutrition ⇒ Gastric Disorders ⇒ Peptic ulcer disease Also covered as part of the Internal Medicine EOR , Emergency Medicine EOR , Family Medicine EOR , and General Surgery EOR topic list 9. A 72-year-old male smoker with a history of cancer presents to the clinic for an annual physical. Vitals are unremarkable. On physical exam, you notice erythema along the course of a superficial vein on his left leg. The area is mildly tender to palpation. The left leg is also larger than the other leg. Which of the following is the next best step? A. Treat with NSAIDs and warm compresses B. Order a duplex ultrasound C. Initial anticoagulation immediately D. Admit to the hospital E. Reassurance and send home Click here to see the answer The answer is B. Order a duplex ultrasound The patient has superficial thrombophlebitis , which is inflammation involving a superficial vein along with the presence of a superficial clot. Risk factors include varicose veins, estrogen, prior DVT, cancer, and hypercoagulable states. Clinical features include tenderness , induration , pain , and erythematous skin over a superficial vein . In many cases, duplex ultrasound is needed to rule out a concurrent DVT (studies show that up to 25-50% of patients with superficial thrombophlebitis have a concurrent DVT). Treatment includes NSAIDs , warm/cold compresses , extremity elevation , and compression therapy . The decision to initiate anticoagulation for superficial thrombophlebitis depends on many factors. VIEW BLUEPRINT LESSON Smarty PANCE Content Blueprint Review: Covered under ⇒ PANCE Blueprint Cardiology ⇒ Vascular Disease ⇒ Phlebitis/thrombophlebitis 10. A 63-year-old male with a history of hepatitis C presents with a pruritic rash located on the flexor surfaces of his wrist. On physical exam, you observe purplish papules with flat-tops. Which of the following is the most likely diagnosis? A. Atopic dermatitis B. Psoriasis C. Lichen planus D. Pemphigus vulgaris E. Scabies Click here to see the answer The answer is C. Lichen planus Lichen planus is a skin disorder characterized by a rash associated with the 5 Ps : p olygonal, p ruritic, p apules, p laques, and p urple. It commonly affects the wrists and ankles. Lichen planus can also cause the classic Wickham’s striae , which are white-lacelike lines (usually in the oral mucosa). A risk factor is hepatitis C . A skin biopsy can help confirm the diagnosis. Treatment includes high potency topical corticosteroids . If a patient has an extensive form of lichen planus, then phototherapy, acitretin, or systemic steroids may help. VIEW BLUEPRINT LESSON Smarty PANCE Content Blueprint Review: Covered under ⇒ PANCE Blueprint Dermatology ⇒ Papulosquamous Disorders ⇒ Lichen planus Also covered as part of the Family Medicine EOR and Pediatric EOR topic list Looking for all the podcast episodes? This FREE podcast series is limited to every other episode, you can download and enjoy the complete audio series by becoming a Smarty PANCE member . I will be releasing new episodes every few weeks. Smarty PANCE is now discounted, so sign up now before it’s too late! Additional resources and links from the show Download your Free interactive PANCE, PANRE, and EOR™ Blueprint Templates Follow Smarty PANCE and The Daily PANCE Blueprint on Instagram Follow Smarty PANCE and The Daily PANCE Blueprint on Facebook My list of recommended PANCE and PANRE review books Sign up for the FREE Daily PANCE and PANRE email series Join the Smarty PANCE NCCPA Content Blueprint Website Get your free 8-week PANRE Blueprint study schedule and the 8-week PANCE study schedule Get your free Trello PANCE study planner Get 20% of any Picmonic membership by using this link This Podcast is available on iOS and Android You can download and listen to past FREE episodes here , on iTunes , Spotify , Google Podcasts , Stitcher , and most podcasting apps. Download the Interactive Content Blueprint Checklist Follow this link to download your FREE copy of the PANCE/PANRE/EOR™ Content Blueprint Checklists Print it up and start crossing out the topics you understand, marking the ones you don’t, and making notes of key terms you should remember. The PDF version is interactive and linked directly to the individual lessons on Smarty PANCE. Smarty PANCE is not sponsored or endorsed by, or affiliated with, the NCCPA or the Physician Assistant Education Association (PAEA). All trademarks are the property of their respective owners.…
T
The Audio PANCE and PANRE Physician Assistant Board Review Podcast


Welcome to episode 98 of the Audio PANCE and PANRE Physician Assistant/Associate Board Review Podcast. Join me as I cover ten PANCE, PANRE, and EOR™ review questions from the Smarty PANCE Instagram/Facebook page and the smartypance.com board review website. Special from today’s episode: Join the Smarty PANCE Member’s Community Check out our all-new End of Curriculum™ (EOC) Exam Course (still in development) Follow Smarty PANCE and The Daily PANCE Blueprint on Instagram Follow Smarty PANCE and The Daily PANCE Blueprint on Facebook Below you will find an interactive exam to complement today’s podcast. The Audio PANCE/PANRE and EOR PA Board Review Podcast I hope you enjoy this free audio component to the examination portion of this site. The full board review course includes over 2,000 interactive board review questions and is available to all members of Smarty PANCE . You can download and listen to past FREE episodes here , on iTunes , Spotify , Google Podcasts , Stitcher , and most podcasting apps. You can listen to the latest episode, take an interactive quiz, and download more resources below. Listen Carefully Then Take the Practice Exam If you can’t see the audio player, click here to listen to the full episode. Podcast Episode 98: Ten PANCE/PANRE and EOR Topic Blueprint Questions 1. An 81-year-old female presents to the ER with acute onset of low back pain. She complains of bowel dysfunction and loss of sensation over her inner thighs. Physical exam reveals decreased lower extremity reflexes. Which of the following is the best diagnostic test for the likely diagnosis? A. Lumbar radiographs B. MRI of the lumbosacral spine C. CSF fluid analysis D. Scoliosis studies E. None of the above Click here to see the answer The answer is B. MRI of the lumbosacral spine Cauda equina syndrome is a surgical emergency caused by severe stenosis in the lumbar spine (often due to acute disc herniation). Clinical manifestations include bowel/bladder dysfunction , decreased lower extremity reflexes , sciatica , saddle anesthesia (loss of sensation over the perineum, buttock, medial aspect of thighs), and decreased anal sphincter tone. Symptoms can develop acutely or chronically. An emergent MRI of the lumbosacral spine is the preferred diagnostic test. Surgery is the preferred treatment. Click here to view the Smarty PANCE lesson Smarty PANCE Content Blueprint Review: Covered under ⇒ PANCE Blueprint Musculoskeletal ⇒ Spinal Disorders ⇒ Cauda equina syndrome Also covered as part of the Emergency Medicine PAEA EOR topic list 2. Which of the following is the most common cause of Cushing syndrome? A. Iatrogenic B. Bronchogenic carcinoma C. Pituitary adenoma D. Adrenal adenoma E. Obesity Click here to see the answer The answer is A. Iatrogenic Cushing syndrome is a condition defined by too much cortisol. The most common cause of Cushing syndrome is exogenous steroid therapy (i.e., medical providers prescribing steroids). Other causes include a pituitary ACTH-secreting adenoma, adrenal tumors, and ectopic ACTH production from neoplasms. Classic clinical features include buffalo hump , moon-facies , purple striae , easy bruising , weight gain , depression , and weakness. Laboratory tests will be remarkable for elevated cortisol. Late night-salivary cortisol or 24-hour urinary free cortisol is typically the initial screening test. A low-dose dexamethasone test can be done. Referral to an endocrinologist is usually done at this point (or even prior). Click here to view the Smarty PANCE lesson Smarty PANCE Content Blueprint Review: Covered under ⇒ PANCE Blueprint Endocrinology ⇒ Adrenal Disorders ⇒ Cushing’s syndrome Also covered as part of the Internal Medicine EOR , Family Medicine EOR , and Emergency Medicine PAEA EOR topic list 3. A 73-year-old female presents with facial flushing, wheezing, and watery diarrhea for a few months. Her symptoms are worsened by certain foods. Which of the following is the best initial diagnostic study to order for the suspected diagnosis? A. 5-HIAA in urine B. Serum ACE levels C. Chest radiograph D. Upper endoscopy E. Bronchoscopy Click here to see the answer The answer is A. 5-HIAA in urine Carcinoid tumors are uncommon tumors that originate from neuroendocrine cells and secrete serotonin. The most common site is the appendix , but they can also be found in the lungs, kidney, etc. Carcinoid syndromes develop 10% of the time; clinical features include flushing , sweating , wheezing , and watery diarrhea . These symptoms are due to the release of serotonin and can be precipitated by foods high in tyramine or ethanol . The best initial diagnostic study is a 24-hour measurement of urinary excretion of 5-HIAA (Note: 5-HIAA is a degradation product from 5HT). Surgical resection is the definitive treatment of choice. Click here to view the Smarty PANCE lesson Smarty PANCE Content Blueprint Review: Covered under ⇒ PANCE Blueprint Pulmonary ⇒ Pulmonary Neoplasms ⇒ Carcinoid tumors Also covered as part of the Internal Medicine PAEA EOR topic list 4. Which of the following is the most common cause of Mallory-Weiss Syndrome? A. Iatrogenic B. Vomiting C. Abdominal trauma D. Hiatal hernia E. Corrosive ingestion Click here to see the answer The answer is B. Vomiting Mallory-Weiss Syndrome (MWS) is defined by a mucosal tear at or below the gastroesophageal junction. MWS is one of the most common causes of an upper GI bleed. It is most commonly due to forceful vomiting . It is classically associated with alcoholism ( binge drinking ), but really anything that can cause vomiting can lead to a tear. Other causes include iatrogenic, trauma, hiatal hernia, and corrosive ingestion. Diagnosis is made via upper endoscopy . Intervention is rarely necessary, but if needed, the patient can undergo surgery or embolization. Do not confuse this with Boerhaave syndrome, which is an actual perforation of the esophagus! Click here to view the Smarty PANCE lesson Smarty PANCE Content Blueprint Review: Covered under ⇒ PANCE Blueprint GI and Nutrition ⇒ Esophageal Disorders ⇒ Mallory Weiss tear Also covered as part of the Internal Medicine EOR and Emergency Medicine PAEA EOR topic list 5. A 51-year-old male presents to the clinic complaining of “band-like pressure” around his head. He has been more stressed lately. Which of the following is the most likely diagnosis? A. Cluster headache B. Migraine with aura C. Sinus headache D. Tension headache E. Medication overuse headache Click here to see the answer The answer is D. Tension headache A tension-type headache (TTH) is the most common headache. It usually presents with steady, aching, “ band-like” pain that circles the entire head. There may also be tightness in the posterior neck muscles. Precipitants include anxiety , depression , and stress . You should treat any underlying depression and/or anxiety. NSAIDs and acetaminophen are the first-line abortive treatment options for mild-moderate TTHs. For long-term prophylactic treatment, amitriptyline is preferred. Click here to view the Smarty PANCE lesson Smarty PANCE Content Blueprint Review: Covered under ⇒ PANCE Blueprint Neurology ⇒ Headaches ⇒ Tension headache Also covered as part of the Internal Medicine PAEA EOR topic list 6. Which of the following physical exam findings would you expect in a patient with mitral regurgitation? A. Holosystolic blowing murmur best heard at the apex with radiation to the axilla B. Diastolic blowing murmur at the left upper sternal border C. Systolic ejection crescendo-decrescendo murmur at upper right sternal border D. Mid-systolic harsh murmur best heard at the left upper sternal border E. None of the above Click here to see the answer The answer is A. Holosystolic blowing murmur best heard at the apex with radiation to the axilla Mitral regurgitation (MR) is characterized by a blowing, holosystolic murmur at the apex with radiation to the axilla . It can be acute or chronic. Acute causes include endocarditis and papillary muscle rupture; chronic causes include mitral valve prolapse (MVP), rheumatic fever, cardiomyopathy. The most common cause of MR is MVP . Remember, regurgitation murmurs will often have blowing sounds, while stenotic murmurs are harsh and rumbling. Diagnosis should be made with an echocardiogram . Treatment (repair vs. replace vs. medical therapy) depends on many factors. Click here to view the Smarty PANCE lesson Smarty PANCE Content Blueprint Review: Covered under ⇒ PANCE Blueprint Cardiology ⇒ Valvular Disorders ⇒ Mitral regurgitation Also covered as part of the Emergency Medicine EOR topic list 7. Which of the following is not an expected clinical manifestation of polyarteritis nodosa? A. Kidney failure B. Livedo reticularis C. pANCA positive D. Pulmonary fibrosis E. Hypertension Click here to see the answer The answer is D. Pulmonary fibrosis Polyarteritis nodosa is a systemic vasculitis of medium-sized vessels (specifically involving the nervous system and GI tract) that leads to thrombi and microaneurysms. It is associated with hepatitis B , HIV , and drug reactions . Clinical features include fever, hypertension, livedo reticularis, abdominal pain, and arthralgias. Remember patients may be positive for pANCA , but are ANCA negative. A definitive diagnosis is made via biopsy of involved tissue (or mesenteric angiography). What sets polyarteritis nodosa apart from other vasculitides is that it spares the lungs . Treatment includes high-dose steroids . The prognosis is not really good. Click here to view the Smarty PANCE lesson Smarty PANCE Content Blueprint Review: Covered under ⇒ PANCE Blueprint Musculoskeletal ⇒ Rheumatologic Disorders ⇒ Polymyalgia rheumatica Also covered as part of the Internal Medicine PAEA EOR topic list 8. An 8-year-old female presents to the clinic with her dad. She is complaining of left ear pain and pruritus for the past few days. The physical exam is remarkable for drainage and pain on tragal pressure. The tympanic membrane can’t be visualized. Which of the following is the best treatment option? A. Oral amoxicillin B. Neomycin/polymyxin B/hydrocortisone drops C. Oral cefepime D. Oral steroids E. Ciprofloxacin/dexamethasone drops Click here to see the answer The answer is E. Ciprofloxacin/dexamethasone drops The patient has otitis externa (aka swimmer’s ear), which is inflammation of the external auditory canal. The most common bacterial cause is Pseudomonas aeruginosa (~40%). Risk factors include water exposure, trauma, ear devices, and dermatologic conditions. Clinical features include ear pain , pruritus , auricular discharge , pain/tenderness when pressure is applied to the tragus , and hearing loss . Treatment is topical ear antibiotic/steroid drops. Medication options include neomycin/polymyxin B /hydrocortisone and ciprofloxacin/dexamethasone . **Remember that aminoglycosides are ototoxic and should be avoided when the tympanic membrane can’t be visualized!** Click here to view the Smarty PANCE lesson Smarty PANCE Content Blueprint Review: Covered under ⇒ PANCE Blueprint EENT ⇒ Ear Disorders ⇒ External ear ⇒ Otitis externa Also covered as part of the Pediatric EOR , Family Medicine EOR , and Emergency Medicine PAEA EOR topic list 9. Which of the following correctly depicts the typical clinical features of Parkinson’s disease? A. Pill-rolling tremor, bradykinesia, rigidity, masked facies B. Loss of declarative episodic memory, apraxia, olfactory dysfunction C. Visual hallucinations, REM sleep behavior disorder, gait issues D. Hyperorality, apathy, socially inappropriate behavior E. None of the above Click here to see the answer The answer is A. Pill-rolling tremor, bradykinesia, rigidity, masked facies Parkinson’s disease is a neurodegenerative disease. Its 3 cardinal features are tremor (“pill-rolling” at rest), bradykinesia , and rigidity . Tremor is the presenting feature in most patients. Other clinical features include hypomimia (masked facies), speech impairment , mood disorders (depression or anxiety), dysphagia , shuffling gait , stooped posture , sleep issues , cognitive dysfunction , and autonomic dysfunction . It is mainly a clinical diagnosis. When patients respond to dopaminergic drugs, this supports the diagnosis. The mainstay of treatment is levodopa or dopamine agonists (pramipexole, ropinirole). For mild disease, monoamine oxidase type B inhibitors or amantadine can be given. Click here to view the Smarty PANCE lesson Smarty PANCE Content Blueprint Review: Covered under ⇒ PANCE Blueprint Neurology ⇒ Movement Disorders ⇒ Parkinson’s disease Also covered as part of the Internal Medicine EOR and Family Medicine PAEA EOR topic list 10. A 72-year-old male presents with an abnormal change in bowel habits and fatigue. The physical exam is remarkable for pale conjunctiva and a palpable abdominal mass. His hemoglobin is 10 mg/dL. Which of the following is likely to be elevated? A. Alpha-fetoprotein B. CEA C. CA-125 D. CA 19-9 E. AAT1 Click here to see the answer The answer is B. CEA The patient has colorectal cancer (CRC), which is the third most common cause of cancer death in the U.S. The USPSTF suggests screening at age 45 for individuals with an average risk . Patients may present with alarming “red flag” symptoms, a bowel obstruction, or CRC may just be found via routine screening. The most common symptom is a change in bowel habits (~70%) . Other symptoms include rectal bleeding, iron deficiency anemia, and pain. Initial diagnostic tests include colonoscopy, CBC, fecal occult blood testing, and tumor markers like carcinoembryonic antigen (CEA). CEA is more often used for surveillance and not screening though (since its sensitivity for CRC is only 46%). Click here to view the Smarty PANCE lesson Smarty PANCE Content Blueprint Review: Covered under ⇒ PANCE Blueprint GI and Nutrition ⇒ Gastrointestinal System Neoplasms ⇒ Colon cancer Also covered as part of the Internal Medicine EOR , Family Medicine EOR , General Surgery PAEA EOR topic list Looking for all the podcast episodes? This FREE podcast series is limited to every other episode, you can download and enjoy the complete audio series by becoming a Smarty PANCE member . I will be releasing new episodes every few weeks. Smarty PANCE is now discounted, so sign up now before it’s too late! Additional resources and links from the show Download your Free interactive PANCE, PANRE, and EOR™ Blueprint Templates Follow Smarty PANCE and The Daily PANCE Blueprint on Instagram Follow Smarty PANCE and The Daily PANCE Blueprint on Facebook My list of recommended PANCE and PANRE review books Sign up for the FREE Daily PANCE and PANRE email series Join the Smarty PANCE NCCPA Content Blueprint Website Get your free 8-week PANRE Blueprint study schedule and the 8-week PANCE study schedule Get your free Trello PANCE study planner Get 20% of any Picmonic membership by using this link This Podcast is available on iOS and Android You can download and listen to past FREE episodes here , on iTunes , Spotify , Google Podcasts , Stitcher , and most podcasting apps. Download the Interactive Content Blueprint Checklist Follow this link to download your FREE copy of the PANCE/PANRE/EOR™ Content Blueprint Checklists Print it up and start crossing out the topics you understand, marking the ones you don’t, and making notes of key terms you should remember. The PDF version is interactive and linked directly to the individual lessons on Smarty PANCE. Smarty PANCE is not sponsored or endorsed by, or affiliated with, the NCCPA or the Physician Assistant Education Association (PAEA). All trademarks are the property of their respective owners.…
T
The Audio PANCE and PANRE Physician Assistant Board Review Podcast


Welcome to episode 97 of the Audio PANCE and PANRE PA board review podcast. Today is a bonus episode rounding out this fabulous five-part podcast series with Joe Gilboy PA-C, all about cardiac murmurs. In this week’s episode, we continue our discussion of cardiac murmurs with ten PANCE and PANRE murmur questions. We’ll cover the ins and outs of all the NCCPA content blueprint valvular disorders and learn how to identify and differentiate them from one another. If you haven’t already, make sure to listen to our previous podcast episodes where we covered tricuspid stenosis , aortic valve murmurs , mitral valve murmurs , pulmonic valve murmurs , and HOCM and MVP . Podcast Episode 97: Murmurs Made Incredibly Easy – 10 PANCE and PANRE Murmur Questions Below is a transcription of this podcast episode edited for clarity. You can download and listen to past FREE episodes here , on iTunes , Spotify , Google Podcasts , Stitcher , Amazon Music , and most podcasting apps. You can listen to this latest episode and access even more resources below. [00:00:00] Welcome back, everybody. This is Joe Gilboy. I work with Stephen Pasquini over there at Smarty PANCE. And today is the most dreaded podcast of all because you know what I’m going to do. That’s right. I’m going to ask cardiac murmur questions. And I know it’s the most dreaded thing in the world. Let’s do this together because what I want to try to do here is make sense of it all. So, let’s just kind of recap some basic rules before we start going down this thing. You know, the previous lectures, what have I been barking about? What’s Joe been saying? First – inspiration, right? Expiration left. [00:00:40] Inspiration, right? Expiration left. So, with inspiration, the right-sided murmurs sound louder. So that’s the tricuspid and the pulmonic valve regardless of if it’s stenosis or regurgitation. Expiration – left. What am I saying? Everything on the left side sounds louder with expiration, whether it’s aortic or mitral – your call. [00:01:01] It can be stenotic or regurgitation, it doesn’t matter, but it’ll sound louder. What is squatting? Squatting is a party. So, what are you doing? You’re bringing it all the blood flow back home. And so, if I bring all the blood flow back home, just from a laminar flow physics point of view, I bring more blood flow back home to the heart, more blood flow over a valve specifically with a diseased disease valve, it sounds louder. [00:01:29] So, right off the bat, squatting will do what? It’s going to make all my murmurs sound louder. That’s the whole point to decrease venous return. In other words, take blood flow away from the heart. What do I do? Well, those are going to be Valsalva and standing. So, what am I doing when I do Valsalva and standing? [00:01:56] Taking blood flow away from the heart. So, what’s going to happen to all my murmurs when I stand and perform Valsalva? It takes blood flow away from the heart. Exactly. And then hand grip. What did handgrip do? So, in school, what’d you learn about hand grip? Well, it increases afterload, right? So, what they’re really saying to you is this, and this is how I want you to visualize it. [00:02:18] When I do hand grip, what I’m really doing is this. You’re right. I’m increasing the afterload. But you must stop and think this one out for a second, folks. What’s really in the heart? Which valve must fight afterload? [00:02:39] Say you increase the peripheral vascular resistance for whatever reason. So, you increase the afterload, who fights it? And your answer will be… oh, wait for a second, Joe, that’s the aortic valve in the left ventricle. Exactly. So, when I perform handgrip, who am I really challenging? Oh, I’m challenging the aortic valve and everything behind it. [00:03:00] Exactly. So that’s the point I need inside your head? Then remember our last podcast was about our low-volume lovers. Who were our low-volume lovers? Who were the special needs murmurs that really like low volume? They’re like, hey, I like low volume, not high volume. I like low volume. Who was that? [00:03:19] That was HOCM and mitral valve prolapse. What did they both like? They both like low volume. So, they’re the opposite of all my regular murmurs. They go in the opposite direction. All right, everybody got that? I am not going to give you a list of answers. So, what I’m going to do is I want you to listen to what I’m going to say. I’m going to say the test question twice. [00:03:47] And then I want you to pause the podcast. Just put it on pause and think about your answer before I explain it. All right. Is everybody with me? Because that’s the best thing to do. All right, here we go. Murmur Question #1: [00:04:11] You have a 76-year-old gentleman that presents to your emergency room and comes in complaining of shortness of breath and chest pain on physical exam. You notice an upper sternal murmur that sounds louder with squatting and goes away with hand grip —one more time. Upper sternal border, sounds louder with squatting, and goes away with hand grip. Who am I? [00:04:45] So upper sternal border. This will be my aortic and pulmonic area. Well, it goes away with handgrip. So, who’s that going to be? So, what happens with hand grip? I increase my afterload. Which valve fights the afterload? The aortic valve . It’s not pulmonic because the pulmonic valve is not fighting the afterload; it is fighting the lung. So, this has to be aortic stenosis, which makes sense for the syncope and the shortness of breath. So aortic stenosis is going to do what? Well, squatting, that’s a party. Of course, it will sound louder. We could actually throw expiration in here as well, couldn’t we? We sure could, but it’s the handgrip. That differentiates it, doesn’t it? Because the handgrip made the murmur go away. And what did the handgrip do? I keep telling you I want you to view handgrip like sitting on your aortic valve, and you can barely open up squeak, squeak, squeak, squeak. [00:05:47] You can barely open up. Now imagine sitting on that valve. Can it open up now? And you’re like, no, it goes away. Exactly. And that’s the point I’m trying to make. The answer is aortic stenosis. https://smartypance.com/podcast-aortic-stenosis-regurgitation/ Murmur Question # 2 [00:06:21] I have a 45-year-old gentleman who comes in with shortness of breath and chest pain. The murmur sounds louder with inspiration. It’s located at the upper sternal border . And I am also noticing that the patient has right ventricular hypertrophy and right bundle branch block on EKG. Who am I? [00:06:44] Upper sternal border. Well, who are we talking about? The aortic valve or the pulmonic valve. Inspiration → right and expiration → left. So, this murmur sounds louder with inspiration, so I know that this is on the right side. Then it’s got to be the pulmonic valve . And now they’re telling me I’ve got right ventricular hypertrophy and right bundle branch block. Then blood must be backing up into the right ventricle. [00:07:10] In other words, something must be backing up, which means the valve must be stenotic . Oh, I see your point. It’s stenotic, and the right ventricle is going to hypertrophy. Remember if you take right ventricular hypertrophy and let it go unopposed. You stretch, and you stretch, and you stretch, and you stretch, and you dilate, and you stretch, and you dilate and stretch that wall. [00:07:28] What are you going to get? The wires are going to get busted. That’s right bundle branch block. For those of you thinking outside the box, and you’re telling me, Joe, if I dilate my left ventricle and just stretch it and stretch and a bust, the wires, I get left bundle branch? Exactly. No, that’s exactly what you’re going to get. So the answer to the question is pulmonic stenosis. https://smartypance.com/podcast-episode-93-murmurs-made-incredibly-easy-part-3-of-5-pulmonary-valve-stenosis-and-regurgitation/ Murmur Question # 3 [00:07:45] You have a young 32-year-old female who recently immigrated to the United States from Indonesia and is currently working and has now been complaining of increasing shortness of breath at work. On exam, you hear a murmur at the left sternal border . It sounds louder with expiration and appears to be diastolic in nature . Who am I? [00:08:38] So again, left sternal border . So that’s just about everybody but the pulmonic valve . The murmur sounds louder with expiration, so you know it will be on the left (louder with inspiration → right and louder with expiration → left). So, based on these parameters, I know I’m dealing with either the aortic valve or the mitral valve , and I am a diastolic murmur . So now you have to stop and think. So, this seems to be in the neighborhood of the mitral area, and it’s a diastolic murmur. [00:09:00] So it can’t be mitral regurgitation. It has mitral stenosis, and that’s the correct answer. And then, of course, as I always say, you’ll see the test questions, do this all the time. They’ll talk about people who immigrated to the United States. And what they’re really saying to you is this. So, this lady most likely has some form of rheumatic fever. [00:09:16] Maybe she had strep when she was a younger child and just wasn’t diagnosed. So, what they’re really sharing with you is this. As soon as I see the word immigrant, what they are really saying is this: Listen, this person is at high risk for TB. All right. That’s the one thing that we kind of worry about. [00:09:29] The second thing is that their care is not as structured as ours. In other words, they slipped through the cracks. Could this patient have had a previous strep throat and had rheumatic fever, which led her to the mitral stenosis? Of course, she could. [00:09:44] And that was the whole point to it. But again, it is expiration → left, left sternal border. Just about everybody is at the LSB, so that doesn’t really help. But it was on the left side. So, I know this is either going to be aortic or mitral. And then you told me it was diastolic, so this will fit mitral stenosis. [00:10:06] So, just trying to get you to look at these murmurs differently. That’s all just more from a laminar flow physics point of view. Murmur Question #4 [00:10:15] Next question. I have a 67-year-old female who comes in with an upper sternal border murmur. This murmur increases in intensity when she is squatting , and with handgrip , it decreases intensity with Valsalva and standing . [00:10:41] One more time. This murmur increases in intensity with squatting and handgrip and decreases in intensity with Valsalva and standing. Who am I? [00:10:54] So let’s think about this upper sternal border . So, what are we talking about? Pulmonic or aortic . They didn’t tell me what side right or left did they? However, they told me that this murmur increases in intensity with squatting – that’s everybody. So that’s not helping me, but they said it increases in intensity with hand grip. Stop. No stop right here. [00:11:06] Handgrip, who’s that challenging? Pulmonic or aortic? Oh, that’s aortic, Joe. I see your point. That makes so much sense. It’s not pulmonic. So, this is going to be either aortic stenosis or aortic regurgitation. [00:11:28] Well, let me think about this. You’re telling me that this murmur sounds louder with handgrip, and wait a second, it’s not shutting it off. It’s making it worse. This means that the valve is wide open. Oh, then this must be aortic regurgitation, and that’s the correct answer. And of course, you know, my low volume maneuvers – Valsalva and standing, of course, will make it go away. The answer is aortic regurgitation. [00:11:51] So does everybody see this? I’m just trying to get you ready for the PANCE. But does every see how we’re looking at this now? This is what I keep hammering away at, approach this not from a memorization point of view but a laminar flow physics point. Stop memorizing! [00:12:07] Start trying to make sense of this because once you make sense of this, it will flow so much easier for you. Ignore those things you memorized in PA school, those little four squares, the graphs, etc.. I’ve seen it all in my 30 years of teaching. You can’t teach me one more trick. And I laugh when I’m sitting here in my studio saying this to you because I know students better than students know students. And I get it, but I’m just trying to teach you what will help you the most on the boards. Murmur Question #5 [00:12:36] Next question. I have a patient who is a 72-year-old gentleman with hypertension, diabetes, and coronary artery disease, who also smokes. He comes in complaining of increasing shortness of breath and bilateral leg swelling. He’s got a murmur noted on physical exam in the upper sternal border . The murmur sounds louder with inspiration . Squatting makes it sound louder. Standing and Valsalva make it go away. And this murmur appears to be diastolic in nature. Who am I? [00:13:31] Upper sternal border (pulmonic and aortic area) and louder with inspiration. So, I’m on the right (inspiration → right and expiration → left). This has got to be pulmonic. So, it’s going to be pulmonic stenosis or pulmonic regurgitation. He’s a smoker. So, he’s got some pulmonary hypertension. I get it. And this is a diastolic murmur. So, what pulmonary valve disorder is diastolic? [00:13:51] You’re going to go, oh, this has got to be pulmonic regurgitation. Exactly and wait a second. Oh, that makes so much sense. All that pulmonary hypertension, with COPD and stuff like that. Oh, that makes so much sense. And then he’s going to have to fight it. And, of course, everything is going to back up into the right ventricle and then go to his legs. The answer is pulmonic regurgitation. https://smartypance.com/podcast-episode-93-murmurs-made-incredibly-easy-part-3-of-5-pulmonary-valve-stenosis-and-regurgitation/ Murmur Question #6 [00:14:27] Next question. I have a 37-year-old female with a previous history of mitral valve prolapse. [00:14:33] However, recently, she’s been coming in with increasing shortness of breath and was diagnosed with atrial fib . Physical exam shows a murmur that increases with squatting and sounds louder with handgrip . And this murmur increases in intensity in the left lateral decubitus position . This murmur also decreases in intensity with Valsalva and standing . Who am I? [00:15:10] One more time. The murmur increases intensity with squatting. Also, hand grip, also left lateral decubitus position, and decreases in intensity with Valsalva and standing. Who am I? [00:15:31] All right. So, let’s think about this, she had a previous history of mitral valve prolapse. I got it. But now she’s got increasing shortness of breath and a fib – oh something went wrong. [00:15:41] Do people with mitral valve prolapse get a fib? No, not at all. And so now this murmur is going to increase in intensity with squatting so that’s not helping me, but the hand grip, it increases with hand grip. So, wait a second. Let me think about this. So, there I am handgrip sitting on the aortic valve, which means the left ventricle has got to fill up with more blood and the left ventricle is going to contract with more blood. [00:16:07] And then that mitral valve that should stay closed is actually blowing right on through. And if I put them in that left lateral decubitus position, that’s bringing the mitral valve to the surface. So let me think about this for a second, Joe. Hold on here. So, you’re telling me that the left lateral decubitus position brings the mitral valve to the right? [00:16:31] So whether it be mitral stenosis or mitral regurgitation, that’s the maneuver, it’ll bring it closest to the chest. So, you hear it better. And we could argue expiration in this question, but this clue wasn’t given to me here. And does everybody see the most common cause for mitral regurgitation is actually mitral valve prolapse that goes untreated? [00:16:48] And that’s how we get it. And then, of course, they start irritating the left atria, which is how they get the AFib. But now, does everybody understand how the handgrip made it worse? Again let’s think about it. Your hand gripping. What are you doing? Increasing the pressure behind the aortic valve. [00:17:05] What’s the left ventricle going to do? Remember the Frank-Starling thing? What did Frank say? When you increase afterload, you’re going to increase preload. What’s the left ventricle going to do? Fill up with more blood. And there’s that mitral regurgitation valve – come and go as you please. Now that left ventricle goes to contract, what’s it going to do? [00:17:20] Oh, it will push more blood flow against my regurgitant mitral valve. Exactly. And that’s the correct answer – mitral regurgitation. https://smartypance.com/podcast-92-mitral-stenosis-regurgitation/ Murmur Question #7 [00:17:31] Next question. I have this 67-year-old gentleman who comes in complaining of right-sided heart failure . He’s also got some right atrial enlargement, and he also has bilateral lower extremity edema. During the exam you note a mid-diastolic murmur at the left lower sternal border that increases in intensity with inspiration , sounds louder with squatting , and goes away with standing and Valsalva . Who am I? [00:18:09] Again, mid-diastolic murmur, left lower sternal border, increasing with intensity with squatting and inspiration, goes away with Valsalva and standing. Who am I? [00:18:30] Let’s think about it. So, what did we get in the question? Well, we got the word inspiration. So, what do we know? Oh, that’s on the right. And then we said left sternal border. [00:18:39] So who’s that? Everybody. But it’s not the pulmonic valve or the aortic valve. So, who are we talking about if it’s on the right and not the pulmonic valve? The tricuspid valve. And then you told me that it was mid-diastolic. So, wait a second. The tricuspid valve, what’s the diastolic murmur associated with the tricuspid valve? Is this regurgitation or stenosis? Oh, this is stenosis. The answer is tricuspid stenosis. https://smartypance.com/podcast-tricuspid-stenosis/ [00:18:59] What are your low volume maneuvers? Standing and Valsalva. Usually, with these maneuvers, all of the murmurs will go away. The low volume lovers are murmurs that sound louder with low volume. That’s HOCM and mitral valve prolapse. Murmur Question #8 [00:19:13] Next question. I have a murmur that is holosystolic in nature that is located on the left midsternal border and sounds louder with squatting and inspiration. This murmur also goes away with Valsalva and standing . Who am I? [00:19:40] One more time. I am a holosystolic blowing murmur. I’m located at the left midsternal border. I sound louder with inspiration and squatting, and I go away with standing and Valsalva. Who am I? So holosystolic means, it’s probably most likely some type of regurgitation and now you’re telling me that it’s on the left midsternal border that’s everybody. But the key piece of information you gave me is inspiration, which means this is right. Which means it’s going to be the tricuspid valve . Now you’re telling me it’s holosystolic, so that can’t be tricuspid stenosis because that is diastolic. So, this is going to be tricuspid regurgitation . The answer is tricuspid regurgitation. [00:20:29] Then there’s that thing about the Carvallo’s sign. The increased murmur intensity with inspiration. And then sometimes we see that pulsatile liver. Because everything’s backing up on the right side, so, all that blood flow is going to the liver, and it’s causing it to kind of shake in a way. Sometimes we call it pulsatile. So again, is everybody starting to understand this? Inspiration → right and expiration → left Squatting is not going to help you much And now you have to ask, where’s the location of this murmur? And remember, when they say left, lower sternal border, they’re saying, hey, this is everybody but the pulmonic and the aortic valve, which are located in the upper sternal border. They’re not going to be nice to you and say, oh, it’s right or left because that gives you too much information. So again, we’re just going to pull back, and we’re going to use Joe’s basic rules. Murmur Question #9 [00:21:17] Next question. You have a healthy 24-year-old female sitting on your exam table who comes in with mild lightheaded and dizziness with very little exertion. On physical exam, you notice that she has a murmur when she performs standing and Valsalva ; however, this murmur also goes away with squatting. The rest of her physical exams is essentially unremarkable. [00:21:57] And when you have her perform handgrip, the murmur disappears. Who am I? [00:22:05] Remember this murmur sounded louder with standing and Valsalva. It went away with squatting, and it went away with handgrip. So, wait a second. You’re going; it went away with squatting? Isn’t squatting a party, Joe, doesn’t that make everybody sound louder? [00:22:22] It does. Wait a second. I’m back to my special needs murmurs, the ones that like low volume. So, this is either HOCM or mitral valve prolapse. Exactly! So, let me take this one step further. So, you’re telling me if I squat, what do I do again? I’m pushing all the blood flow back home. So, you’re taking the left ventricle and filling it with blood. [00:22:47] So if I have HOCM, then all that blood will do what? It’s going to push the walls apart. But wait a second, the murmur is going to go away. That makes sense. Wait a second. It went away with hand grip as well? So let me think about this again. Hold on. Handgrip. What’d you do? Oh, that’s right, I increased my afterload. Joe said basically you’re just sitting here on the aorta. What did Frank say? You increase afterload; you increase preload. So, what’s the left ventricle going to do to fight that afterload? Fill it with more blood. And what are the walls going to do? Push apart. That’s HOCM. You see what you have to remember is that HOCM is not a valve problem. Say to yourself, HOCM is not a valve problem. And if you didn’t get it the first two times, let’s say it a third time, HOCM is not a valve problem. It’s a wall problem. The left ventricle walls are hitting. And so now, does everybody understand, as they’re at rest, what do you want to do to make the murmur louder? You know you want the walls to hit. Just lightly tap, and you want them to hit. So, what’s in between the walls? Blood. So, what makes it sound louder? Oh, standing and Valsalva. Put blood in there, and what will we do with the walls? Push them apart. So how do you put blood in the left ventricle? Oh, I could squat, and I could do hand grip. Exactly. And that’s the whole point I’m trying to make about HOCM. The answer is HOCM. https://smartypance.com/podcast-episode-96-mvp-and-hocm/ Murmur Question #10 [00:24:10] Last question. You have a young, healthy 31-year-old female who comes into your clinic, and she’s been complaining of progressive, mild shortness of breath and palpitations. On physical exam, you notice this murmur increases in intensity with Valsalva and standing . You also notice that this murmur goes away with squatting and handgrip . You also note that this murmur sounds louder in mid-systole. Who am I? [00:25:04] So, let’s think about the information we were given. We were given information that this murmur sounds louder with Valsalva and standing (our low volume maneuvers). [00:25:13] We also know that this murmur goes away with squatting and handgrip (our high volume maneuvers, and then they give us this mid-systole. So who am I? [00:25:25] You’re telling me that squatting makes it go away, which means it’s quite the opposite of what we would expect, which means it’s got to be a low volume lover, which is correct because it sounds louder with Valsalva and standing. Also, this murmur is a mid-systolic murmur. [00:25:43] So wait a second, when I look at my “low volume lover” murmurs, you have to remember there’s really only one low volume murmur. That’s mitral valve prolapse because you remember HOCM has nothing to do with the valves. It has everything to do with the walls. So, you’re like, oh, they’re telling me that this thing sounds louder mid-systole. [00:26:05] One more time, when is the left ventricular pressure highest? Mid systole. Then there is that redundant mitral valve that has prolapsed, and the valve can’t hold. And then what does it do? The blood blows right on through. And that’s the correct answer – this is mitral valve prolapse . https://smartypance.com/podcast-episode-96-mvp-and-hocm/ [00:26:24] All right, listen, I need you to replay this podcast until you get tired of me talking, OK? [00:26:32] It’s like, oh my gosh, someone put Joe out of his misery, right? Tell him to stop talking to me. And I want you to keep replaying this and replaying this. But most importantly, stop memorizing! [00:26:46] So let’s go over it again just as a recap. Upper sternal borders, who are they talking about? Aortic and pulmonic. Where’s everybody else? The left lower sternal border. So left lower sternal border is not helping you out much. But if I say upper sternal border, I’m either pulmonic or aortic. If it’s on the right upper sternal border, that’s aortic. If it’s on the left upper sternal border, that’s pulmonic. Louder with inspiration → right and louder with expiration → left. What squatting? Party maneuver, who parties? Everybody, everyone’s going to sound louder. Who are your low-volume maneuvers? That’s standing and Valsalva. So, you have less blood flow in the heart, and most murmurs sound quieter. Who are your special needs murmurs? What were the two murmurs or two areas that like low volume? They like less blood in the ventricles. That is mitral valve prolapse, which is a valve problem, and HOCM, which is really a wall problem. Then inspiration → right and expiration → left. How do you take the mitral valve and put it close to the chest? The left lateral decubitus position. That’s what I do. So, the left lateral decubitus position is best, whether it be mitral stenosis or mitral regurgitation. And one more time. What’s hand grip? Handgrip is me sitting on the aortic valve. If I have aortic stenosis, I can barely open up, and you sit on the aortic valve, and is it going to be able to open up? Heck no! But if I have aortic regurgitation and now all this blood flow is going back against the aortic valve. Is there anything to stop it? No. And remember if I have a normal aortic valve and perform hand grip, what will I do? Remember what Frank Starling’s curve said, when I increase afterload, I increase preload, so what’s the left ventricle going to do? It’s going to fill it with more blood. So if I have HOCM, what are you doing to the walls? Well, I’m filling it with more blood, but what am I doing with the walls? Pushing them apart. [00:28:32] Hopefully, I’ve demystified the heart murmur issues for you, and I hope this makes more sense. Like I said, just replay this podcast episde. Remember what I’ve been trying to teach about inspiration and expiration and squatting and handgrip and all these maneuvers we can do to make the murmur sound louder and disappear. And if you go with that approach, I promise you, no matter how the PANCE comes at you with a heart murmur question – You’re going to nail it. [00:28:57] If you go at it from the memorization point of view, you will be in a boatload of trouble. Because all the buzzwords are gone and all those kinds of stuff. So listen, it’s been an honor. And it’s always a pleasure to speak with you out there in podcast land. And so right now, I got pixie. [00:29:13] My rescue dog whose lying on my bed, sitting here, and she’s giving me that look like she wants to go outside for a walk. So it’s time to take my rescue dog out for a walk. All the best, and please, as you’re getting ready for the PANCE, like I said, in the previous podcast, make sure you take care of yourself, take care of your health and take care of your brain. [00:29:32] All the best, Joe. Resources and links from the show Join our brand-new Smarty PANCE Member’s Community View all the episodes in this series Cardiac Murmurs Made Incredibly Easy View the NCCPA content blueprint valvular disorders Download your Free interactive PANCE, PANRE, and EOR Blueprint Templates Follow Smarty PANCE and The Daily PANCE Blueprint on Instagram and Facebook Check out my list of recommended PANCE and PANRE review books Sign up for the FREE Daily PANCE and PANRE email series Join the Smarty PANCE NCCPA Content Blueprint Website Get your free 8-week PANRE Blueprint study schedule and the 8-week PANCE study schedule Get your free Trello PANCE study planner Get 20% off any Picmonic membership by using this link , or if you are a Smarty PANCE member, get Picmonic for Smarty PANCE . This podcast is available on every device. You can download and listen to past FREE episodes here , on iTunes , Spotify , Google Podcasts , Stitcher , Amazon Music , and all podcasting apps. Download the Interactive Content Blueprint Checklist Follow this link to download your FREE copy of the PANCE/PANRE/EOR Content Blueprint Checklists . Print it up and start crossing out the topics you understand, marking the ones you don’t, and making notes of key terms you should remember. The PDF version is interactive and linked directly to the individual lessons on Smarty PANCE.…
T
The Audio PANCE and PANRE Physician Assistant Board Review Podcast


Welcome to episode 96 of the Audio PANCE and PANRE PA board review podcast. Today is part five of this fabulous five-part series with Joe Gilboy PA-C, all about cardiac murmurs. In this week’s episode of the Audio PANCE and PANRE podcast , we continue our discussion of cardiac murmurs with a focus on Mitral Valve Prolapse (MPV) and Hypertrophic Obstructive Cardiomyopathy (HOCM). We’ll cover the ins and outs of these two NCCPA content blueprint murmurs and learn how to identify and differentiate them from other types of murmurs. If you haven’t already, make sure to listen to our previous podcast episodes where we covered tricuspid stenosis , aortic valve murmurs , mitral valve murmurs , and pulmonic valve murmurs. HOCM and MVP (a brief introduction) Hypertrophic Obstructive Cardiomyopathy (HOCM) is a cardiac abnormality that leads to the muscle in the wall of the heart growing and thickening to the point that it blocks blood flow exiting the heart. The condition can be mild or severe, and it can lead to a variety of symptoms, including shortness of breath, chest pain, and irregular heartbeat. Complications may include heart failure, an irregular heartbeat, and sudden cardiac death. HOCM is a hereditary condition, and it is usually diagnosed in adulthood. There is no cure for HOCM, but treatments are available to manage the symptoms and help reduce the risk of complications. With proper care, people with HOCM can live long and healthy lives. *Hypertrophic cardiomyopathy is covered under the PANCE cardiology content blueprint -> cardiomyopathy -> hypertrophic cardiomyopathy HOCM is also covered as part of the PAEA EOR pediatric rotation -> cardiovascular topic list -> hypertrophic cardiomyopathy Mitral Valve Prolapse (MPV) is a condition in which the leaflets of the mitral valve bulge or prolapse back into the left atrium during systole. This may cause blood to flow backward into the left atrium, leading to a heart murmur. In some cases, MPV may also cause symptoms such as fatigue, dizziness, chest pain, and shortness of breath. While MPV is usually benign, it can occasionally lead to serious complications such as heart failure or stroke. Treatment for MPV typically involves lifestyle modification and management of symptoms. In severe cases, surgery may be necessary to repair or replace the mitral valve. *Miral valve prolapse is covered under the PANCE cardiology content blueprint -> valvular disorders -> mitral valve prolapse Podcast Episode 96: Murmurs Made Incredibly Easy (Part 5 of 5) – MVP and HOCM Below is a transcription of this podcast episode edited for clarity. You can download and listen to past FREE episodes here , on iTunes , Spotify , Google Podcasts , Stitcher , Amazon Music , and most podcasting apps. You can listen to this latest episode and access even more resources below. Welcome back, everybody out there in the podcast world. This is Joe Gilboy, and I work with Stephen Pasquini at Smarty PANCE. Today is part five of our five-part series covering heart murmurs – one of the most dreaded subjects in PA land. Today we are going to cover what I call the low-volume lovers. And who are the murmurs that like low blood volumes? In other words’ low blood volumes make these murmurs sound louder? That is Hypertrophic Obstructive Cardiomyopathy (HOCM) and mitral valve prolapse. Hypertrophic Obstructive Cardiomyopathy (HOCM) So, what is happening with HOCM? Let us go back and view this from a pathophysiology point of view. So what do you have with HOCM? I have this young adult, and he is going to start exercising. So what is the left ventricle going to do on a typical day during exercise? You will stress out the left ventricle, which can lead to hypertrophy. I want everybody to look in the space you are in right now. Maybe you are in a room. Perhaps you are in a car. I want you to look at the volume of this room or car and look at the wall. Now I want you to imagine that you start working out. You see, the wall will get thicker. Now, what happens when those walls are thicker, my muscles get thicker. Thus, what is going to happen next? My stroke volume will go up, my cardiac output will increase, and my resting heart rate will decrease. And that is a nice normal day. That is the way it is supposed to work. This is why aerobic exercise is so good. That’s what your Apple Watch or Fitbit is telling you. They are going, “Hey, we hit the target zone. You’re stressing out your left ventricle.” Your left ventricle will hypertrophy, which is a normal response to exercise. Your wall will get thicker, but the volume of the room you are sitting in stays the same. Let this marinate for a second. I stress out my left ventricle, and it’s going to hypertrophy. The wall is going to get thicker, with more healthy tissue. My stroke volume will go up, my cardiac output will go up, my resting heart rate will go down, and my exercise tolerance will improve. That’s a nice normal day. Hi, I have HOCM by genetics, no fault of my own. I was just born with this genetically. I will stress out my left ventricle, except instead of hypertrophying out, with HOCM, I am going to hypertrophy in. Okay, please stop and think about what I am saying. Now. See the room you are sitting in, that wall. It is coming in. Not out, but in. Look at the volume. You have in your room now. What have you done? You have decreased the volume (the size) of your room. Now, since I have less volume, what is the last thing you want to take away from me? Volume? So what drug is contraindicated in HOCM? Diuretics! The last thing I want to do is have these walls hit with low volume and a high pulse. That’s when bad things can happen. This is the VTACH and the VFIB that you see these young athletes with HOCM die from. So, these walls get thick and come in and make my room smaller. So, the last thing I want to do is take volume away, and the last thing I want to do is give them a diuretic. What is a lifestyle thing I can tell my patients with HCOM to do? Drink more water! This will increase the volume, and the walls to be pushed apart. Does everybody see that? Okay, my patient with HCOM is sitting at the edge of the bed at rest. He is not exercising, and he does not have an elevated pulse and low volume – that’s that nightmare scenario we were talking about where a patient with HOCM is exercising, and they have low volume. They are sweating, and their pulse is high. Now the walls hit, and this is when the bad things happen. So, I have this patient sitting at the edge of the bed in an exam room at rest. Remember, HOCM is not a valvular issue . No valve is not functioning correctly here. These are walls that are enlarged (hypertrophied). Now I want to hear the walls hit each other while my patient with HOCM is at rest. That’s all I want. I want the walls to hit. What is in the way between those two walls? Blood! If I want the walls to hit, I have to get the blood out. I can do this with my low-volume maneuvers. What are my low-volume maneuvers? Standing and Valsalva. When I have HOCM, if I perform standing and Valsalva (the low volume maneuvers), I can get the blood out of the way so walls can get closer and hit. So, what makes the murmur of HOCM sound louder? Standing and Valsalva. How do I push the walls of the ventricle apart? I can add more blood. How do I add more blood? If I squat! That is right; squatting is a party. If I go in there and squat and all this blood comes running back home, what will I do with the walls of the left ventricle? I am going to push them apart. So, this is the opposite of all those murmurs I learned about! This is why I call HOCM and MVP the low volume lovers – because they like low volume. Handgrip and HOCM Let us take it one step further. Let’s do that one maneuver everybody hates out there in the podcast world: Handgrip. Okay, think about it. Stop memorizing. Let’s make sense of this. So, we perform handgrip. What am I doing? I am sitting on the aortic valve. You sat on the aortic valve and added more pressure to the valve by doing this. You have added more afterload. What is the left ventricle going to do? Well, since I have more afterload, I need more preload. So, my ventricles are going to have to fill up more. With the increased volume, the walls will move apart from each other. So, you are telling me that when I squat and put all that blood into the left ventricle, it will push the walls apart? Furthermore, when I do handgrip, it will also cause those walls to move apart because I must increase my preload and increase the volume. Exactly! So, when I perform handgrip and have HOCM, the murmur goes away. HOCM summary HOCM It is not a valvular issue. To make the murmur of HOCM louder, you want the ventricular walls to hit, which requires lowering the volume of blood between the ventricles. In contrast, for my patient’s health, the last thing I want to have is low volume, so I do not want my patient to sweat. I do not want to give him a diuretic and lower the volume even more. And what is his nightmare situation for patients with HOCM? Low volume and high pulse! What can we give patients with HOCM that will keep their pulse low? We can give them a beta-blocker. Remember, those walls are disorganized, and the tissue is stiff. So, I want to relax the tissue. What is an excellent smooth most relaxer? Calcium channel blockers! And that is exactly how we treat HOCM with beta-blockade to lower the pulse and calcium channel blockers to relax the stiff tissue. This is everything you need to know about HOCM: It is not a valvular issue. To make HOCM louder, you want the volume out so the walls can hit. What are the low-volume maneuvers? Standing and Valsalva. How do you put volume (aka blood) in the room? Standing and handgrip? Exactly. That is precisely how this works. Mitral Valve Prolapse The next murmur that likes low volume is the murmur of mitral valve prolapse. In comparison to HOCM, this one takes a bit more thought. To demonstrate mitral valve prolapse put your two hands together like you are making the letter A. Okay, see how your fingertips are touching each other. This is how the mitral valve usually closes. Now point your thumbs in the middle and view your thumbs as the chordae tendineae. They are pulling your fingers down. That is how it works on an average day. To demonstrate mitral valve prolapse, take your right hand and place it where your metacarpals are – again, kind of like the letter A. Do you see how your fingers are over the top of the other ones? They do not match up perfectly. This is what we sometimes call a redundant valve. In other words, there is extra tissue, and things are not lining up. When the left atrium contracts on a normal day, the mitral valve comes down, and everything works as expected. However, we will run into trouble with MVP when the left ventricular pressure is highest. And when is the left ventricular pressure at its highest? Oh, that is right, I memorized that during PA school. It is highest during mid systole. That’s why we get a mid-systolic click – because the valve cannot hold the pressure below it during mid-systole. That is because the connection is not very good. It is loose. So, the valve pops open during mid-systole, and we call that a mid-systolic click. How can I crank that pressure up in the left ventricle? Oh, I could exercise. Exactly! This is why we often hear about people with mitral valve prolapse having palpitations with exercise. Okay. I want the murmur of MVP to sound louder. Think about this. Let us go back to where my hands were in the example above. Remember how my fingers were coming together to form the letter A? Except when I have mitral valve prolapse, my right hand is touching my metacarpals, and I have these extra fingers on top of each other, creating a redundant valve. So, I want this valve to collapse in on itself. I want it to fall in on itself. What is down in the left ventricle? Blood. Now we need to get the blood out of there so the valve can fall down on itself. We will need to call in our low-volume maneuvers to do this. What are our low-volume maneuvers? Standing and Valsalva! Standing and Valsalva are low volume maneuvers and will make the murmur associated with mitral valve prolapse sound louder. Now, if I put blood back in the ventricle, can that redundant valve fall down on itself? No, it cannot. So, squatting is a high-volume maneuver that puts blood back into the left ventricle, and it is not going to make the murmur of MVP sound louder because the valve is not going to fall in on itself. However, if I take the blood away, there is nothing down there in the left ventricle, and the valve will fall back down. So mitral valve prolapse is also a murmur that gets louder with low volume maneuvers. Recap What are your low-volume maneuvers? Standing and Valsalva! With low volumes and MVP, the valve will fall back down on itself. With high volumes and MVP, what am I going to hear? I am going to hear a mid-systolic click. Who is my patient going to be? MVP is more common in females, so we will likely be presented with a young female with palpitations during exercise. What are you going to write off as a young female with palpitations? Anxiety. These patients are often misdiagnosed with anxiety. What will the patient say to you that will be the clue that this is not anxiety? The patient may talk about the palpitations with exercise. Remember, with exercise, the left ventricle is full of blood, and there is a lot of pressure in the left ventricle. During mid systole, when the volume is the highest, you will hear a mid-systolic click What if your patient is sitting at the edge of the bed in the clinic resting and not exercising? How can you hear the murmur of MVP – this valve that does not connect very well? I want to make the valve fall back down on itself. What is below the mitral valve? The left ventricle. What is in the left ventricle? Blood that needs to get out! What can I do to decrease the volume of blood in the heart, make the valve fall back down on itself, and increase the sound of the murmur at rest? Standing and Valsalva! Handgrip and Mitral Valve Prolapse There is just one more thing we need to cover. The ultimate apex question. Are you ready? Handgrip! With handgrip and HOCM, I could see how by increasing the volume of blood in the heart, the ventricle walls would get pushed apart, and the murmur goes away. Now we will cover handgrip and mitral valve prolapse. Okay, handgrip, so what did I say? I am sitting on the aorta. Okay. So, what is the left ventricle going to do? I just increased my afterload. This means I also increased my preload. So what did you do? You just put more blood in your left ventricle! Now you need to push that blood out, and you are pushing out more blood. The mitral valve is prolapsed and does not have a good connection. Is it going to be able to hold? No. So, will I hear the murmur of mitral valve prolapse if I do handgrip? Yes, the murmur of MVP will increase with handgrip. How do I differentiate the low-volume lovers (MVP and HOCM)? By having the patient perform handgrip. With HOCM, when handgrip is applied, you push the walls of the ventricles apart. With mitral valve prolapse and handgrip, you increase left ventricular pressure, and what happens to the mid systolic click? Oh, it is still there, and if ff anything is louder. Because what did I do? I increased the pressure in my left ventricle. Come back to this podcast and replay it again when you have a moment. This will make so much sense if you follow my physics, especially the laminar flow physics part. Remember that HOCM and MVP are low volume lovers – meaning they like low volume – that is when the murmurs sound louder. They are the opposite of mitral stenosis, mitral regurgitation, aortic stenosis, and aortic regurgitation. Those usual players like more blood against the diseased valve. Remember, in HOCM; it is not a valve issue; it is a wall issue. Mitral valve prolapse is a valve issue, but remember that the valve does not connect very well. It is still there. It works well, but the connection does not hold tight. What’s next? Next week we are going to start doing murmur questions. We have covered aortic stenosis and aortic regurgitation, we went over mitral stenosis and mitral regurgitation, we covered pulmonic stenosis and pulmonic regurgitation, we have learned about tricuspid stenosis and tricuspid regurgitation, and in this episode, we covered what I call exceptions to the rule – which is HOCM, and mitral valve prolapse. What is next? Test questions – where I will present these murmurs to you in a very ambiguous way, and you will have to figure out which murmur it is. Take care of yourselves, and stay safe. I will see you soon. Take care, Joe. Resources and links from the show View all the episodes in this series Cardiac Murmurs Made Incredibly Easy View the Mitral valve prolapse and Hypertrophic Cardiomyopathy Smarty PANCE content blueprint lessons Download your Free interactive PANCE, PANRE, and EOR Blueprint Templates Follow Smarty PANCE and The Daily PANCE Blueprint on Instagram and Facebook Check out my list of recommended PANCE and PANRE review books Sign up for the FREE Daily PANCE and PANRE email series Join the Smarty PANCE NCCPA Content Blueprint Website Get your free 8-week PANRE Blueprint study schedule and the 8-week PANCE study schedule Get your free Trello PANCE study planner Get 20% off any Picmonic membership by using this link , or if you are a Smarty PANCE member, get Picmonic for Smarty PANCE . This podcast is available on every device. You can download and listen to past FREE episodes here , on iTunes , Spotify , Google Podcasts , Stitcher , Amazon Music , and all podcasting apps. Download the Interactive Content Blueprint Checklist Follow this link to download your FREE copy of the PANCE/PANRE/EOR Content Blueprint Checklists . Print it up and start crossing out the topics you understand, marking the ones you don’t, and making notes of key terms you should remember. The PDF version is interactive and linked directly to the individual lessons on Smarty PANCE.…
T
The Audio PANCE and PANRE Physician Assistant Board Review Podcast


Welcome to episode 95 of the Audio PANCE and PANRE Physician Assistant/Associate Board Review Podcast. Join me as I cover ten PANCE, PANRE and EOR review questions from the Smarty PANCE Instagram/Facebook page and the smartypance.com board review website. Special from today’s episode: Follow Smarty PANCE and The Daily PANCE Blueprint on Instagram Follow Smarty PANCE and The Daily PANCE Blueprint on Facebook Below you will find an interactive exam to complement today’s podcast. The Audio PANCE/PANRE and EOR PA Board Review Podcast I hope you enjoy this free audio component to the examination portion of this site. The full board review course includes over 2,000 interactive board review questions and is available to all members of Smarty PANCE . You can download and listen to past FREE episodes here , on iTunes , Spotify , on Google Podcasts , Stitcher , and most podcasting apps. You can listen to the latest episode, take an interactive quiz, and download more resources below. Listen Carefully Then Take the Practice Exam If you can’t see the audio player, click here to listen to the full episode. Podcast Episode 95: Ten PANCE/PANRE and EOR Topic Blueprint Questions 1. A 52-year-old patient is admitted with a lower gastrointestinal bleed. He is given 2 units of packed red blood cells. A few hours later the patient develops a fever but has no other symptoms or changes in vital signs. Lab studies reveal no significant changes. Which of the following is the most likely diagnosis? A. Transfusion-associated circulatory overload B. Acute immune-mediated hemolytic reaction C. Transfusion-related acute lung injury D. Febrile non-hemolytic transfusion reaction E. None of the above Click here to see the answer The answer is D. Febrile non-hemolytic transfusion reaction The most common transfusion reaction is a febrile non-hemolytic transfusion reaction (FNHTR). Patients with FNHTR may present with a fever a few hours after receiving packed red blood cells (PRBCs) , which is caused by the small number of white blood cells and cytokines that can be found in each unit of PRBCs. Treatment consists of giving acetaminophen and ruling out other causes of fever (e.g., central line infection, urinary tract infection, etc). Incorrect Answers: Transfusion-associated circulatory overload (Choice A) occurs when the volume of the transfused component causes hypervolemia. Acute immune-mediated hemolytic reaction (Choice B) occurs when there are antibodies presented to blood donor antigens. Patients may present with hypotension, fever, or coagulopathic lab abnormalities. Transfusion-related acute lung injury (Choice C) is due to antibodies reacting with antigens leading to the release of mediators that causes edema in the lungs. Symptoms include fever, hypoxia, and dyspnea. VIEW BLUEPRINT LESSON Smarty PANCE Content Blueprint Review: Covered under ⇒ PANCE Blueprint Hematology ⇒ Immunologic disorders ⇒ Transfusion reaction 2. Which of the following is not a side effect of lithium? A. Hyperparathyroidism B. Hypothyroidism C. Cognitive fog D. Tremors E. All are side effects Click here to see the answer The answer is E. All are side effects Lithium is the first-line treatment for bipolar disorder. It has many side effects that you should remember, such as hypothyroidism , hyperparathyroidism , nephrogenic diabetes insipidus , cognitive fog , and tremors . Lithium levels should be monitored to avoid toxicity. Remember, bipolar disorder is characterized by the symptoms DIG FAST : D istractibility, I rritability, G randiosity, F light of ideas, A ctivity increased, S leep deficits, and T alkativeness. VIEW BLUEPRINT LESSON Smarty PANCE Content Blueprint Review: Covered under ⇒ PANCE Blueprint Psychiatry ⇒ Bipolar and related disorder Also covered as part of the Emergency Medicine EOR , Psychiatry EOR , and Family Medicine PAEA EOR topic list 3. Which of the following best explains the pathophysiology of vasovagal syncope? A. Increase in parasympathetic signals and withdrawal of sympathetic signals B. Severe narrowing of the aortic valve C. Drop in blood pressure upon standing due to inadequate peripheral vasoconstriction D. Occlusion of the pulmonary artery leading to right ventricle dysfunction E. Blood accumulation in the brain leading to compression of adjacent brain structures Click here to see the answer The answer is A. Increase in parasympathetic signals and withdrawal of sympathetic signals Vasovagal syncope is the most common cause of syncope. Syncope is essentially transient loss of consciousness due to the lack of cerebral perfusion. Vasovagal syncope is usually triggered by something , such as emotion or stress. A leading hypothesis of vasovagal syncope is that it is due to an increase in parasympathetic signals and withdrawal of sympathetic nervous signals . Incorrect Answers: A drop in blood pressure upon standing due to inadequate vasoconstriction describes neurogenic orthostatic hypotension (Choice C). Occlusion of the pulmonary artery leading to RV dysfunction is describing a pulmonary embolism , which can lead to syncope (Choice D). Blood accumulation in the brain leading to compression of brain structures is describing an intracranial hemorrhage (Choice E), which is a (rare) cause of syncope. Narrowing of the aortic valve aka aortic stenosis (Choice B) can lead to syncope, but again, this is not vasovagal syncope. VIEW BLUEPRINT LESSON Smarty PANCE Content Blueprint Review: Covered under ⇒ PANCE Blueprint Neurology ⇒ Vascular Disorders ⇒ Syncope Also covered as part of the Internal Medicine EOR , Family Medicine EOR , Emergency Medicine EOR , Pediatric EOR , and General Surgery PAEA EOR topic list 4. A 71-year-old male smoker with prostate cancer presents to your clinic complaining of unilateral leg swelling and pain. Vital signs are unremarkable. On a physical exam, the entire leg is swollen with localized tenderness along the venous system. Which of the following is the next best step? A. Order a d-dimer B. Order a duplex ultrasound C. Admit to the hospital immediately D. Reassurance and follow-up in 6 months E. Order a CT angiogram Click here to see the answer The answer is B. Order a duplex ultrasound This patient needs an ultrasound because he has a high pre-test probability for thrombosis. Ordering a d-dimer, in this case, would be inappropriate. Lots of things (e.g., cancer, trauma, etc) can elevate d-dimer. Well’s Score helps us determine a patient’s risk for DVT based on these criteria: active cancer, surgery or bedridden, calf swelling, collateral veins presence, entire leg swollen, localized tenderness along deep venous system, pitting edema, previous DVT, recent immobilization, & alternative diagnosis more likely. The main idea: d-dimer should be ordered in low-risk patients, and a venous duplex US should be ordered for high-risk patients. VIEW BLUEPRINT LESSON Covered under ⇒ PANCE Blueprint Cardiology ⇒ Vascular Disease ⇒ Venous thrombosis Also covered as part of the General Surgery PAEA EOR topic list 5. Which of the following individuals would be considered to have a positive purified protein derivative test at 6 millimeters of induration? A. 31-year-old who was in contact with a person who has active TB B. 65-year-old intravenous drug user C. 53-year-old who lives in a prison D. 90-year-old with hypertension E. None of the above Click here to see the answer The answer is A. 31-year-old who was in contact with a person who has active TB Tuberculosis (TB) is a worldwide health concern and is deadly. A purified protein derivative (PPD) test can be used to screen for TB. A PPD test is considered positive depending on the patient’s risk factors. Remember, we measure induration , NOT erythema. See below for a high-yield list of positive PPD tests (not exhaustive): > 5 mm: HIV, immunosuppressive condition, or recent contact with active TB case > 10 mm: IVDU, came from a country with a high TB prevalence (<5 y prior), health care worker, resident/employee in a high-risk congregate setting > 15 mm: everyone else VIEW BLUEPRINT LESSON Smarty PANCE Content Blueprint Review: Covered under ⇒ PANCE Blueprint Pulmonary ⇒ Infectious Pulmonary Disorders ⇒ Tuberculosis Also covered as part of the Internal Medicine EOR , Emergency Medicine EOR , and Family Medicine PAEA EOR topic list 6. A 48-year-old male presents with headaches, vision loss, enlarged jaw, and enlarged hands. Which of the following laboratory studies is the preferred initial diagnostic test? A. Beta glycoprotein 2a B. Growth hormone C. Prolactin D. Calcitonin E. Insulin growth factor 1 Click here to see the answer The answer is E. Insulin growth factor 1 The patient has acromegaly , which is an endocrine disorder characterized by excess secretion of growth hormone after epiphyseal closure. The most common cause is a pituitary adenoma . Clinical manifestations include coarsening of facial features, enlarged hands/feet, macrognathia, bitemporal hemianopsia, sleep apnea, and headaches. Insulin growth factor-1 is the best initial diagnostic test to screen for acromegaly. A CT or MRI can help confirm the presence of a pituitary adenoma. A glucose suppression can also be ordered. Definitive treatment is transsphenoidal resection of the pituitary adenoma. VIEW BLUEPRINT LESSON Smarty PANCE Content Blueprint Review: Covered under ⇒ PANCE Blueprint Endocrinology ⇒ Pituitary Disorders ⇒ Acromegaly/gigantism Also covered as part of the Internal Medicine PAEA EOR topic list 7. A 14-year-old obese male presents to the ER complaining of dull pain in the right hip and thigh for the past few weeks. No history of trauma. On physical exam, the affected leg is held in an externally rotated position and is shorter than the left leg. What is the most likely diagnosis? A. Legg Calve Perthes B. Septic arthritis C. Slipped capital femoral epiphysis D. Transient synovitis E. Pelvic stress fracture Click here to see the answer The answer is C. Slipped capital femoral epiphysis The patient most likely has slipped capital femoral epiphysis (SCPE), which is defined as displacement of the capital femoral epiphysis from the femoral neck (through the physeal plate). The classic clinical presentation is hip pain in an obese, adolescent male. Remember, it can be bilateral (~20-40%)! On physical exam, the affected leg may be externally rotated and shorter than the other leg. Cases are diagnosed via plain radiographs (AP view and lateral views of both hips). Once diagnosed with SCPE, most patients should be made non-weight-bearing and referred to an orthopedic surgeon . VIEW BLUEPRINT LESSON Smarty PANCE Content Blueprint Review: Covered under ⇒ PANCE Blueprint Musculoskeletal ⇒ Lower extremity disorders ⇒ Disorders of the hip ⇒ Slipped capital femoral epiphysis Also covered as part of the Pediatric Rotation PAEA EOR topic list 8. A 32-year-old male with sarcoidosis presents to the clinic complaining of shortness of breath, fatigue, and palpitations for a few months. His in-office EKG reveals QRS prolongation and some premature ventricular beats. A chest radiograph shows cephalization of the pulmonary vessels and cardiomegaly. Which of the following is the most likely diagnosis? A. Cryptogenic organizing pneumonia B. Hypertensive encephalopathy C. Cardiac sarcoidosis D. Acute coronary syndrome E. Pneumoconiosis Click here to see the answer The answer is C. Cardiac sarcoidosis Sarcoidosis is a multisystem granulomatous disorder; it impacts the lungs, eyes, lymph nodes, heart, skin, and nervous system. Cardiac sarcoidosis (CS) is often under-recognized and can occur as a clinical feature of sarcoidosis. Presenting symptoms may include palpitations , syncope , fatigue , & dyspnea. CS happens because granulomas infiltrate the heart, leading to conduction issues, tachyarrhythmias, cardiomyopathies, & heart failure. Holter monitoring and echocardiography are two helpful tests. The gold standard would be a heart biopsy (not routinely done because it is so invasive; cardiac MRI is preferred). Treatment is steroid therapy and anti-arrhythmic drugs. Incorrect Answers: Cryptogenic organizing pneumonia (Choice A) is a type of diffuse interstitial lung disease that typically presents with cough, fever, dyspnea, and malaise. Hypertensive encephalopathy (Choice B) manifests as headaches, vomiting, confusion, and neurological symptoms. The patient has none of these. Acute coronary syndrome (Choice D) is more likely to present with diaphoresis and crushing chest pain. Pneumoconiosis (Choice E) is a fancy word that means the accumulation of dust (e.g., coal, asbestos, etc) within the lungs. This patient has absolutely nothing in his history that suggests this. VIEW BLUEPRINT LESSON Smarty PANCE Content Blueprint Review: Covered under ⇒ PANCE Blueprint Pulmonary ⇒ Restrictive Pulmonary Disease ⇒ Sarcoidosis Also covered as part of the Internal Medicine PAEA EOR topic list 9. A 64-year-old male presents with tremors in both of his hands. He says the tremors worsen with movement and caffeine. The tremors improve with alcohol use. Which of the following is the best initial treatment option for his likely diagnosis? A. Dopamine agonist B. Beta-blocker C. Alpha antagonist D. NMDA antagonist E. None of the above Click here to see the answer The answer is B. Beta-blocker The patient likely has an essential tremo r (ET) which is the most common cause of action tremor in adults. It occurs bilaterally in both hands, commonly worsens with anxiety , and improves with alcohol use . The tremor becomes more noticeable when the hands are outstretched or during goal-directed movement (e.g., writing with a pencil). ET is a clinical diagnosis. Treatment includes beta blockers (propranolol), anticonvulsants (primidone, gabapentin, topiramate) and benzodiazepines . Propranolol and primidone are the most effective treatment options. Remember there is often a strong genetic component (family history is present 30-70% of the time). VIEW BLUEPRINT LESSON Smarty PANCE Content Blueprint Review: Covered under ⇒ PANCE Blueprint Neurology ⇒ Movement Disorders ⇒ Essential tremor Also covered as part of the Family Medicine and Internal Medicine PAEA EOR topic list 10. Which of the following is the most common cause of infective lactational mastitis? A. Escherichia coli B. Staphylococcus aureus C. Bacteroides species D. Staphylococcus epidermidis Click here to see the answer The answer is B. Staphylococcus aureus Lactational mastitis presents as a red, painful, swollen breast usually during the first three months of breastfeeding . It commonly results from poor drainage. The most common cause is Staphylococcus aureus . If it develops over 24 hours, patients may also have flu-like symptoms and a fever — this is considered “infective” lactational mastitis. Treatment includes cold compresses and complete emptying of the breasts. For infective lactational mastitis, antibiotics (e.g., clindamycin , cephalexin , dicloxacillin ) should be given. Remember, patients should NOT stop breastfeeding. VIEW BLUEPRINT LESSON Smarty PANCE Content Blueprint Review: Covered under ⇒ PANCE Blueprint Reproductive System ⇒ Breast Disorders ⇒ Mastitis Also covered as part of the Women’s Health EOR and Emergency Medicine PAEA EOR topic list Looking for all the podcast episodes? This FREE podcast series is limited to every other episode, you can download and enjoy the complete audio series by becoming a Smarty PANCE member . I will be releasing new episodes every few weeks. Smarty PANCE is now discounted, so sign up now before it’s too late! Resources and Links from the Show Download your Free interactive PANCE, PANRE, and EOR Blueprint Templates Follow Smarty PANCE and The Daily PANCE Blueprint on Instagram Follow Smarty PANCE and The Daily PANCE Blueprint on Facebook My list of recommended PANCE and PANRE review books Sign up for the FREE Daily PANCE and PANRE email series Join the Smarty PANCE NCCPA Content Blueprint Website Get your free 8-week PANRE Blueprint study schedule and the 8-week PANCE study schedule Get your free Trello PANCE study planner Get 20% of any Picmonic membership by using this link This Podcast is available on iOS and Android You can download and listen to past FREE episodes here , on iTunes , Spotify , on Google Podcasts , Stitcher , and most podcasting apps. Download the Interactive Content Blueprint Checklist Follow this link to download your FREE copy of the PANCE/PANRE/EOR Content Blueprint Checklists Print it up and start crossing out the topics you understand, marking the ones you don’t, and making notes of key terms you should remember. The PDF version is interactive and linked directly to the individual lessons on Smarty PANCE.…
Willkommen auf Player FM!
Player FM scannt gerade das Web nach Podcasts mit hoher Qualität, die du genießen kannst. Es ist die beste Podcast-App und funktioniert auf Android, iPhone und im Web. Melde dich an, um Abos geräteübergreifend zu synchronisieren.